Sie sind auf Seite 1von 250

Problem Books in Mathematics

Edited by P.R. Halmos


T. Cacoullos

Exercises in Probability

With 22 Illustrations

Springer-Verlag
New York Berlin Heidelberg
London Paris Tokyo
T. Cacoullos
Department of Mathematics
University of Athens
Athens 157 10
Greece

Editor
Paul R. Halmos
Department of Mathematics
Santa Clara University
Santa Clara, CA 95053
U.S.A.

Mathematics Subject Classification (1980): 60-01

Library of Congress Cataloging-in-Publication Data


Cacoullos, T. (Theophilos)
Exercises in probability.
(Problems in mathematics)
Translated from the Greek.
Bibliography: p.
1. Probabilities-Problems, exercises, etc. I. Title.
II. Series.
QA273.25.C33 1989 519.2'076 88-4932

Printed on acid-free paper.

Original Greek edition published in two volumes, Askiseis Theorias Pithanotiton (Exercises in
Probability) and Askiseis-Provlemata Theorias Pithanotition (Exercises-Problems in Probability
Theory), in 1971 and 1972, respectively, by Leousis-Mastroyiannis, Athens, Greece.

© 1989 by Springer-Verlag New York Inc.


Softcover reprint of the hardcover 1989
All rights reserved. This work may not be translated or copied in whole or in part without the
written permission of the publisher (Springer-Verlag, 175 Fifth Avenue, New York, NY 10010,
U.S.A.), except for brief excerpts in connection with reviews or scholarly analysis. Use in connec-
tion with any form of information storage and retrieval, electronic adaptation, computer software,
or by similar or dissimilar methodology now known or hereafter developed is forbidden.
The use of general descriptive names, trade names, trademarks, etc. in this publication, even if
the former are not especially identified, is not to be taken as a sign that such names, as understood
by the Trade Marks and Merchandise Marks Act, may accordingly be used freely by anyone.

Typeset by Asco Trade Typesetting Ltd., Hong Kong.


Printed and bound by R.R. Donnelley & Sons, Harrisonburg, Virginia.
Printed in the United States of America.

9 8 7 6 5 4 3 2 1

ISBN-13: 978-1-4612-8863-3 e-ISBN-13: 978-1-4612-4526-1


DOl: 10.1007/978-1-4612-4526-1
Preface

This problem book can serve as a companion text for an introductory or


intermediate-level one- or two-semester probability course, usually given to
undergraduate or first-year graduate students in American universities. Those
who will benefit most are those with a good grasp of calculus and the
inclination and patience required by mathematical thinking; and yet many
more, with less formal mathematical background and sophistication, can
benefit greatly from the elements of theory and solved problems, especially of
Part I. In particular, the important part of discrete probabilities with problems
of a combinatorial nature, relating to finite sample spaces and equally likely
cases, may be challenging and entertaining to the student and amateur alike,
through the spectrum from mathematics to the social sciences.
For some time, since I have been teaching courses in probability theory
both to undergraduates at the University of Athens (School of Science) and
graduate students in American universities, I have noted the need of students
for a systematic collection of elementary and intermediate-level exercises in
this area. The relevant international bibliography is rather limited. The aim
of the present collection is to fill such a gap.
This book is based on the two volumes of the Greek edition entitled
Exercises in Probability by the same author. Of the 430 exercises contained in
the original Greek edition (1972), 329 (with solutions) were selected for the
present translation. A small number of problems in stochastic processes is not
included here, since it is felt that a minimal treatment of this important area
requires more theory and a larger number of problems. However, an adden-
dum of over 160 exercises and certain complements of theory and problems
(with answers or hints) from the recent (1985) second Greek edition is included
here as Supplement I: Miscellaneous Exercises, and as Supplement II: Comple-
ments and Problems.
VI Preface

In addition to the Supplements, the present book is divided into three


parts. Part I: Elementary Probabilities (Chapters 1-4), Part II: Advanced
Topics (Chapter 5-10), and Part III: Solutions (to the exercises of Part I and
Part II). In most chapters, the exercises are preceded by some basic theory
and formulas.
In Part I (with 170 exercises) emphasis is given to classical probabilities
(equally likely cases). It concerns mainly one-dimensional, discrete, and con-
tinuous distributions (Chapter 2). The expected value, variance, and moments
of a distribution are treated in Chapter 3. Chapter 4, in spite of its elementary
nature, contains quite a few challenging problems. In general, problems of a
higher level of difficulty are marked with an asterisk throughout the text.
Part II deals with more advanced topics: multivariate distributions (Chap-
ter 5), generating and characteristic functions (Chapter 6), distribution of
functions of random variables (Chapter 7), and Laws of Large Numbers and
Central Limit Theorems (Chapter 8). Chapter 9 deals with special topics:
stochastic inequalities, geometrical probabilities, and applications of differ-
ence equations in probability. The last chapter (Chapter to) contains general
overview exercises.
Clearly, a collection of problems owes a lot to the relevant bibliography.
The exercises come from many sources, some of them are new, a large number
are variations of more or less standard exercises and problems of elementary
or intermediate level, and a few are based on research papers. Such problems,
as well as complements, are also marked with an asterisk. The level and
sources are indicated in the select Bibliography.
It is hoped that the present edition will make proportionately as many
friends as it did among the ten thousand or so students all over Greece who
have used the text since its first edition in 1971-72. Naturally, one benefits
from such a text by first really trying to solve a problem, before falling into
the "temptation" of looking up the solution. That is why solutions are given
separately, in the last part of the book.
Thanks are due to my colleagues Dr. Ch. Charalambides and Dr. Ourania
Chrysaphinou for preparing some solutions for the first Greek edition, as well
as preparing a first draft of a major part of this translation.

New York T. CACOULLOS


June, 1987 Visiting Professor
Columbia University
Contents

Preface v

Part I Elementary Probabilities

CHAPTER I
Basic Probabilities. Discrete Spaces 3
Basic Definitions and Formulas 3
Exercises 7
1. Sets. Events: 1-8 7
2. Combinatorics: 9-16 8
3. Properties of Binomial Coefficients: 17-25 8
4. Properties of Probability: 26-34 9
5. Classical Probabilities. Equally Likely Cases: 35-53 10
6. Independent Events. Conditional Probability: 54- 79 13

CHAPTER 2
Distributions. Random Variables 17
Elements of Theory 17
Exercises 20
1. Discrete Distributions: 80-89 20
2. Continuous Distributions: 90-100 21

CHAPTER 3
Expectation. Variance. Moments 24
Elements of Theory 24
Exercises 26
1. Theoretical Exercises: 101-113 26
2. Mean and Variance: 114-125 28
Vlll Contents

CHAPTER 4
General Problems: 126-170 30

Part II Advanced Topics 37

CHAPTER 5
Multivariate Distributions 39
Elements of Theory 39
Exercises: 171- 190 43

CHAPTER 6
Generating Functions. Characteristic Functions 47
Elements of Theory 47
Exercises: 191-215 51

CHAPTER 7
Distribution of Functions of Random Variables 55
Elements of Theory 55
Exercises: 216-250 58

CHAPTER 8
Limit Theorems. Laws of Large Numbers. Central Limit Theorems 63
Elements of Theory 63
Exercises: 251-269 67

CHAPTER 9
Special Topics: Inequalities, Geometrical Probabilities,
Difference Equations 70
Elements of Theory 70
A. Inequalities 70
B. Geometrical Probabilities 72
C. Difference Equations 72
Exercises 74
A. Inequalities: 270-282 74
B. Geometrical Probabilities: 283-289 75
C. Difference Equations: 290-300 76

CHAPTER 10
General Exercises: 301-329 78

Supplements 83

SUPPLEMENT I
Miscellaneous Exercises: 1-1-1-56 85
Contents ix

SUPPLEMENT II
Complements and Problems 94
1. Multivariate Distributions: 1.1-1.27 94
2. Generating Functions: 2.1- 2.22 101
3. Transformation of Random Variables: 3.1-3.15 106
4. Convergence of Random Variables: 4.1-4.19 109
5. Miscellaneous Complements and Problems: 5.1-5.29 115

Part III Solutions 123

Solutions: 1-329 125

Bibliography 243
Index 245
PART I

ELEMENTARY PROBABILITIES
CHAPTER 1

Basic Probabilities. Discrete Spaces

Basic Definitions and Formulas


1. Sample space 0: The totality of possible outcomes of a random (chance)
experiment.
The individual outcomes are called elementary or simple events or points
(cases in the classical definition of probability by Laplace).
2. Discrete sample space 0: 0 is at most a denumerable set of points.
3. Events: Subsets* of O.
4. Union of n events AI"'" An' denoted by Al U A z ... u An, is the realiza-
tion of at least one of the events AI' ... , An"
5. Realization or occurrence of an event A means the appearance of an
element (point) in A.
6. Intersection or product of two events A and B, denoted by A n B or AB,
is the simultaneous realization of both events A and B. Similarly for n events.
7. Complement of A, denoted by A Cor A', means A does not occur.
8. The difference of two events A and B is defined by A - B = ABC, i.e., A
occurs but not B. Thus A = 0 - A.
C

9. Sure or certain event: The sample space O.


10. Impossible event: The complement of the sure event, i.e., the empty set.

* Rigorously, measurable subsets of n, i.e., members of a so-called Borel or a-field B(n) on n: a


family of subsets of n containing n itself and closed under the set operations of complementation
and countable union.
4 1. Basic Probabilities. Discrete Spaces

11. Probability. A probability (function) P is a set function on 0 (strictly


on the Borel field B(O) on 0) which satisfies the following three axioms of
Kolmogorov:
(I) P(O) = 1.
(II) For every event A, P(A) ~ o.
(III) For every sequence of mutually exclusive events
i #j,

12. (a) Equally likely cases. Let 0 be a finite sample space with N elements
(points) WI' W z, ... , W N ; if
1
P[{w;}] = N' i = 1, ... , N,

then the Wi are called equally likely elementary events (cases).


(b) Laplace definition of probability. Let A c 0, where the points of 0 are
assumed equally likely. Then
peA] = number ofpo~nts ~n A
number of POllltS III 0
number of favorable cases to A
number of cases in 0

13. Basic formulas for probabilities.


(i) P(A C ) = 1 - P(A).
(ii) P(A - B) = P(A) - P(AB).
(iii) Addition theorem of probabilities: P(A u B) = P(A) + P(B) - P(AB).

Poincare's theorem. For n events A I ' ... , A.,


P(A I U Az U··· u A.) = SI - S2 + ... + (_1)n- I S., (1.1)

where, for each k = 1,2, ... , n, Sk is defined by

Sk = L
it <i 2 <'" <i k
P(A i1 A i2 ••• Ai.)'

i.e., the summation is over all combinations of the n events taken k at a time
(k=I, ... ,n).

14. Conditional probability of B given A:

P(BIA) = P(AB)
P(A) ,
P(A) > o.

15. Multiplication formula for probabilities:


P(AB) = P(BIA)P(A) = P(AIB)P(B).
Basic Definitions and Formulas 5

In general,

16. Independent events (statistically or stochastically):


(a) Two events A, B: if P(AB) = P(A)P(B).
(b) n events A 1, ... , An: if P(A it ... Ai.} = P(A it )··· P(A i.} for 1 ::;; i1 < i2 <
... < ik ::;; n, 2 ::;; k ::;; n.

17. Independent experiments. Let E 1, E 2, ... , En be n chance experiments


and 0 1 , ... , On the corresponding sample spaces. The experiments are called
stochastically or statistically independent or simply independent if for every
Ai c ili (i = 1,2, ... , n),

Remark. Physically independent experiments are assumed statistically in-


dependent, e.g., successive throws of a coin, a die, etc.

18. Total probability formula. If Bi n Bj = 0, i =f. j (i,j = 1, ... , n) and A c


(B1 U B2 U ... u Bn), then

(1.3)

19. Bayes's theorem (formula). As in Formula 18,

P B A _ P(AIBi)P(B;)
i = 1, ... , n. (1.4)
( d ) - P(AIBdP(Bd + ... + P(AIBn)P(Bn)'

20. Combinatorics
Binomial coefficient. For every real x and any positive integer k we define

( x) = x(x - 1)···(x - k + 1).


k k!
In particular, when x is a positive integer, then

(x)k =
x!
k! (x - k)!
equals the number of combinations of x distinct objects taken k at a time.
Pascal's triangle is based on the recursive relation

21. Let S be a set (population) of N distinct elements.


(a) A subset of k elements (a k-tuple) is called a sample of size k (a combina-
tion).
(b) An ordered k-tuple of elements of S is called an ordered sample of size
6 I. Basic Probabilities. Discrete Spaces

k (a permutation). There are

(N)
k N'
k! = (N _. k)! == (Nh = N(N - 1) .. ' (N - k + 1)
permutations of N objects taken k at a time.
(c) S can be divided into k subsets, the first subset (subpopulation) con-
taining r l elements, the second r2' etc., the kth rk objects in

(1.5)

ways; (1.5) is called a multinomial coefficient.

22. Placing balls into cells.


(a) s distinguishable balls can be placed into n cells in n S ways.
(b) s nondistinguishable balls can be placed into n cells in

(1.6)

°
ways; that is, also the number of ordered n-tuples (rl' r2, ... , rn) of integers
rj Z which are solutions of the equation
rl + r2 + ... + rn = s.
The numbers rj are referred to as the (cell) occupancy numbers (see Exercise 44).

23. Sampling without replacement. From an urn containing W white balls


and B black balls, n balls are drawn (one after another or at once) at random.
The probability Pr that r white balls are drawn is given by

N= W+B, (1.7)

max(O, n - B) :.::: r :.::: min(n, W).


The probabilities Pr define the so-called hypergeometric distribution.

24. Multiplication principle. If an "operation" A I can be performed in n I


ways, another "operation" A2 in n2 ways, etc., the kth "operation" in nk ways,
then the k "operations" can be performed, one after another, in n l n 2 .. • nk
ways.
This is equivalent to the proposition: If Aj consists of nj points (i = i, ... , k),
then the size (number of points) of the cartesian product
Al x A2 x .. · x Ak={(xI,,,,,xk):xjEAj,i= 1, ... ,k}
is equal to n l n 2 ••• nk •
Exercises 7

25. The principle of mathematical induction. For every integer n let Pn be a


proposition which is either true or false, i.e., Pn may be true for some nand
false for other values of n; if, (i) PI is true, (ii) for every n, Pn true => Pn +1 true,
then Pn is true for every n.

Exercises
1. Sets. Events

1. Let n denote the totality of students at the University of Athens and AI'
A 2 , A 3, A 4 , the sets offreshmen, sophomores,juniors, and seniors, respectively.
Moreover, let F denote the set of female students and C the set of Cypriot
students. Express in words each of the following sets:
(a) (AI U A 2 ) (\ F; (b) FC'; (c) Al FC; (d) A3FC'; (e) (AI U A 2 )CF.

2. Give the simplified forms of the sets:


(a) (A u B) (\ (A u C); (b) (A u B) (\ (A' u B); (c) (A u B) (\ (A' u B) (\
(A' u B').

3. Express each of the following events in terms of the events A, B, and C,


and the operations of complementation, union, and intersection:
(a) at least one of the events A, B, C occurs;
(b) at most one of the events A, B, C occurs;
(c) none of the events A, B, C occurs;
(d) all three events occur;
(e) exactly one of the events A, B, C occurs;
(f) A and B occur but not C;
(g) A occurs, if not then B does not occur either.
Give the Venn diagram for each of the above.
4. Let n denote the sample space corresponding to the chance experiment
of tossing a coin three times. Let A be the event that heads appear exactly
twice, let B be the event that at least two heads appear, and let C the event
that heads appear when tails have appeared at least once.
(a) Give the elements of A, B, C;
(b) Describe the events: (i) A'B; (ii) A'B'; (iii) AC.
In each of the Exercises 5-8, give the number of points in the sample space
n and the points of the events defined therein.
5. A family has 4 children. Let the events: A: boys and girls alternate, B:
the first and fourth child are boys, C: as many boys as girls, and D: three
successive children of the same sex.
6. A salesman is arranging his schedule for visiting each of three towns a,
b, c, twice. A: the first and last visit are in a.
8 1. Basic Probabilities. Discrete Spaces

7. Certain diseases cause cell disorders which can be distinguished into


four categories. Blood tests are conducted on each of 4 patients and in each
case the category is noted. Let A: the patients belong to the same category, B:
two patients belong to the same category.
8. An elevator carries two persons and stops at three floors. Let A: they
get off at different floors, B: one gets off at the first floor.

2. Combinatorics
9. In how many ways can a lady having 10 dresses, 5 pairs of shoes, and 2
hats be dressed?
10. In how many ways can we place in a bookcase two works each of three
volumes and two works each offour volumes, so that the volumes ofthe same
work are not separated?
11. In how many ways can r objects be distributed to n persons if there is
no restriction on the number of objects that a person may receive?
12. In how many ways can 5 boys and 5 girls be seated around a table so
that no 2 boys sit next to each other?
13. Eight points are chosen on the circumference of a circle. How many
chords can be drawn by joining these points in all possible ways? If the 8
points are considered vertices of a polygon, how many triangles and how many
hexagons can be formed?
14. In how many ways can n persons be seated around a table if 2 arrange-
ments are regarded as the same when each person has the same right and left
neighbors? At a dinner at which the n persons are seated randomly, what is
the probability that a specific husband is seated next to his wife?
15. In how many ways can 20 recruits be distributed into 4 groups each
consisting of 5 recruits? In how many ways can they be distributed into 4
camps, each camp receiving 5 recruits?
16. Using 7 consonants and 5 vowels, how many words consisting of 4
consonants and 3 vowels can we form?

3. Properties of Binomial Coefficients


17. Show that for every x and for every positive integer n:

(i) C~ 1) G) (x: I}
+ =

(ii) (x)n (xn-l


= - 1) + (xn-l - 1).
- 2.) + .. , + (nn-1
Exercises 9

18. If n, m, r, are positive integers prove that

(~)(n ~ m) + (7)(: =7) + ... + (7)(n ~ m) = (;).


19. For every integer n ~ 2, show the following:

(i) 1 - G) + (;) + ... + (-It(:) = 0;

(ii) G) + 2(;) + 3(;) + ... = n2"-I;

(iii) 2.1.(;) + 3.2-(;) + 4.3.(:) + ... = n(n _1)2"-2.


20. Using Pascal's triangle, show that for every a and positive integers r, n:
(i) i (a - k) = (a ++ 11) _(a +- 1n);
k=O r r r

(ii) i (_If,(a)=(_I)"(a-l).
k=O k n
21. Using the result of Exercise 18, show that

22. Using the result of Exercise 21, prove that

Jo" (v!)2[(n(2n)!- V)!]2 =


(2n)2
n .

23. Using the binomial theorem, show by induction that

i (n)( _1)k-1 = i !.
k=1 k k k=1 k
24. As in Exercise 23, show that for positive integers rand n

i (k +r -r-1 1) (n +r r).
k=O
=

Show that this is a special case of Problem 20(ii).


25. In the expansion of (1 + x)" where x> 0 and n is a positive integer, let
a k be the term containing Find the values of k for which ak becomes
Xk.
maximum. What is the maximum term of (1 + e)IOO?

4. Properties of Probability
26. Given peA) = 1/3, PCB) = 1/4, P(AB) = 1/6, find the following prob-
abilities:
peA'), peA' u B), peA u B'), P(AB'), peA' u B').
10 1. Basic Probabilities. Discrete Spaces

27. Given P(A) = 3/4 and P(B) = 3/8, show that:


(a) P(A u B) ;;:: 3/4,
(b). 1/8 ~ P(AB) ~ 3/8. Give inequalities analogous to (a) and (b) for
P(A) = 1/3 and P(B) = 1/4.
28. For any two events A and B show that:
P(AB) - P(A)P(B) = P(A')P(B) - P(A' B) = P(A)P(B') - P(AB').

29. By induction show that for arbitrary events Ai' A z,"" Am the following
inequality (due to Bonferoni) holds:
n n
P(A 1 A z ··· An) ;;:: L P(A;) -
;=1
(n - 1) = 1 - L P(Af).
;=1

30. For any three events A, B, and C, show that (cf. (1.1»
PEA u B u C] = P(A) + P(B) + P(C) - P(AB) - P(AC)
- P(BC) + P(ABC).
Application: The percentages of students who passed courses A, B, and C
are as follows: A: 50%, B: 40%, C: 30%, A and B: 35%, Band C: 20%, and
15% passed all three courses. What is the percentage of students who succeeded
in at least one of the three courses?
31. A box contains balls numbered 1,2, ... , n. A ball is drawn at random:
(a) What is the probability that its label number is divisible by 3 or 4?
(b) Examine the case in (a) as n -+ 00.
32. In terms of P(A), P(B), P(C), P(AB), P(AC), P(BC), and P(ABC) express,
for k = 0, 1,2,3, the probabilities that:
(i) exactly k of the events A, B, and C, occur;
(ii) at least k of the events A, B, C occur.
33. By induction prove Poincare's theorem, i.e., formula (1.1).
34. * If d(A, B) = P(A 6. B), show that d has all the properties of a distance
function; the symmetric difference A 6. B between two sets A and B is defined
by A 6.B = AB' u A'B.

5. Classical Probabilities. Equally Likely Cases


35. Three winning tickets are drawn from an urn of 100 tickets. What is
the probability of winning for a person who buys: (a) 4 tickets? (b) only one
ticket?
36. A bakery makes 80 loaves of bread daily. Ten of them are underweight.
An inspector weighs 5 loaves at random. What is the probability that an
underweight loaf will be discovered?
Exercises 11

37. Find the probability that among seven persons:


(a) no two were born on the same day of the week (Sunday, Monday, etc.);
(b) at least two were born on the same day;
(c) two were born on a Sunday and two on a Tuesday.
38. A group of 2N boys and 2N girls is randomly divided into two equal
groups. What is the probability that each group has the same number of boys
and girls?
39. The coefficients a, b, c of the quadradic equation ax 2 + bx + c = 0 are
determined by throwing a die three times. Find the probabilities that: (a) the
roots are real; (b) the roots are complex.
40. In the game of poker a "hand of cards" means 5 cards randomly
selected (without replacement) from a deck of 52 cards. What is the probability
that a hand of cards:
(a) consists of an ace, a queen, a jack, a king, and a ten of the same suit?
(b) contains 4 cards of the same denomination (aces, etc.)?
(c) consists of cards with consecutive values, except aces, jacks, kings, and
queens?
41. In bridge, "a hand of cards" consists of 13 cards drawn at random
(without replacement) from a deck of 52 cards (the 52 cards are equally
distributed to four players). Find the probabilities that "a hand of cards" will
contain:
(i) Vi clubs, V2 spades, and V3 diamonds;
(ii) V aces (v = 0, 1, ... ,4);
(iii) Vi aces and V2 kings.
In a bridge game find the probabilities that:
(a) each player has an ace;
(b) some player has all the aces;
(c) some player has Vi aces and his partner V2 aces (Vi + V2 :s; 4).
42. Six girls are to enter a dance with 10 boys to form a ring so that every
girl is between two boys:
(a) What is the probability that some specified boy remains between 2
boys?
(b) A spectator notices that a certain girl enters next to a certain boy. Is it
random?
43. Five letters are selected at random one after another from the 26 letters
of the English alphabet; (a) with replacement, (b) without replacement. Find;
for each of the cases (a) and (b), the probabilities that the word formed: (i)
contains an "a", (ii) consits of vowels, (iii) is the word "woman".
44. * Prove that the number of ways in which r indistinguishable balls can
be distributed in n cells (or, equivalently, the number of different solutions of
12 1. Basic Probabilities. Discrete Spaces

the equation Xl + X 2 + '" + Xn = r where Xi 2 0 (i = 1, 2, ... , n) are integers),


~M~ (
n+ r-1 1) = (n + r- 1) .
n - r

45. * Consider r indistinguishable balls randomly distributed in n cells


(Bose-Einstein statistics). What is the probability that exactly m cells remain
empty?
46. From an ordinary deck of 52 cards, cards are drawn successively until
an ace appears. What is the probability that the first ace will appear: (a) at the
nth draw? (b) after the nth card?
47.* Birthday problem. In a classroom there are v students.
(a) What is the probability that at least two students have the same
birthday?
(b) What is the minimum value of v which secures probability 1/2 that at
least two have a common birthday?
48. * N letters are placed at random in N envelopes. Show that the prob-
ability that each letter will be placed in a wrong envelope is t
k-2
(_I)k (k\)'
.

49.* An urn contains nr balls numbered 1,2, ... , n in such a way that r
balls bear the same number i for each i = 1, 2, ... , n. N balls are drawn at
random without replacement. Find the probability that, (a) exactly m of the
numbers will appear in the sample, (b) each of the n numbers will appear at
least once.
50. * (Continuation). Balls are drawn until each of the numbers 1, 2, ... , n
appears at least once. What is the probability that m balls will be needed?
51. N men run out of a men's club after a fire and each takes a coat and a
hat. Prove that:
(a) the probability that no one will take his own coat and hat is
N k(N - k)!
k~l (-1) N! kl;
(b) the probability that each man takes a wrong coat and a wrong hat is

[ t (-I)k~J2.
k-2 k.
52.* Suppose every packet of the detergent TIDE contains a coupon
bearing one of the letters ofthe word TIDE. A customer who has all the letters
of the word gets a free packet. All the letters have the same possibility of
appearing in a packet. Find the probability that a housewife who buys 8
packets will get: (i) one free packet, (ii) two free packets.
53.* An urn contains Nl white and N2 black balls. When two balls are
randomly drawn the probability that both be white is 1/2.
Exercises 13

(a) What is the minimum value of Nl?


(b) What is the minimum value of Nl when N2 is an even number?
(c) What is the minimum value of the total number N = Nl + N2 of balls
in the urn?

6. Independent Events. Conditional Probability


54. If the events A, B are independent, show that the pairs (A, B'), (A', B),
(A', B') also consist of independent events.

55. If the events A, B, and C are mutually independent, then the pairs
(A, BC), (B, AC), (C, AB) also consist of independent events.

56. Suppose that for the independent events A, B, and C we have peA) = a,
peA u B u C) = 1 - b, P(ABC) = 1 - c, and P(A'B'C) = x. Prove that the
probability x satisfies the equation
ax 2 + [ab - (1 - a)(a - c - I)Jx + b(1 - a)(1 - c) = 0.

Hence conclude that


(1 - a)2 + ab
c>-----
1- a
Moreover, show that
(1 - c)(x + b) x
PCB) = , P(C) = --b'
ax x+
57. Let the events A l , A 2 , ... , An be independent and peA;) = p (i = 1,
2, ... , n). What is the probability that:
(a) at least one of the events will occur?
(b) at least m of the events will occur?
(c) exactly m of the events will occur?
58. A die is thrown as long as necessary for an ace or a 6 to turn up. Given
that no ace turned up at the first two throws, what is the probability that at
least three throws will be necessary?
59. A parent particle can be divided into 0, 1, or 2 particles with prob-
abilities 1/4, 1/2, 1/4, respectively. It disappears after splitting. Beginning with
one particle, the progenitor, let us denote by Xi the number of particles in the
i generation. Find, (a) P(X 2 > 0), (b) the probability that Xl = 2 given that
X 2 = 1.

60. An urn contains n balls numbered 1, 2, ... , n. We select at random r


balls, (a) with replacement, (b) without replacement. What is the probability
that the largest selected number is m?
61. Two athletic teams A and B playa series of independent games until
one of them wins 4 games. The probability of each team winning in each game
14 1. Basic Probabilities. Discrete Spaces

equals 1/2. Find the probability that the series will end, (a) in at most 6 games,
(b) in 6 games given that team A won the first two games.
62. It is suspected that a patient has one of the diseases AI, A 2 , A 3 • Suppose
that the population percentages suffering from these illnesses are in the ratio
2: 1 : 1. The patient is given a test which turns out to be positive in 25% of the
cases of AI, 50% of A 2 , and 90% of A 3 • Given that out of three tests taken
by the patient two were positive, find the probability for each of the three
illnesses.
63. The population of Nicosia (Cyprus) is 75% Greek and 25% Turkish.
20% of the Greeks and 10% of the Turks speak English. A visitor to the town
meets someone who speaks English. What is the probability that he is a Greek?
Interpret your answer in terms of the population of the town.
64. Two absent-minded room mates, mathematicians, forget their um-
brellas in some way or another. A always takes his umbrella when he goes
out, while B forgets to take his umbrella with probability 1/2. Each of them
foregets his umbrella at a shop with probability 1/4. After visiting three shops
they return home. Find the probability that:
(a) they have both umbrellas;
(b) they have only one umbrella;
(c) B has lost his umbrella given that there is only one umbrella after their
return.
65. Consider families of n children and let A be the event that a family has
children of both sexes, and let B be the event that there is at most one girl in
the family. Show that the only value of n for which the events A and Bare
independent is n = 3, assuming that each child has probability 1/2 of being a
boy.
66. At the college entrance examination each candidate is admitted or
rejected according to whether he has passed or failed the test. Of the candidates
who are really capable, 80% pass the test; and of the incapable, 25% pass the
test. Given that 40% of the candidates are really capable, find the proportion
of capable college students.
67. Huyghens problem. A and B throw alternately a pair of dice in that
order. A wins ifhe scores 6 points before B gets 7 points, in which case B wins.
If A starts the game what is his probability of winning?
68. Three players PI' P2 , and P3 throw a die in that order and, by the rules
of the game, the first one to obtain an ace will be the winner. Find their
probabilities of winning.
69. N players AI' A 2 , ••. , AN throw a biased coin whose probability of
heads equals p. Al starts (the game), A2 plays second, etc. The first one to
throw heads wins. Find the probability that Ak (k = 1,2, ... , N) will be the
winner.
Exercises 15

70. Urn A contains WI white balls and bi black balls. Urn B contains W z
white balls and bz black balls. A ball is drawn from A and is placed into B,
and then a ball is transferred from B to A. Finally, a ball is selected from A.
What is the probability that the ball will be white?
71. Ten percent of a certain population suffer from a serious disease. A
person suspected of the disease is given two independent tests. Each test makes
a correct diagnosis 90% of the time. Find the probability that the person really
has the illness:
(a) given that both tests are positive;
(b) given that only one test is positive.
72. A player randomly chooses one of the coins A, B. Coin A has prob-
ability of heads 3/4, and coin B has probability of heads 1/4. He tosses the
coin twice.
(a) Find the probability that he obtains: (i) two heads, (ii) heads only once.
(b) Instead of the above procedure, suppose the player can choose an
unbiased (symmetric) coin, which he tosses twice. Which procedure should he
follow in order to maximize the probability of at least one head?
73. Urn A contains 5 black balls and 6 white balls, and urn B contains 8
black balls and 4 white balls. Two balls are transferred from B to A and then
a ball is drawn from A.
(a) What is the probability that this ball is white?
(b) Given that the ball drawn is white, what is the probability that at least
one white ball was transferred to A?
74. The Pap test makes a correct diagnosis with probability 95%. Given
that the test is positive for a lady, what is the probability that she really has
the disease? How do you interpret this? Assume that one in every 2,000
women, on average, has the disease.
75. A secretary goes to work following one of three routes A, B, C. Her
choice of route is independent of the weather. If it rains, the probabilities of
arriving late, following A, B, C, are 0.06, 0.15, 0.12, respectively. The corre-
sponding probabilities, if it does not rain, are 0.05, 0.10, 0.15.
(a) Given that on a sunny day she arrives late, what is the probability that
she took route C? Assume that, on average, one in every four days is rainy.
(b) Given that on a day she arrives late, what is the probability that it is a
rainy day?
76. At an art exhibition there are 12 paintings of which 10 are original. A
visitor selects a painting at random and before he decides to buy, he asks the
opinion of an expert about the authenticity of the painting. The expert is right
in 9 out of 10 cases on average.
(a) Given that the expert decides that the painting is authentic, what is the
probability that this is really the case?
(b) If the expert decides that the painting is a copy, then the visitor returns
16 1. Basic Probabilities. Discrete Spaces

it and chooses another one; what is the probablity that his second choice is
an original?
77. What is the conditional probability that a hand at poker (see Exercise
40) consists of spades, given that it consists of black cards?
Note: Of the 52 cards of an ordinary deck, the 13 spades and 13 clubs are
black whereas the 13 hearts and 13 diamonds are red.
78. The probability Pk that a family has k children is given by Po = Pl = a,
Pk = (l - 2a)2-(k-l) (k ~ 2). It is known that a family has two boys. What is
the probability that:
(a) the family has only two children?
(b) the family has two girls as well?
79. An amplifier may burn one or both of its fuses if one or both of its
tubes are defective. Let the events:
Ai: only tube i is defective (i = 1, 2);
A3: both tubes are defective;
B/ fuse j burns out (j = 1, 2);
B3: both fuses burn out.
The conditional probabilities of burning the fuses (given the state of the
tubes, i.e., the P[BjIA;]), appear in the following table:

i/j Bl B2 B3
Al 0.7 0.2 0.1
A2 0.3 0.6 0.1
A3 0.2 0.2 0.6

Find the probabilities that:


(a) both tubes are defective given that both fuses were burnt out;
(b) only tube 2 is defective given that both fuses were burnt out;
(c) only tube 2 is defective given that at least one fuse was burnt out.
The events A 1 , A 2, A3 have probabilities 0.3, 0.2, 0.1, respectively.
CHAPTER 2

Distributions. Random Variables

Elements of Theory
1. A function (strictly measurable) X(w) defined on the sample space
n = {w} is called a random or stochastic variable.
A random variable is called discrete ifit takes on (with positive probability)
at most a countable set of values, that is, if there exists a sequence Xl' X2' ...
with 00

P[X = x;] = Pi > 0 and I. Pi = 1.


i=l
(2.1)

The sequence {Pn, n = 1, ... } defines the so-called probability (mass) function
or frequency function of X.
A random variable X is called continuous (strictly, absolutely continuous) if
for every real c there exists a function f (almost everywhere continuous) such
that
P[X :::; c] = foo f(x) dx. (2.2)

The function f is called the probability density function or simply the density
of X. It follows that:

2. The distribution function or cumulative distribution function of a random


variable X is defined by
F(x) == P[X :::; x] for every real x.
Thus for a continuous random variable,

F(x) = f~oo f(t) dt for every x, (2.3)


18 2. Distributions. Random Variables

while for a discrete random variable


F(x) = I P[X = x;J.

From (2.3), it follows that if F is differentiable at x then


dF(x) _ f(
dx - x).

Moreover, the probability differential f(x) dx can be interpreted as


f(x) dx = P[x < X < x + dx].
For a continuous random variable X, we have
P[a < X < fJ] = P[a ::; X ::; fJ] = P[a < X ::; fJ] = P[a ::; X < fJ],
i.e., no point carries positive probability and every set A of the real line of zero
length carries zero probability.

3. The main discrete distributions


(i) The hypergeometric (cf. (1.7)) with parameters N, n, and p;

q = 1 - p. (2.4)

(ii) The binomial with parameters n (number of independent Bernoulli


trials) and p (probability of success on each trial):

k = 0, 1, ... , n, q = 1- p. (2.5)

(iii) The Poisson distribution with parameter 2:


2k
P[X = k] == p(kI2) = e-.l. k! ' k = 0,1,2, .... (2.6)

(iv) The geometric with parameter p (the number of Bernoulli trials required
before or including the first success):
P[X = k] = pqk, k = 0, 1,2, ... ,
(2.7)
or P[X = k] = pqk-l, k = 1,2,3, ....
(v) The Pascal distribution with parameters p and r (number of failures
before the rth success in a sequence of Bernoulli trials):

P[X=k]= (r+ k - 1)
r-l p'qk= (- r)k p'(-q)k, k = 0, 1, .... (2.8)

If r is simply positive (not necessarily an integer) then (2.8) is also called a


negative binomial distribution.
Elements of Theory 19

4. The main continuous distributions


(i) Uniform or rectangular in the interval (oc, P) with density
1
f(x) = -p-, oc < x < p.
-oc
(ii) Exponential (or negative exponential) with density
f(x) = ()e- OX , x> O.
(iii) Normal or Gaussian or Gauss-Laplace with parameters Ji (mean) and
(12 (variance) denoted by N(Ji, (12) with density

1 [1
f(x) = . - - exp
(1fo
--
2
(x - Ji)2 ]
(1
2 ' -00 <x< 00.

(iv) Laplace distribution (bilateral or double exponential) with density


f(x) = ie-lxi, - 00 <x< 00. (2.9)
(v) p distribution with parameters p > 0 and q > 0 with density

r(p + q) P-l(l _
P( xl p, q ) = r(p)r(q) X)q-l 0< x < 1,
x ,

_ --
- 1 x p-1(1 - x
)q-l. (2.10)
B(p, q)
where the r function is defined for every p > 0 by

r(p) = I'" xp-1e- x dx,

and the B function is defined for every p > 0, q > 0 by

B(p, q) = II x p - l (1 - X)q-l dx.

(vi) The gamma (r) distribution with parameters it > 0 (scale parameter)
and s > 0 (shape parameter) with density
its
y( xlit s) = __ e-.<xx s - l x> O. (2.11 )
'r(s) ,

When it = 1/2 and 2s = v = integer, the gamma (r) distribution is called a X2


(chi-square) distribution with v degrees of freedom, denoted by X~.
(vii) Cauchy distribution with density

-00 <x< 00. (2.12)

(viii) Student's or t distribution with v degrees of freedom. The distribu-


tion of the ratio Z/~ where Z is N(O, 1) independent of the X~. It has
20 2. Distributions. Random Variables

density

r(~) 1
fv(t) = (v)(
for - 1 +-
t 2 )(1+V)/2'
-00 <t< 00.

2 v
(ix) F or Snedecor's distribution with m and n degrees of freedom. The
distribution of the ratio nx;,/mx~ where the X;' and X~ are independent. It has
density
m+ n) (m)m/2
r (-2~ x(m/2)-1
f(x) = (m) (n)
r - r -
~ (1 +-xm )(m+n)/2'
x> O.
2 2 n

Exercises
1. Discrete Distributions

80. An urn contains 7 white balls numbered 1,2, ... , 7 and 3 black balls
numbered 8, 9, 10. Five balls are randomly selected, (a) with replacement, (b)
without replacement.
For each of the cases (a) and (b) give the distribution:
(I) of the number of white balls in the sample;
(II) of the minimum number in the sample;
(III) of the maximum number in the sample;
(IV) of the minimum number of balls needed for selecting a white ball.
81. (Continuation). As in Problem 80 with the black balls numbered 1,2,
and 3.
82. A machine normally makes items of which 4% are defective. Every hour
the producer draws a sample of size 10 for inspection. If the sample contains
no defective items he does not stop the machine. What is the probability that
the machine will not be stopped when it has started producing items of which
10% are defective.
83. One per thousand of a population is subject to certain kinds of accident
each year. Given that an insurance company has insured 5,000 persons from
the population, find the probability that at most 2 persons will incur this
accident.
84. A certain airline company, having observed that 5% of the persons
making reservations on a flight do not show up for the flight, sells 100 seats
on a plane that has 95 seats. What is the probability that there will be a seat
available for every person who shows up for the flight?
85. Workers in a factory incur accidents at the rate of two accidents per
Exercises 21

week. Calculate the probability that there will be at most two accidents, (i)
during 1 week, (ii) during 2 weeks (iii) in each of 2 weeks.
86. Suppose that the suicide rate in a certain state is four suicides per one
million inhabitants per month. Find the probability that in a certain town of
population 500,000 there will be at most four suicides in a month. Would you
find it surprising that during 1 year there were at least 2 months in which more
than four suicides occurred?
87. A coin is tossed 30 times. For n = 1,2, ... ,20, what is the conditional
probability that exactly 10 + n heads appear given that the first 10 tosses
resulted in heads. Show that the conditional probability that 10 + n tosses
will result in heads, given that heads appeared at least ten times, equals

C03~ n);n
20 ( 30 ) 1 .
k~O 10 + k 2k
88. How many children should a family have so that with probability 0.95
it has at least a boy and at least a girl.
89. Show that the Poisson probabilities p(kIA) satisfy the recurrence relation
A
p(kIA) = 'kP(k - l1A),

and hence determine the values of k for which the terms p(kIA) reach their
maximum (for given A).

2. Continuous Distributions
90. Verify that each of the following functions f is a probability density
function and sketch its graph.
(a) f(x) = 1 - 11 - xl for 0<x<2
1 13
(b) f(x) = -; 132 + (x - af for -00 <x < 00,

(c) f(x) = ~e-(lx-IlIl/t, for -00 < x < 00,


20'
(d) f(x) = txe- x/2 for 0< x < 00.

91. The amount of bread (in hundreds of kilos) that a bakery sells in a day
is a random variable with density
ex for 0:::;; x < 3,
f(x) = { ~(6 - x) for 3:::;; x < 6,
otherwise.
22 2. Distributions. Random Variables

(i) Find the value of c which makes f a probability density function.


(ii) What is the probability that the number of kilos of bread that will be
sold in a day is, (a) more than 300 kilos? (b) between 150 and 450 kilos?
(iii) Denote by A and B the events in (a) and (b), respectively. Are A and B
independent events?
92. Suppose that the duration in minutes of long-distance telephone con-
versations follows an exponential density function;
f(x) = te- x /5 for x> O.
Find the probability that the duration of a conversation:
(a) will exceed 5 minutes;
(b) will be between 5 and 6 minutes;
(c) will be less than 3 minutes;
(d) will be less than 6 minutes given that it was greater than 3 minutes.
93. A number is randomly chosen from the interval (0, 1). What is the
probability that:
(a) its first decimal digit will be a 1;
(b) its second decimal digit will be a 5;
(c) the first decimal digit of its square root will be a 3?
94. The height of men is normally distributed with mean J1 = 167 cm and
standard deviation (J = 3 cm.
(I) What is the percentage of the population of men that have height, (a)
greater than 167 cm, (b) greater than 170 cm, (c) between 161 cm and 173 cm?
(II) In a random sample of four men what is the probability that:
(i) all will have height greater than 170 cm;
(ii) two will have height smaller than the mean (and two bigger than the
mean)?
95. A machine produces bolts the length of which (in centimeters) obeys a
normal probability law with mean 5 and standard deviation (J = 0.2. A bolt
is called defective if its length falls outside the interval (4.8, 5.2).
(a) What is the proportion of defective bolts that this machine produces?
(b) What is the probability that among ten bolts none will be defective?
96. Consider a shop at which customers arrive at random at a rate of
twenty persons per hour. What is the probability that the time intervals
between succesive arrivals will be:
(a) shorter than 3 minutes;
(b) longer than 4 minutes.
(c) Suppose that 10% of the customers buy a certain object. Find the
distribution of the number of customers who buy an object in an hour.
97. If X is a continuous random variable with cumulative distribution
function F and density function f, show that the random variable Y = X 2 is
also continuous and express its cumulative distribution function and density
in terms of F and f.
Exercises 23

98. (Continuation). Find the density of Y = X 2 when X has:


(a) the normal distribution N(Il, (j2);
(b) the Laplace distribution (see (2.9));
(c) the Cauchy distribution (see (2.12)).
99. As in Exercises 97 and 98 with Y = IXI.
100. The lognormal distribution. If the log X is normally distributed then
X is said to have a lognormal distribution. Find its density.
CHAPTER 3

Expectation. Variance. Moments

Elements of Theory
1. The expected value or expectation or mean value or simply the mean of a
random variable X, denoted by E(X), is defined by

{ ~XiP[X = xJ for a discrete X,


E(X) =
t: xf(x) dx for a continuous X,

provided the series or the integral converge absolutely, in which case we say
that E(X) exists and write E(X) < 00.
The linearity property of the expectation operation: If E(X) < 00, E(Y) < 00,
then for any constants a and b we have

E[aX + bY] = aE(X) + bE(Y).

The mean value of a function of a random variable: Let

Y = g(X)

be a (measurable) function of X with frequency function fx. Then

LYJY(YJ,
E(Y) = E[g(X)] = { Ii
00

yfy(y) dy,
-00
Elements of Theory 25

= {~g(X;)f(X;) for X discrete,

roooo g(x)f(x) dx for X continuous, (3.1)

where fy denotes the frequency function of Y; (3.1) permits the computation


of E(Y) by means of fx without finding first fy.

2. Moments. The moment of order r about the origin of a random variable


X is defined by
J1~ == E(X'), r = 1,2, ....

The first moment J1'1 coincides with the mean E(X) == J1.
The central moment of order r is defined by
J1, == E(X - J1)' r = 2, 3, ....
The variance of X is the second (order) central moment, i.e.,
E(X - J1)2 = E(X2) - J12. (3.2)

We write V or Var for variance. The positive square root ofVar(X) is called
the standard deviation of X, usually denoted by (5.
The moments of an integer-valued positive random variable, such as the
binomial, the Poisson, etc., are more conveniently computed by means of the
so-called factorial moments.
The factorial moment Jr, of the random variable X is defined by
Jr, = E[X(X - 1) .. . (X - r + 1)] = E[(X),].
Thus
(3.3)

3. Quantiles and percentiles. For every p (0 < p < 1), the p quantile point
or the lOOp percentile point, X p say, of a random variable X (or its distribution)
with distribution function F, is defined as a solution of
F(Xp-)::; P ::; F(Xp).
For a continuous and increasing F, Xp is uniquely defined by
F(Xp) = p.
For p = 0.5 we have the median Xo.s. X O . 2S is called the first quartile and
X O . 7S the third quartile. The difference X O . 7S - X O . 2S is called interquartile
range and can be used as a measure of scatter of the distribution.

4. Probability or factorial moment generating function. This is useful mainly


for integer-valued positive random variables and is defined by
00 00

P(t) == E(t X ) = L
k~O
tkP[X = k] =
k~O
L Pk tk ;
it exists at least for It I ::; 1.
26 , 3. Expectation. Variance. Moments

The rth factorial moment ITr is given by

_ [drp(t)] _ <r)
ITr - ~ 1=1 - P(1)'

so that, by (3.3),
E(X) = P'(1), Var(X) = P"(I) + P'(I) - [P'(I)Y

The probabilities Pk = P[X = k] are given by

_ ~ [dkP(t)]
Pk - k'• dt k 1=0 .

5. Moment generating function of a random variable X defined by

(discrete X),

(continuous X).

If M(t) exists, i.e., there exists a b > 0 such that E(e 1x ) < 00 for alII t I < b, then
all the moments E(xr) (r = 1, 2, ... ) exist and can be obtained from

E(xr) = [drM(t)] = M<r)(o).


dt r 1=0

Exercises
1. Theoretical Exercises

101.* Show that the integral S~~ Ix - mlf(x) dx becomes minimum when
m is the median of the distribution with density f.
102. Let f(x) denote the density function of the random variable X. Sup-
pose that X has a symmetric distribution about a, that is, f(x + a) = f(a - x)
for every x. Show that the mean E(X) equals a, provided it exists.
103.* Cauchy-Schwarz inequality. Show that E2(XY):::;; E(X2)E(y2) pro-
vided the second moments exist.
104. Show that IE(X)I :::;; E(IXI).
105. For every set A of real numbers, we define the indicator function by

I if x E A,
{
IA(x) = 0 if x ¢ A.
Exercises 27

L
Show that
P(A) = E[IA(X)] = dFx(x).

106. If E(X) = E(X2) = 0, show that P(X = 0) = 1.


Hint: Use Chebyshev's inequality. See (8.5).
107. Show that the mean J.l of a random variable X has the property
min E(X - C)2 = E(X - J.l)2 = V(X).

108.* The mean value geometrically. Show that for a continuous random
variable X with density function f and cumulative distribution function F

J.l = E(X) = foo [1 - F(x)] dx - J:oo F(x) dx.


Consequently, in the graph below J.l = area(A) - area(B).

~~wwwwwwww~ _____________________ x
o

109. If the nth moment ofthe random variable X with distribution function
F exists, show that

E(X - C)k = k 1'" (x - C)k-l [1 - F(x)] dx

- k foo (x - C)k-l F(x) dx, 1 $; k $; n.

110. Show that the first n moments determine the first n central moments
and, conversely, that the first n central moments with the mean determine the
first n moments.
111. If X is bounded, i.e., there is a constant k < oosuchthatP[IXI $; k] =
1, then X has moments of every order.
28 3. Expectation. Variance. Moments

112. * If the n-order moment J1~ exists, then show that there exist all J11c
(k = 1,2, .... , n - 1).
113. Show that a necessary condition for the mean of a random variable
with distribution function F to exist is that
lim xF(x) = lim x[1 - F(x)] = O.
x--+-oo x-+oo

2. Mean and Variance


114. * In a lottery, n numbers are selected from the N numbers 1, 2, ... , N.
Find the variance of the sum Sn of the selected numbers.
115. An urn contains Nl white balls and N2 black balls; n balls are drawn
at random, (a) with replacement, (b) without replacement. What is the expected
number of white balls in the sample?
116. A student takes a multiple-choice test consisting of two problems. The
first one has 3 possible answers and the second one has 5. The student chooses,
at random, one answer as the right one from each of the two problems. Find:
(a) the expected number, E(X), of the right answers X of the student;
(b) the Var(X).
Generalize.
117. In a lottery that sells 3,000 tickets the first lot wins $1,000, the second
$500, and five other lots that come next win $100 each. What is the expected
gain of a man who pays 1 dollar to buy a ticket?
118. A die is thrown until the result "ace or even number" appears three
times. Find the expected number of throws:
(a) in one performance of the experiment;
(b) in ten repetitions.
119. A pays 1 dollar for each participation in the following game: three
dice are thrown; if one ace appears he gets 1 dollar, if two aces appear he gets
2 dollars and if three aces appear he gets 8 dollars; otherwise he gets nothing.
Is the game fair, i.e., is the expected gain of the player zero? If not, how much
should the player receive when three aces appear to make the game fair?
120. A player has 15 dollars. If heads appear on the first toss he receives 1
dollar and he withdraws from the game. If tails appear he bets 2 dollars and
if he wins the second toss he withdraws; otherwise he continues playing on a
bet of 4 dollars. If he loses, he bets the remaining amount of 8 dollars. What
is the expected gain of the player?
121. Of the parcels mailed abroad 10% never reach their destination. Two
books may be sent separately or in a single parcel. Each book is worth $2.
The postage for each book sent separately is 10¢ and for both books in a single
Exercises 29

parcel 15¢. For each of the two ways of mailing, find:


(a) the probability that both books reach their destination;
(b) the probability that at least one book reaches its destination;
(c) the expected net (after postage) value of the commodities reaching their
destinatination.
For each of the three criteria (a), (b), and (c), which way of mailing is preferable?
122. (a) Two indentical coins are thrown once in such a way that if one
shows heads so does the other (dependent). Let P[heads] = P and X be the
total number of heads that appear. Find the mean and the variance of the
random variable X.
(b) Two different coins with probabilities of heads PI and P2 are thrown
independently. Find the mean and the variance of the random variable X as
defined in (a).
123. Mixed distribution. Let X be a random variable with distribution
function
I - 0.8e- for x;:::: 0,
X

F (x) = {
o for x < O.
Plot the graph of F(x) and then evaluate E(X). Give an example of a random
variable X that has the above distribution.
124. If the random variable X is N(/1, 1), show that the random variable
Y = [1 - <l>(X)]lcp(X), where <l> and cp denote the distribution function and
the density of N(O, 1), respectively, has mean value 1/11.
125. The truncated Poisson distribution with the zero class missing has
probability function
),k
P(X = k) = -(e-:-;'---I-)k!' k = 1,2, ....

Find E(X) and Var(X).


CHAPTER 4

General Problems

126. Three players A, B, and C playa game as follows: At the first stage A
and B play against each other while C stays out of the game. The winner plays
against C at the second stage. Then the winner of the second game plays
against the last loser, and so on. The winner is declared the one who wins two
games in succession. Find the elements (simple events) of the sample space n
of all possible outcomes of the game. If the probability of each player winning
a game is 1/2, determine the probabilities of the corresponding possible
outcomes and show that their sum equals one.
127. A symmetrical coin is tossed until the same result appears twice in
succession. Describe the sample space in terms of the results H ("heads") and
T ("tails"). Find the probabilities of the events:
(a) the experiment terminates earlier or at the seventh trial;
(b) an even number of trials is required.
What is the expected number of trials?
128. A pair of dice is tossed six times. What is the probability that all the
faces will appear twice?
129. A deck of 52 cards is divided among four players so that each of them
gets 13 cards (game of bridge). Find the probability that at least one player
will have a complete suit (spades, hearts, diamonds, clubs).
130. Two coins CI and C2 have a probability of falling heads PI and P2'
respectively. You win a bet if in three tosses you get at least two heads in
succession. You toss the coins alternately starting with either coin. If PI > P2'
what coin would you select to start the game?
131. Ten pairs of shoes are in a closet. Four shoes are selected at random.
Find the probability that there will be at least one pair among the four shoes
selected.
4. General Problems 31

132. Let N cells be numbered 1,2, ... , N. We randomly throw balls into
them. The process is continued until a ball falls in the cell bearing the number
1. What is the probability that:
(a) n throws will be necessary?
(b) more than n throws will be necessary?
133. (Continuation). The process terminates when anyone of the cells
receives two balls.
(a) What is the probability that more than n throws will be necessary?
(b) What is the expected number of throws?
134. From a usual deck of cards we draw one card after another. What is
the probability that:
(a) the nth card will be the first ace?
(b) the first ace will appear among the first n cards?
135. The 52 cards of an ordinary deck of cards are placed successively one
after the other and from left to right. Find the probability that the thirteenth
spade will appear before the thirteenth diamond.
136. Three balls are drawn at random one after another and without
replacement from n balls numbered 1,2, ... , n. Find the probability that the
first will bear a number smaller than that of the second ball.
137. An urn contains m kinds of objects all in the same proportion. Objects
are drawn with replacement, one after another, until each kind appears at least
once. What is the probability Pv that v objects will be required?
138. A newspaper dealer gets n papers every day for sale. The number X
of papers sold is a random variable following the Poisson distribution with
parameter A.. For each paper sold he earns 1 cent, for each unsold paper he
loses b cents. Let Y denote the net gain of the dealer. Find:
(a) E(Y);
(b) How many newspapers should he get every day in order to maximize
his profit?
139. Three numbers are selected at random one after another and without
replacement from n numbers 1,2, ... , n. What is the probability that the first
number drawn will be the smallest and the second number the largest?
140. The events A, B, and C are independent with P(A) = 0.2, P(B) = 0.3,
and P( C) = 0.1. Find the probability that at least two will occur among the
three events.
141. We throw four coins simultaneously and we repeat it once more. What
is the probability that the second throw will result in the same configuration
as the first throw when the coins are, (a) distinguishable, (b) indistinguishable.
142. An urn contains w white balls and b black balls. We draw balls
successively and without replacement one after another until a white ball
appears for the first time. Let X be the required number of draws. Find:
32 4. General Problems

(a) the distribution of X;


(b) E(X).
(c) Using the result of (a) show the identity
n-w (n-w)(n-w-l) (n-w) ... 2.l n
1 +--+ + ... + - - - - - - -
n-l (n-l)(n-2) (n-l) ... (w+l)w w
where n = w + b.
143. (Continuation). Let w = b = N and draw balls successively and with-
out replacement one after another until all the 2N balls are drawn. What is
the probability that in a stage of the experiment the same number of white
and black balls has been drawn?
144. * Let B 1 , ••• , Bvo ... , be a partition of the sample space Q and P( C) > O.
Show the following theorem of total probability:

L P[BjIC]P[AIBjC].
00

P[AIC] =
j~l

145. Each of the three Misses T. C-Rena, Nike, and Galatea-wants to


accompany me on a trip. Since I cannot take all of them, I play the following
game: I say to them, "I will think of one of the numbers 1, 2, 3. Rena, you
guess first. If you guess right you will come with me; otherwise Nike will guess
next. If she guesses right she will come with me. Otherwise Galatea will
accompany me". Galatea complains that the game is unfair. Is she right?
Suppose everyone of them writes down on a piece of paper the number which
she guesses I thought of and I follow the above sequence checking Rena's
number first, etc. This is repeated until one is chosen to come with me. Galatea
and Nike protest this time. Are they justified?
146. A die is thrown v times. Find the probability that each of k (1 ~ k ~ 6)
given faces of the die appears at least once.
147. The folded normal distribution has density

I(x) = c~l_e-x2/2, x> O.


~
Determine the constant c and then evaluate the mean of the distribution.
148. Suppose the lifetime of an electric lamp of a certain type obeys a
normal law with mean /1 = 180 hours and standard deviation (J' = 20. In a
random sample of four lamps:
(a) What is the probability that all four lamps have a lifetime greater than
200 hours?
(b) The random sample of four lamps is placed in an urn and then we
randomly draw one lamp from it; what is the probability that the lamp will
have a lifetime greater than 200 hours?
Generalize the conclusion.
4. General Problems 33

149.* The game of "craps" is played as follows. The gambler throws two
dice. If at the first throw he gets 7 or 11 he wins, and if he gets 2, 3, or 12 he
loses. For each of the other sums the game is continued in two ways:
(a) the gambler continues throwing the two dice until he wins with a 7 or
he loses with the results of the outcome of the first throw;
(b) the gambler continues until he loses with 7 or wins with the result of
the first throw.
What is the probability of the gambler winning in cases (a) and (b)?
150. Two gamblers A and B agree to playas follows. They throw two dice
and if the sum S of the outcomes is < 10 B receives S dollars from A, otherwise
B pays A x dollars. Determine x so that the game is fair.
151. An urn contains n white balls and n red balls. We draw two balls at
first, and then another two, and so on, until all the balls are drawn. Find the
probability that each of the selected pairs consists of a white baH and a red ball.
152. Each of two urns A and B contains n balls numbered 1 to n. We draw
one ball from each of the urns. Find the probability that the ball drawn from
A bears a number smaller than that drawn from B.
153. The rumor mongers (see Feller, 1957, p. 55). In a town of N + 1
inhabitants, a person tells a rumor to a second person, who in turn repeats it
to a third person, and so on. At each step the recipient of the rumor is chosen
at random from the N inhabitants available.
(i) Find the probability that the rumor will be told n times without, (a)
returning to the originator, (b) being repeated to any person.
Do the same problem when at each step the rumor is told to k persons.
(ii) In a large town where the rumor mongers constitute 100p% of the
population, what is the probability that the rumor does not return to the
originator?
154. * Pascal's problem. In the game of tossing a fair coin, the first one to
obtain n successes (heads or tails) wins. Show that the game is fair (i.e., each
gambler has a probability of winning equal to 1/2). Suppose that the game
was interrupted when the first gambler had won k tosses and the second
gambler had won m tosses (0 ::s; k, m < n). Calculate for each gambler the
probability of winning if the game is continued. Hence deduce how the stake
should be divided after interrupting the game.
155.* Chebyshev's problem. Find the probability that a given fraction min
(where m and n are integers) is irreducible.
156. * Let qJ(n) denote Euler's function, i.e., the number of (positive) integers
which are primes relative to n and smaller than n. Using a probabilistic
argument, show that
qJ(n) = n fI
pin
(1 - !),
p
where the product extends over all prime divisors p of n.
34 4. General Problems

157. * Assume that the number of insect colonies in a certain area follows
the Poisson distribution with parameter A, and that the number of insects in
a colony has a logarithmic distribution with parameter p. Show that the total
number of insects in the area is a negative binomial with parameters q = 1 - P
and - A/log(1 - p).
158. Electronic tubes come in packages, each containing N tubes. Let Pk
denote the probability that a package contains k defective tubes (0 ~ k ~ m).
A sample of n tubes is taken from a package and it is observed that r ~ m
tubes are defective.
(a) What is the probability that the selected package actually contains k
(:;::: r) defective tubes?
(b) If according to customer demand a package is considered not accept-
able whenever it contains d :;::: r defective tubes, what is the probability that
the package is not accepted?
159. The probability Pn that n customers visit a supermarket in one day is
Pn = pnq, n = 0, 1, .... Two out of three customers, on average, buy a certain
type of item. The probability that an item is defective is 1/4.
(a) What is the probability that a customer buys a nondefective item?
(b) Given that k nondefective items were sold, show that the conditional
probability an that n customers visited the shop is given by

an = G)p n- k (2 - p)k+l/2n+1.

160. A manufacturer sells an item for $1. If the weight of the item is less
than Wo , it cannot be sold and it represents a complete loss. The weight Wof
an item follows the normal distribution N(Il, 1); the cost c per item is given
by c = a + f3W (a, f3 positive constants). Determine the mean Il so that the
expected profit is maximized.
161. A clerk lives at A and works at C and he starts work at 9 a.m. The
clerk always takes the train from A to B which is supposed to reach B at 8: 40
a.m. Buses from B leave for C every 15 minutes and the bus which leaves at
8: 45 a.m. is supposed to arrive at 8: 56 a.m.
The train on average experiences delays of 2 minutes and has a standard
deviation of 4 minutes. The bus always leaves on time, but arrives on average
after a 2-minute delay and a standard deviation of 3 minutes. What is the
probability that the cleark arrives late? The clerk's employer drives to his
office; he leaves at 8: 45 a.m. and the driving time to the office has a mean
value of 12 minutes and a standard deviation of2 minutes. Find the probability
that:
(a) both, clerk and employer, arrive late;
(b) the employer arrives earlier than the clerk.
Assume that the distributions involved are normal.
162.* Quality control. In a continuous manufacturing process the percen-
tage of defective articles is p. To maintain the quality of the product at a certain
4. General Problems 35

standard, the articles are examined, one by one, until a sequence of fixed length
r appears with no defective article. Then total inspection terminates and only
a certain fraction f of the product is chosen at random for inspection until a
defective article appears when the above process of 100% inspection continues.
Under the described sampling scheme find:
(a) the probability of a defective sequence, i.e., that a defective article is not
followed by r successive good articles;
(b) the expected number of articles in a defective sequence. Hence deduce
the expected number, (i) of defective sequences, (ii) of inspected articles, after
a defective article;
(c) the expected proportion of the product subject to inspection;
(d) the expected percentage p of defective articles going out for sale provided
that any discovered defective article is replaced by a good one;
(e) for given values of f and r, what value p*, say, of p maximizes p.
163.* The dilemma of the convict. Three convicts A, B, and C appeal for
parole and the board decides to free two of them. The convicts are informed
of this decision but are not told who the two to be set free are. The guard,
who is A's friend, knows which convicts are going to be set free. A sees that
it would not be right to ask the guard about his own fate, but thinks that he
might ask for the name of one of the other two who will be set free. He supposes
that before he asks the probability of his being set free is 2/3, and when the
guard answers that B is pardoned, for example, the probability that he will
be set free is lessened to 1/2, because either A and B will be set free or Band
C. Having thought about this, A is afraid to ask the guard. Is his fear justified?
164.* Collection of coupons. In each box of a given product there is a
coupon with a number from 1 to 6. If a housewife succeeds in getting the series
1-6 she receives a free box of the product. How many boxes must she buy, on
average, before she gets a free one?
165.* In a row of 19 seats in an amphitheatre 10 male students and 9 female
students sit at random. Find the expected number of successive pairs of seats
in which a male student and a female student sit.
166.* A, B, and C are about to play to following fatal game. Each one has
a gun and will shoot at his target-A, B, and C taking turns at each other in
that order-and it is assumed that anyone who is hit will not be shot at again.
The shots, one after another, continue until one of the players remains unhit.
What strategy should A follow?
167. Samuel Pepys (who was about to place a bet) asked Newton which
of the following events A, B, or C is more probable: A -at least one six when
6 dice are thrown; B-at least two sixes when 12 dice are thrown; and C-at
least three sixes when 18 dice are thrown. Which answer (the correct one) did
Newton give?
168.* Birthday holidays. The Worker's Legal Code in Erehwon specifies as
a holiday any day during which at least one worker in a certain factory has a
36 4. General Problems

birthday. All other days are working days. How many workers must the
factory employ so that the number of working man-days is maximized during
the year?
169. Bertrand's paradox. A chord AB is randomly chosen in a circle of
radius r. What is the probability that the length of AB is less than r?
170.* The neophyte at the horseraces. At a horserace a neophyte better,
who is about to bet a certain amount, wants to choose the best horse. It is
assumed that no two horses are the same. The better looks at the horses as
they pass by, one after another, and he can choose as the winner anyone of
the horses, but he cannot bet on a horse he has already let pass. Moreover,
he can tell whether any horse passing by is better or worse compared with the
preceding ones. Suppose n horses take part in the race. How should he proceed
to choose the horse to bet on in order to maximize his probability of winning?
What proportion of horses should he wait to pass by before he makes his bet
when n is large? (It is assumed that horses parade in front of the neophyte in
a random order.)
PART II

ADV ANC ED TOPICS


CHAPTER 5

Multivariate Distributions

Elements of Theory
1. Multidimensional or vector random variable: A real-valued vector func-
tion defined on the sample space Q (more strictly, measurable with respect to
the (J' algebra on Q).

The following definitions concerning bivariate distributions extend easily


to more than two variables.

2. Thejoint distribution function of the random variables X and Y is defined


by
F(x, y) == P[X ::; x, Y::; y] for every point (x, y) E R2.

3. The distribution of the random pair (X, Y) is called discrete if there exists
a denumerable set of points (Xi' y) such that
P[X = Xi' Y = Yj] = Pij > 0,
and

The probabilities Pij (i = 1, 2, ... , j = 1, 2, ... ) define the so-called joint prob-
ability distribution or frequency function of X and Y. Clearly we have
F(x, y) = L
XjS;:X
Pij'
Yj";Y

4. The distribution of the pair (X, Y) is called continuous if there exists a


function f(x, y) such that for every (x, y) the joint distribution function F(x, y)
40 5. Multivariate Distributions

of X and Y can be written as

F(x, y) = f:oo foo f(u, v) du dv.


The function f(x, y) is called the joint density function of X and Y. In this case,
for every continuity point (x, y) of f (almost all points are such), we have
o2F(x, y) . P[x < X ::s; x + Ax, y < Y::s; y + Ay]
f(x, y) = = hm .
ox oy ~X~O Ax Ay
~y~O

5. Marginal distributions. The distribution of X, as obtained from the joint


distribution of X and Y, is referred to as the marginal distribution of X.
Similarly for Y. Thus for jointly discrete variables the marginal probability
distribution of X is given by
Pi = P[X = xJ = L Pij'
j
(5.1 )

t:
while for continuous variables the marginal density fx of X is given by

fx(x) = f(x, y) dy. (5.2)

6. Conditional distributions. The conditional distribution function of X,


given Y E B with P[Y E B] > 0, is defined by
P[X::S; x, Y E BJ
Fx(xl Y E B) = P[X ::s; xl Y E B] = P[Y E B ] · (5.3)

The conditional probability distribution of X, given Y = yj' is defined by


P[X = x· Y = y.J p ..
fX(Xil Y = y) = "J =---'1, (5.4)
P[Y=yJ qj
where we set qj = P[Y = Yj] (j = 1,2, ... ).
The conditional density of X, given Y = y, for the continuous pair (X, Y) is
defined by
f(x, y)
f x(xl Y = y) = -r-- = 1·1m f x (xly < Y ::s; y + LlY)
A

JY(Y) ~y~O
(5.5)
= lim P[x < X ::s; x + Ax, y < Y::s; y + Ay].
~x~o Ax·P[y < Y::s; y + AyJ
~y~O

Hence, and by virtue of (5.1) and (5.2), we deduce the following.

7. Formula of total probability for random variables:

Pi = P[X = xJ = L P[X = x;l Y = yj]P[Y = Yj] = L fx(x;l Y= y)qj'


f:
j j

fx(x) = fx(xl Y = y)fy(y) dy.


Elements of Theory 41

Similarly, we obtain the

8. Generalized Bayes formula:

(5.6)
fx(xl Y = y) = 00 fx(x)fy(Ylx = x)
Loo fy(Ylx = x)fx(x) dx
9. The expected or mean value of a function 9 of two random variables is
defined by

L:i L g(Xi' Yj)Pij (for discrete),


{
f: f:
j
E{g(X, Y)} =
g(x, y)f(x, y) dx dy (for continuous).

We say it exists if the (double) series or integral is absolutely convergent.

10. The mixed central moment of orders r + s of X and Y is defined by


Jlx, = E[(X - JlxY(Y - Jl y )'],
where Jlx = E(X), Jly = E(Y), and e.g., for continuous (X, Y), E(X), is given by

E(X) = L: f: xf(x, y) dx dy = L: x (L+oooo f(x, y) dY) dx


= f+oo
-00 xfAx) dx.

11. The mixed central moment of the second order


JlII = E(X - Jlx)(Y - Jl y ) == Cov(X, Y)

is called the covariance of X and Y.


The number p (-1 ~ p ~ 1), defined by
Cov(X, Y)
p (X, Y ) = ,
(lx(ly

where (I; = Var(X), (I; = Var(Y), is called the correlation coefficient between
X and Y.
The following properties can easily be established. For all constants a, b,
c, and d we have:
(i) Cov(X + a, Y + b) = Cov(X, Y) (in variance under translations);
(ii) Cov(cX, dY) = cd Cov(X, Y);
42 5. Multivariate Distributions

(iii) p(aX + B, c Y + d) = sg(ac)p(X, Y) where

°
1 for x> 0,
sg(x) = { _ 1 for x <
(iv) Var(X ± Y) = Var(X) + Var(Y) ± 2 Cov(X, Y).

12. The regression (function) of Y on X, m2(x) say, is defined as the

t:
conditional expected value of Y given X = x, that is,

m 2(x) = E(YIX = x) = f 00

-00
yfy(ylX = x) dy =
yf(x, y) dy
f)
x(x
The curve y = m2 (x) is called the mean regression curve of Yon X. Similarly,
we can define the regression of X on Y.

13. The dispersion or variance-covariance matrix of a random vector X =


(X I ' ... , X.), usually denoted by L, is defined by
where (Jij = COV(Xi' X), i,j = 1, ... , n. (5.7)
We shall write D(X) = L. If a = (ai, ... , a.), is a vector of constants then
• •
Var(alX I + ... + a.X.) = Var(a'X) = a'D(X)a = a'La = L L aiaj(Jij'
i=1 j=1

If the rank r of L is less than n, then the distribution of X is called singular.


This is equivalent to the distribution being concentrated in an r-dimensional
subspace of the n-dimensional Euclidean space E·. For example, if n = 2 and
r = 1 the distribution is concentrated on a straight line in the (X I' X 2) plane
and Ip(XI' X 2 )1 = 1.

14. The random variables XI'"'' X. are called completely stochastically


independent or simply independent if one of the following holds:
(I) For all Borel sets of the real line A I ' ... , A.
P[XI E AI' ... , X. EA.] = P[XI E AI] ... P[X. EA.].
(II) The joint distribution function F of X I, ... , X. can be written as
for every (XI' ... , x n ),
where Fi denotes the marginal distribution function of Xi'
(III) The joint frequency (density) function f can be written as
f(x l , ... , x n ) = fl(X1) .. ·!.(xn ),
where /; denotes the marginal frequency function of Xi'

15. The main multivariate distributions


(a) Discrete distributions
Exercises 43

(i) Double hypergeometric: R, W, B integers N = R +B+W Then

Pij = (~)(~)C -~ -J/(~)' Osi+jsn,

(a generalization of the simple hypergeometric, see Exercise 175).


(ii) Multinomial (k dimensional)
n!
P[X 1 -- n 1 , •.• , X k -- nk ] -
- , , ,PI"I ". ".+1
",Pk Pk+1' (5.8)
n1····nk • nk + 1·
where it was set

(iii) Negative multinomial (k dimensional)

P[X 1 = n1 , ••• , Xk = nk ] = ( 1 +
k+1
,L ()j
)-5-V resr()+ v) n I()?' k+1
(5.9)
nj .
°
J=l S j=1

where s > 0, ()j > (nj = 0, 1,2, ... ), and nk+1 = n - (n1 + ... + nd.
(b) Continuous distributions
(i) Uniform in a bounded set S of E" with density
f(x) = c, XE S,
where c- 1 = measure (length, area, volume, etc.) of S.
(ii) n-dimensional (nonsingular) normal N(I1, L) with density
f(x) = [(2n)PI!:lr 1/2 exp[ -!(x - 11),!:-l(X - 11)], (5.10)

where 11 = (111' ... , I1n)' = E(X) denotes the mean vector and L denotes the
(positive definite) covariance matrix of the normal random vector X with
density (5.10).
(iii) Dirichlet distribution with density
k+1
L
i=l
Xi = 1, Xi ~ 0. (5.11)

Exercises
171. The joint distribution of (X, Y) is defined by P[X = 0, Y = 0] =
P[X = 0, Y = 1] = P[X = 1, Y = 1] = 1/3.
(a) Find the marginal distribution functions of X, Y.
(b) Examine whether the points PI: (-1/2,0), P2 : (0, 1) are continuity points
of F(x, y).
172. Show that the function

F(x, y) = {01 for X +Y< 1,


for x +y~ 1,
is not a joint distribution function.
44 5. Multivariate Distributions

173. We consider a family with two children. Let Xn = 1 if the nth child is
a boy for n = 1, 2 and X 3 = 1 if there is only one boy, otherwise Xi =
(i = 1, 2, 3). Show that the Xi are pairwise independent but not completely
°
independent.
174. X and Y have the joint density

for °< x < y < 00.

Find (a) the constant c, (b) the marginal distributions of X and Y.


175. The k-dimensional hypergeometric distribution has probability func-
tion

where

Pk+l = 1- (Pl + P2 + ... + Pk), n i > 0, Pi> 0.


Show that the conditional distribution of X k given Xl' ... , X k - l is hyper-
geometric.
176. If Xl' X 2' ... , X k are distributed according to the multinomial distri-
bution, the conditional distribution of Xl' given X 2 = n 2 , ••• , X k = nk , is
binomial with parameters n - (n2 + ... + nd and pd(Pl + Pk+l).
177. If Xl' X 2 , ••• , X k have a Dirichlet distribution, show that the condi-
tional distribution of XdSk' for Xi = Xi' where Sk = 1 - (Xl + .... + Xk-d
is {3(n k, nk+d.
178. Let X = (Xl' X 2, X 3) be uniformly distributed in the subset of the
positive orthant defined by Xl ~ 0, X2 ~ 0, X3 ~ 0, Xl + X2 + X3 ::;; c. Find,
(a) the density of X, (b) the marginal distribution of (Xl , X 2 ).
179. A bivariate normal distribution has density
f(x, y) = c exp[ _x 2 + xy _ y2].
(a) Find the constant c and the moments of order 2.
(b) Find the curves of constant density f(x, y) = c*.
180. If (X, Y) is uniform in the triangle X ~ 0, y ~ 0, X + y ::;; 2, find, (a)
the density of (X, Y), (b) the density of X, (c) the conditional density of Y for
X = x, (d) E(YIX = x).
181. A die is thrown 12 times.
(a) Find the probability that every face appears twice.
(b) Let X be the number of appearances of 6 and Y the number of appear-
ances of 1. Find the joint distribution of X, Y.
(c) Find Cov(X, Y).
Exercises 45

182.* Show that the most probable value (n?, ng, ... , nr+l) of the multi-
nomial distribution satisfies the relations
npi - 1 < n? s (n + k)Pi' i = 1, 2, ... , k + 1.
Hint: Show that
for i -# j, i,j = 1,2, ... , k + 1.
183. * Multiple Poisson distribution. If the number oftrials n is large and the
Pi small so that the npi = Ai are moderate, show that the multinomial distribu-
tion can be approximated by the so-called multiple Poisson distribution

184. Given that the density of X and Y is


2
f(x, y) = (1 + x + y)3' x> 0, y > 0,

find, (a) F(x, y), (b) fx(x), (c) fy(YIX = x).

185. Find the density function f(x, y) of the uniform distribution in the
circle x 2 + y2 S 1. Find the marginal distributions of X and 1'. Are the
variables X and Y independent?

186. The joint density of the variables X, Y, Z is


f(x, y, z) = 8xyz, 0< x, y, z < 1.
Find P[X < Y < Z].

187. For each of the following densities f(x, y), find F(x, y), Fx(x), Fy(y),
fx(x), fy(y), fx(xl Y = y), fY(ylX = x).
(a) f(x,y)=4xy, O<x, y<l,
(b) f(x, y) = i(x 2 - y2)e- X , 0< x < 00, Iyl < x.
188. The joint probability function P(Xi' y) of X, Y is given in the following
table:

~-1
-1
0.1
0
0.15 0
2
0.1
p(x)
0.35
0 0.15 0 0.1 0.2 0.45
0.05 0.05 0 0.1 0.20
q(y) 0.3 0.2 0.1 0.4 1.00

(a) Calculate the regression lines of X on Y and of Yon X.


46 5. Multivariate Distributions

(b) Hit is possible to observe only X, what is the best estimate of Yin terms
of X in the sense of the mean square error?
189.* Show that the discrete variables X and Y with joint probability
function Pij = P[X = Xi' Y = yJ are independent if and only if the matrix of
probabilities P = (Pij) has rank 1.
190. Show that two orthogonal (uncorrelated) variables each taking two
values are independent.
CHAPTER 6

Generating Functions.
Characteristic Functions

Elements of Theory
1. The probability or factorial moment generating function P(t) of a non-
negative integer-valued random variable X is defined by

I I
00

P(t) = E(t X ) = P[X = k]t k = Pk tk .


k=O
It exists at least for -1 ~ t ~ 1.
If the factorial moment n" say, of order r exists,
nr = E[X(X - 1) ... (X - r + 1)] = E[(X)r],
then this is given by
nr = p(r) (1 ), r = 1,2, ... , (6.1)
where p(r)(t) denotes the derivative of order r of P(t). Thus we obtain
n 1 = E(X) = P'(I),
and hence,
Var(X) = P"(l) + P'(1) - [P'(I)Y (6.2)

2. Distribution of the sum of a random number of random variables. Let


(6.3)

where the random variable N = 0, 1, ... and the Xi are completely independent.
If the Xi have the same distribution with probability generating function Px(t),
then the probability generating function of SN
(6.4)
48 6. Generating Functions. Characteristic Functions

Moreover, if E(X) and E(N) exist, then


E(SN) = E(X)E(N). (6.5)

3. Compound Poisson distribution. This is the distribution of SN when N


has the Poisson distribution. Therefore its probability generating function is
(6.6)
where A. = E(N). If the Xi are Bernoulli random variables, then S~ is Poisson
with parameter A.p, where p = E(X;).

4. The moment generating function M(t) of the random variable X is


defined by
M(t) = E(e tX )
provided it exists for - 0 < t < 0 and some 0 > O.

Proposition. If Mx(t) exists, then it has derivatives of every order for It I < 0
(0 > 0) and the moments of every order exist. In fact, we have for every r =
1,2, ...
E(xr) = M(r)(o),
where M(r)(t) denotes the rth derivative of M. Furthermore, it determines
uniquely (characterizes) the distribution of X.

5. Complex random variable: Z = X + iY where the X and Y have a joint


probability function. Its expected value is defined by
E(Z) = E(X) + iE( Y), i=J=1.

6. The characteristic function of a random variable X is defined by


cp(t) = E(e i/ X ) = E(cos tX) + iE(sin tX).
It exists for every value of the real parameter t since lei/xI = 1.

Properties of a characteristic function cp(t)


(i) cp(O) = 1, Icp(t)1 ::; 1.
(ii) cp( - t) = cp(t) where a denotes the conjugate of a complex number a.
(iii) cp(t) is uniformly continuous everywhere.
(iv) If the rth moment J1.~ = E(xr) exists, then cp(t) has a derivative of order
r, cp(r)(t), and

thus J1.'1 = E(X) = i-1cp'(0), J1.2 = -cp"(O).


(v) If cp(t) has a derivative of order r at t = 0, then all moments up to order
r exist if r is even and up to order r - 1 if r is odd.
Elements of Theory 49

(vi) If E(xr) exists then qJ(t) has the Mac Laurin expansion
r , (it)k k
qJ(t) = kf:O J1.k k! + O(t ), (6.7)

where O(x) denotes a quantity such that O(x)/x -+ 0 as x -+ O.


Moreover, the so-called cumulant generating function or second charac-
teristic function
I/!(t) = log qJ(t)
has the expansion
Ct)i
I/!(t) = Lr
Kj - ; - + O(tr), (6.8)
j=O ).

where K j is called the cumulant of order j of X.


Note: For a proof of (iv)-(vi) we refer, e.g., to Cramer (1946).

7. The characteristic function of a random variable X determines uniquely


(characterizes) the distribution of X. In fact, we have the following inversion
formula of characteristic functions (Fourier transforms).
If x + h and x - h (h > 0) are continuity points of the distribution function
F of X then

F(x + h) - F(x - h) = lim -1 fC sinh t .


_ _ e-·xtqJ(t) dt.
c-oo n -c t
For a discrete random variable we have

(6.9)

In the special case when X is integer valued we have

1
Pk = P[X = k] = -2
n
flt -It
e -ikt qJ(t) dt.

If the characteristic function qJ(t) is absolutely integrable in ( - 00,(0), then


the corresponding distribution is continuous with density f(x), given by the
inversion formula

1
f(x) = -2 foo .
e-·xtqJ(t) dt. (6.10)
n -00

8. Common property of the probability generating function, the moment


generating function, and the characteristic function. This is very useful for the
study of the distribution of sums of independent random variables. Let G(t)
denote any of these functions. Then, if Sn = Xl + X 2 + ... + Xn where the Xi
50 6. Generating Functions. Characteristic Functions

are independent we have


n Gx.(t).
n
GsJt) = (6.11)
i=l

9. If P(t), M(t), and ((J(t) exist for a random variable X, then the following
relations hold:
Mx(t) = Px(e ' ) = ({Jx( - it),
((Jx(t) = Mx(it) = Px(e it ).

10. In addition to characterizing a distribution, a characteristic function


facilitates the study of the asymptotic behavior of a sequence of random
variables as indicated by the following.
Continuity theorem for characteristic functions (Levy-Cramer). Let {Xn}
bea sequence of random variables and let {Fn(x)}, {({In(t)} be the corresponding
sequences of distribution functions and characteristic functions. Then a
necessary and sufficient condition that the sequence Fn(x) of distribution
functions converges to a distribution function F(x) for every continuity point
x of F (weak convergence) is that ({In(t) --+ ({J(t), as n --+ 00 for every t and ({J(t)
is continuous at t = O. Whenever this holds the limiting characteristic function
({J(t) is the characteristic function of the limiting distribution function F.

11. Infinitely divisible distributions. The random variable X or its distribu-


tion is called infinitely divisible if for every integer n, X can be represented as
the sum of n independent and identically distributed random variables, i.e.,
X = Xl + ... + X n.
Hence the characteristic function ({J(t) of X must satisfy
for every n,
where ((In(t) is a characteristic function.

12. Generating functions for multivariate distributions. The definitions of


probability generating functions, moment generating functions, and char-
acteristic function(s) of univariate distributions extend easily to multivariate
distributions of (Xl' ... , Xn) = X' by replacing t by the row vector t' =
(tl' ... , t n) and X by X, so that tX is replaced by t'X = t 1 X 1 + ... + tnXn.
Thus, for example, the probability generating function of X and Y with
joint probability function
Pjk = P[X = j, Y = k], j, k = 0,1, ... ,
is defined by
P(tl' t 2 ) = E(tftD = L
j,k
Pjkt{t~,

and the characteristic function of X = (Xl' ... , Xn)' by


({Jx(t) = ({Jx(t 1 , ... , t n) = E(e it •X ) = Eei(t,X,+"·+lnX n).
Exercises 51

It can easily be shown that:


(i) Px(td = P(tl' 1), Py(t2) = P(1, t 2).
(ii) Px+y(t) = P(t, t}.
(iii) X and Yare independent if and only if

P(tl' t 2} = P(tl' 1)P(1, t 2) = PX(tl)Py(t 2) for every (tl' t 2),


<P(tl' t 2) = <P(tl, 0)<p(0,t 2} = <PX(tI)<Py(t 2} for every (t I ' t 2 )·

(iv) The mixed moment P-}k = E(Xiyk} is given by

I _ ai+k<p(O, O} .-U+k)
P-ik - at{ at~ I .

By analogy to the inversion formula for univariate distributions, we have the


following:
If the vertices of the generalized rectangle (or n-cell) Xi - hi ~ Xi ~ Xi + hi
(i = 1, ... , n) (hi> 0) are continuity points of the distribution function
F(Xl' ... , x n }, then

P[Xi - hi < Xi ~ Xi + hi' i = 1, ... , n]


1 fC fC sin h·t·
= lim n
c-oo 1t -c
...
-c
<P(tl,···, t n } Il--'-'
n

i=l ti
exp( -iXit i} dt i·

In the special case of an absolutely integrable <p(t I ' ... , t n }, the distribution is
continuous with density

The continuity theorem also holds, i.e.,

when <p is continuous at (0, ... , O).

Exercises
191. Find the probability generating function and hence the mean and
variance ofthe following distributions: (a) binomial, (b) Poisson, (c) geometric,
(d) Pascal. Moreover, deduce the corresponding moment generating func-
tions and characteristic functions.
192.* Let Xl = X 2 = X when X has the Cauchy density
1 1
f(x} =;. 1 + X2' -00 < X < 00.
52 6. Generating Functions. Characteristic Functions

Prove that
<PX 1 +X 2 (t) = <Px 1 (t)<PX 2 (t),
while X I' X 2 are not independent.
193. From the probability generating function Px(t) of X = 0, 1,2, ... , find
the probability generating function of Y = 3X + 2.
194. Using the probability generating function prove that the random vari-
able SN of(6.3) has variance given by Var(SN) = E(N) Var(X) + Var(N)E2(X).
195. A woman continues to have children until she has a boy! Suppose
that the probability of having a blond child is P; find the probability that the
woman will have k blond children. (Assume that the color is independent of
sex.)
196. The number N of visitors to a shop has the Poisson distribution. On
average, 150 women visit the shop every day. Of these, 100PI% buy mini,
100p2% buy midi, 100p3% buy maxi, and the remaining buy nothing. (No
woman buys more than one dress!!) Let Xl' X 2, X3 be the corresponding
numbers of mini, midi, and maxi that are bought in a day. Find the joint
probability function of Xl' X 2, X 3. Are they independent?
197. * Equal characteristic functions in a finite interval do not necessarily
define the distribution uniquely. Let X, Y be independent with densities

( ) = a + b - a cos(ula) - b cos(ulb)
f xU 2 2 nu '

-oo<u<+oo.
r ) _ c - c cos(ulc)
lY(U - 2'
nu
Prove that <Px(t) = <py(t) for It I :<;:;; min{lla, lib, lie}, 2c = a + b.
198.* Let Xl' X 2, ... , Xn be independent random variables having the same
normal distribution with mean Jl and variance 1. We set

Moreover, let the random variable T = aYv where a, v are suitable constants,
and Yv follows the X2 distribution with v degrees of freedom. Using charac-
teristic functions, find the constants a and v so that the random variables
Sn and T have the same means and the same variances.
199. Prove that SN of (6.3) has the characteristic function
<PsN(t) = PN(<Px.(t)).
200. Prove that <p(t) = exp[2(e- I'1 - 1)] is the characteristic function of a
Poisson distribution compounded with (generalized by) a Cauchy distribution.
Then, using the inversion formula, find its density.
Exercises 53

201. * Let Z. be a random variable following the binomial distribution with


parameters nand p = ),/n p, > 0). Moreover, let the random variable Z have
a Poisson distribution with parameter A. Prove that lim n _", cpzJt) = cpz(t)
(t E R), where cpzJt) and cpz(t) are the characteristic functions of Z. and Z,
respectively. Then deduce the approximation of the binomial by the Poisson.

202. Calculate the characteristic function of a Gamma distribution with


density

f(x) = /(:) e-Axx s - 1 , x> 0 (s > 0).

Then deduce the characteristic function of x;.


203. At the nth toss of a coin a player receives or pays 2-' of a drachma
depending on whether heads or tails appears. Let y" be the gain of the player
after n tosses. Show that the distribution of y" as n -4 00 tends to the uniform
distribution in the interval (-1, 1). First find the characteristic function of Yn'

204. X has distribution function F(x) which is a mixture of two distribution


functions Fl , F2 as follows:
F(x) = ),Fl (x) + (l - ),)F2(x), A> O.
If Fl is N (111' aD and F2 is N (112' aD find the characteristic function of F and
then the E(X) and Var(X).
205. Find the second characteristic function (cumulant generating function)
t/!(t) of the exponential with density
x> 0,
and prove that the cumulant of order r Kr is given by
(r - 1)!
Kr = r = 1,2, ....
(]'

206. (Continuation). Let Y be a discrete random variable defined as follows.


For h > 0 and k = 0, 1, 2, ... ,
P[Y = (2k + 1)h/2] = P[kh :$; X:$; (k + 1)h].
Find the probability generating function of Y and then show that E(X) > E( Y),
Var(X) < Var(Y).
207.* Suppose that the density f(x, y) = g(X2 + y2), i.e., it has circular
symmetry about the origin. Show that the characteristic function of f, cp(t, u),
is a function of t 2 + u 2 (only). Hence show that the only distribution of inde-
pendent random variables with the above property is the normal. (Consider
an orthogonal transformation of t, u.)
208. Consider two independent random samples Xl' X 2 , ... , X. and
54 6. Generating Functions. Characteristic Functions

Yl , Y2 , ••• , Yn from the Laplace distribution

-00 < x< 00.

Show that the means of the differences Xi - 1; and Xi + 1; have the same
distribution but are not independent. Are they orthogonal?
209. If cp(t) is a characteristic function show that, (a) Icp(tW, (b) e).(q>(t)-l) are
also characteristic functions.
210.* Calculate the probability generating function of the multinomial
distribution and then show that

211. (Continuation). If, as n -> 00, npj -> Aj (j = 1,2, ... , k + 1), then the
multinomial distribution tends to the k-dimensional Poisson Y = (Yl , Y2 , •.. ,
~) where the 1; are independent Poisson, 1j with parameter Aj (cf. Exercise
183).
212.* Calculate the characteristic function of the negative multinomial
distribution (5.9) and hence
(a) Verify that E(Xj) = s(}j, Var(X) = sej(1 + e), Cov(X j , X k ) = seA.
(b) Show that if the conditional distributions of the Xj given V = v are
independent Poisson with parameters vAj , respectively (j = 1,2, ... , r) and V
has the Gamma distribution with density
AS
/(v) = r(s) e-}.V-l, S > 0,

then X = (Xl' X 2 , ••• , X k ) has the above negative multinomial distribution


with parameters ej = AjlA.
213. X, Y have a distribution of the continuous type with characteristic
function cp(t l ' t2). Show that the conditional characteristic function of X given
Y= y is

f_OOoo e-it2Ycp(tl' t 2) dt2


CPx(tll Y = y) = foo
-00 e- it2Y cp(0, t2) dt2

214. A random sample Xl' X 2, ... , X n,+n2- n on a continuous random


variable X with - 00 < X < 00, is divided at random into three subsets of
(nl - n), (n2 - n), and n observations (nl > n, n 2 > n). Let Sl' S2' and S3
denote the corresponding sums. Find the characteristic function of
and E(YIZ = z).
215. Show that the following distributions are infinitely divisible: (a) Pascal,
(b) Cauchy, (c) Laplace, (d) Gamma.
CHAPTER 7

Distribution of Functions of
Random Variables

Elements of Theory
In finding the distribution of a function of a scalar or vector random variable
the following formulas turn out to be very useful.

1. Distribution of monotone functions. Let X be a continuous random


variable with distribution function Fx and y = g(x) a differentiable and mono-
tone (increasing or decreasing) function with inverse function x = g*(y) (i.e.,
°
g'(x) is everywhere either> or < 0). Then the random variable Y = g(X) is
also continuous with distribution function Fy given by

F ( ) = {Fx(g*(y», g'(x) > 0,


y y 1 - Fx(g*(y», g'(x) < 0,
and density function

dg* I 1
fy(y) = fx(g*(y» I""dY = fx(g*(y» Ig'(x)I' (7.1 )

2. Under the preceding assumptions, except that the equation


y = g(x)

has roots Xl' ... , xn, •.. , i.e., for given y we have

and the derivatives


g'(x;) #- 0, i = 1,2, ... ,
56 7. Distribution of Functions of Random Variables

Y = g(X) is also continuous with density

fx(x 1) fx(x n)
fy(y) = Ig'(xdl + ... + Ig'(xn}l + .... (7.2)

3. Functions of two random variables. An interesting case is when we have


a function of two continuous random variables X and Y with joint density
f(x, y).
(a) Sum of two random variables. The distribution function Fz(z) of the
sum
Z=X+Y

t: r:Y
is given by

f L+Y~z f(x, y) dx dy =
r:
Fz(z) = dy f(x, y) dx

= f:oo dx xf(x, y) dy = toooo dx roo f(x, y - x) dy

= t: dy roo f(x - y, y) dx.

t:
Differentiating with respect to Z we have the density of Z

fz(z) = f(z - y, y) dy = f: f(x, z - x) dx.

In the special case of independence of X and Y, the density fz(z) of the sum is
called the convolution of the density functions fx and fy, and we have

fz(z) = t"'oo fx(z - y)fy(y) dy = f: fx(x)fy(z - x) dx. (7.3)

r
The distribution function Fz is given by

Fz(z) = f: Fx(z - y)fy(y) dy =


e
", Fy(z - x)fx(x) dx. (7.4)

Note. If X and Yare discrete, the integrals are replaced by sums.


(b) Distribution of the quotient of two continuous random variables. The
distribution function Fz of the ratio
X
Z=-
Y
is given by

Fz(z) = 1° 00
dy
fYZ
-00 f(x, y) dx +
fO
-ro dy
r'" f(x, y) dx,
JZY (7.5)
Elements of Theory 57

and the density by

fz(z) = L'Xl yf(yz, y) dy - f:oo yf(yz, y) dy. (7.6)

In the special case of independence of X and Y, (7.5) and (7.6) become

Fz(z) = too Fx(zy)fy(y) dy + f:oo [1 - Fx(zy)Jfy(y) dy,


(7.7)
fz(z) = f:oo IYlfx(zy)fy(y) dy.

4. Functions of several random variables. This is the general case in which,


given the joint density function f(x 1, ... , xn) of the random variables Xl' ... ,
X n • we want the distribution of
j = 1, ... ,m. (7.8)
Then the joint distribution function G(Yl' ... ' Ym) of Yl , ... , Ym can be
obtained from

G(Yl' ... , Ym) = fi>· f f(x l , ... , xn) dx l , ... , dxn,

where the region of integration D is defined by the relation


D = {(Xl' ... , Xn): gj(X l , ... , Xn)::;; yj,j = 1, ... , m}.
For discrete random variables the distribution is obtained by replacing the
multiple integral by a multiple sum over D. Of special interest is the case m = n
for which the problem, under certain conditions, has a specific solution.
If the functions gj of (7.8) (j = 1, ... , n) have continuous partial derivatives
of the first order so that the Jacobian J of the transformation (7.8) is #0, i.e.,
Ogl Ogl
oX l oXn
#0, (7.9)
ogn ogn
oX l oXn
then there exists an inverse transformation
i = 1, ... , n, (7.10)
and the random variables Yl , ... , y" are continuous with joint density (cf.
(7.1 ))
f*(Yl' ... , Yn) = f(gf(Yl' ... , Yn), ... , g:(Yl, ... , Yn))IJ(Yl' ... , Yn)l, (7.11)
where J(Yl' ... , Yn) denotes the Jacobian of the inverse transformation (7.10),
58 7. Distribution of Functions of Random Variables

I.e.,

For example, in the case of linear transformations


n

Yi = I
j=l
aijXj' i = 1, ... , n,

the Jacobian equals the determinant of A = (a i),

J(XI' ... , xn) = IAI·


When A is nonsingular, i.e., IA I i= 0, the density of the random vector
Y=AX,
where Y = (Yl> ... , Y"),, X = (Xl' ... , Xn)' is given by
f*(YI"'" Yn) = f(A-Iy)IAI- I. (7.12)

Exercises
216. The radius of a circle is approximately measured so that it has the
uniform distribution in the interval (a, b). Find the distribution:
(a) of the length of the circumference of the circle;
(b) of the area of the circle.
217. Suppose that X and Yare independent random variables with the
same exponential density
X> 0.
Show that the sum X + Y and the ratio X/Yare independent.
218. Suppose that X and Yare independent uniform variables in (0, 1).
Find the probability that the roots of ). 2 + 2X A + Y = are real. °
219. The angle cp at which a projectile is fired with initial velocity v follows
the uniform distribution in the interval (0, n/2). Find the distribution of the
horizontal distance d between the firing point and the point at which the
projectile falls.
220. * Suppose that X and Yare normal variables with mean 0, variance
(f2, and correlation coefficient p.
(a) Find the distribution of X/Y.
(b) Using the above result prove that
1 1 arc sin p
P[X<0,Y>O]=2 P [XY<O]=4- 2n .

Verify this by integrating the density of (X, Y).


Exercises 59

221. Let A = (X, Y) be the point of impact of a shot on a vertical target

220 with p = °
with center at origin 0 and suppose the distribution of A is that of Exercise
(called circular normal). Show that the distance R = (OA)
between the point of impact and the centre of the target and the angle of OA
with the horizontal axis (i.e., the polar coordinates of A) are independent
variables. Find the distribution of R2.
222. If X has the uniform distribution on the interval (0, 1), then:
(a) Prove that Y = aX + b is also uniformly distributed (a and b constants).
(b) Find the distribution of Y = AX 2 + BX + C (A, B, C constants).
223. If X I, X 2 are independent and uniform on the interval (0, 1), find the
densities of: (a) Xl + X 2 , (b) Xl - X 2 , (c) IX I - X21, (d) X I /X 2.
224. Two friends A and B agree to meet between 12 (noon) and 1 p.m. at
a restaurant. Supposing that they arrive at random between 12 and 1 p.m.
independently of each other and the lunch lasts 30 minutes, what is the
probability that they meet in the restaurant? Let T be the instant of their
meeting. What is the conditional distribution of T, (a) given that they meet,
(b) given that they meet and A arrives first?
225. Let X and Y be independent with densities
1 1 Y -y/2a .
fx(x) = - f1---:z' Ixl < 1, I' ( )
JYY -ze
_ 2

n v 1- x (J

Show that X Y is N(O, (J2).

226.* Given n independent random numbers Xl' X 2 , ... , Xn from (0, 1),
called pseudorandom numbers, show that:
(a) Y = - :L?=l 2 log X I has the X2 distribution with 2n degrees offreedom;
(b) the variables
~ = -J=2~ cos(2nX 2 ),
1/ = J - 2 log X I sin(2nX2)'
are independent N(O, 1).
Thus (a) generates samples of X2 distributions with an even number of
degrees of freedom, while (b) generates a pair of independent normal variables.
227.* Let X be a continuous random variable X with distribution function
F( . ); consider the random variable

Y = F(X) = f:oo f(u) duo


This transformation is called the probability integral transformation.
Show that the distribution of Y is uniform in the interval (0, 1), that is,
Fy(y) = y, °< y < 1.
Given a random sample X I ' X 2, ... , Xn from the uniform distribution, show
60 7. Distribution of Functions of Random Variables

thatthesolutionsYl>Y2, ... ,YnofYl = F- l (xd'Y2 = F- l (X2), ... ,Yn = F-l(x n)


form a random sample from the F distribution.
228. (Continuation). Suppose the height X of men has the normal dis-
tribution N(167 cm, 9 cm 2 ) and a random sample of 10 men is taken with
heights Xl' X 2 , .•. , X IO • What is probability that the (random) interval
(j = [min(X l , ... , X 10), max(X l , ... , X 10)J covers at least 95% of the popula-
tion of heights? That is, the P{P[X E (jJ ~ 0.95} is to be calculated; note that
the P[X E (jJ, depending on the random interval (j, is a random variable.
229. X is called a lognormal variable, if the log X = Y has a normal
distribution N(fJ., (]"2).
(i) Find, (a) the density of X, (b) E(X) and Var(X).
(ii) If the Xi are independent lognormal random variables, their product
X 1 X 2'" Xn is also lognormal.

230. Show that if F(x, y) is the distribution function of X and Yand


Z = max(X, Y), W = min (X, Y),

then
Fz(z) = F(z, z), Fw(w) = Fx(w) + Fy(w) - F(w, w).

If F(x, y) is continuous find the densities of Z and W


231. (Continuation). If X and Yare independent N(O, I), show that
1
E{max(X, Y)} = In'
232.* Let X = (Xl' X 2 , •.• , Xd have the Dirichlet distribution. Find the
distribution, (a) of 1;. = Xl + X 2 + ... + Xb and (b) of 1'; = Xl + X 2 + ... +
Xi (i = 1, 2, ... , k).
233.* (Continuation). If Xi (i = 1,2, ... , k + 1) are independent Gamma
variables with parameters n l , ... , nk+l and )., respectively, show that the

y=. x.I
i = 1,2, ... , k,
, Xl + ... + Xk+l
have the k-dimensional Dirichlet distribution (5.11).
234. Let X and Y be independent Poisson variables with parameters A and
fJ.. Show that:
(a) the sum X + Y is also Poisson (use the convolution formula, cf. (7.3);
(b) the conditional distribution of X given that X + Y is binomial.
235. Let X and Y be independent binomial with parameters N, p and M,
p, respectively. Show that:
(a) the sum of X + Y is binomial (use the convolution formula);
(b) the conditional distribution of X given X + Y is hypergeometric, inde-
pendent of p.
Exercises 61

236.* Show that the mean


_ 1 n
X=- LXi
n i~l
and the variance
2 1 ~ - 2
S = ~~ L... (Xi - X)
n - 1 i~l
of the random sample X I' X 2' ... , Xn from N (tt, (T2) are independent. Hence
deduce the distribution of (n - l)s2.
Hint: (a) Use any orthogonal transformation Y = HX such that
Y,. = Jnx, Y = (Yl , ... , Y,.)" x = (Xl' ... , Xn)'·
- -

(b) Cov(X, Xi - X) = 0.
Show that Cov(X, S2) = tt3/n for samples from nonnormal populations, where
tt3 is the population central moment of order 3.

237.* Test for the Behrens-Fisher problem. To test whether two hetero-
scedastic normal populations have the same means one uses the statistic

t = [m(m - 1)]1/2(X - .Y)/L~ (u j -


2
U)2T ,

where X, yare the sample means of the independent samples from N (ttl' (Tn
and from N(tt2' (Tn, respectively, m ::;; nand uj = Xj - (~)Yj (j = 1,2, ... ,
m). Show that t has Student's distribution with m - 1 degrees offreedom when
ttl = tt2 = tt·
238.* Find the joint density of the ordered statistics X(l) < X(2) < ... < X(n)
from a distribution of the continuous type.
239. If Xl' X 2 , ... , Xn is a random sample from the continuous distribution
F. Find:
(a) the joint density function of the random variables

Y = F( min Xi)' Z = F( max Xi);


l~t~n l~l~n

(b) E(R) = E(Z - Y) where R is the range of the sample.

240. Let X(l) < X(2) < ... < X(n) be the ordered sample from a continuous
distribution F(x). Show that

E[X(k+l) - X(ka = G) r+",OCl F"-k(X) [1 - F(X)]k dx.

Hint: Use the joint density of X(kl' X(k+l)'


241.* A random sample of size n is taken from a uniform distribution in
the interval (0, 1). Find, (a) the mean E[X(k)]' (b) the distribution of R =
X(n) - X(l)'
62 7. Distribution of Functions of Random Variables

242. If Yl < Y2 < Y3 are the ordered observations of a random sample of


size 3 from a distribution with density
I, e - 1 < x < e + 1,
f(x) = { 0,
elsewhere.
For rJ. = f3 = 0.4 show that p[e - rJ. < Y2 < e + f3] = 0.944.
243. Let Yl < Y2 < Y3 be ordered observations of a random sample of size
3 from a distribution with density
2X. 0< x < 1,
f(x) = { 0,'
elsehwere.
Show that Zl = Yl /Y2 , Z2 = Y2 /Y3 , and Z3 = Y3 are completely independent.
244. If Xl' X z are independent f3 variables with densities f3(n l , nz) and
f3(n l + 1/2, n z ), respectively, where for n l > 0, n z > 0,
1
f3(m, n) = -- x m - l (1 - x)n-l, O<x<l.
B(m, n)

Show that Y = J XIX z has also the f3 distribution f3(2n l , 2nz)·


245. The random pair (X, Y) has density
1
f(x, y) = r(m}r(n) xm-l(y - x)n-l e - y , 0< x < y < 00.

Find the distribution of Z = Y - X. Are Z and X independent? (cf. Exercise


174).
246. Show that Z = 2ft¥, where X and Yare two independent r vari-
ables with parameters ()ol' n) and (2 z , n + 1/2), respectively, has the r distribu-
tion with parameters (2, 2n) where 2 = J2 l ).z.
247. For the pair (X, Y) of Exercise 245 show that W = X/Y has the
density f3(m, n) of Exercise 244.
248. If X and Yare independent N(O, 0"2), show that
XY
Z = -r===;===;c
JXz+y z '
are also independent normal.
249.* Let X have an n-variate normal distribution N(fl, I:). Find the
density of Y = AX where A is a nonsingular matrix.
250. The points of impact of two players AI' A z on a vertical target follow
the circular normal distribution with mean the center of the target and
variances O"f and 0"1, respectively. Find the probability that the impact point
of Al is closer to the center than that of A z .
CHAPTER 8

Limit Theorems. Laws of Large Numbers.


Central Limit Theorems

Elements of Theory
1. Convergence of a sequence of random variables. A stochastic sequence,
i.e., a sequence {Xn} of random variables defined on a probability space
{Q, B, P} may converge in several ways.
(a) In probability or stochastically or weakly to a random variable X
denoted by

if for every e > 0


lim P[IX. - XI> e] = o. (8.1)

(b) With probability one or almost surely (a.s.) or strongly to a random


variable X denoted by

if

p[lim Xn = X] = 1, (8.2)
.-00
or, equivalently, if for every e > 0

lim
N-oo
p[sup IX
n~N
n - XI> e] = o. (8.3)

(c) In quadratic mean to a random variable X, denoted by


Xn~X
64 8. Limit Theorems. Laws of Large Numbers. Central Limit Theorems

if

n~oo

(d) In law or distribution to X, denoted by


Xn~X
if at every continuity point x of the distribution function F of X
lim Fn(x) = F(x),

where Fn denotes the distribution function of X n.

2. Relations between the modes of convergence. For every constant e the


following hold:

(i) Xn ~ X => Xn ~ X; ( 1·1·) Xn P


--> e ¢>
Xn L
--> e;
(iii) Xn ~ X => Xn ~ X; (iv) Xn~X => Xn~X; (8.4)
L
00

and E(Xn - e)2 < 00 => Xn ~ e.


n=1

3. Laws of large numbers (LLN). They refer to the weak or strong con-
vergence of a sample mean Xn = (lin) L?=1 Xi to a corresponding population
(distribution) mean /1. Thus we have the
- p
Weak LLN (WLLN): If Xn --> /1;
Strong LLN (SLLN): If Xn ~ /1.
The basic tool in the proof of the WLLN is the so-called

Chebyshev-Bienayme inequality. If E(X) = /1, Var(X)= (1"2, then for every


constant e or A. (A. > I)
(1"2
P[IX - /11;:::: eJ ::; 2" or (8.5)
e
This follows from

Markov's inequality. If Y;:::: 0, i.e., P[Y;:::: OJ = I and E(Y) < 00, then for
every constant e > 0
P[Y;:::: eJ ::; E(Y) . (8.6)
e

The Chebyshev-Markov WLLN. If the random variables Xb ... , X n , •••


are independent and E(XJ = /1i, Var(XJ = (1"l with
Var(Xn ) --> 0 as n --> 00,
Elements of Theory 65

then
1 n
where [in = -
n
L i=l
l1i; (8.7)

as a consequence we have the WLLN for a random sample (i.e., independent


and identically distributed random variables) from a population with mean 11
and variance (J2, i.e.,
(8.8)
When the Xi are Bernoulli variables then (8.8) gives, the Bernoulli WLLN, i.e.,
the proportion Pn = x/n = Xn of successes!.. p, the probability of success in a
trial (see (2.5)).

Khinchine's WLLN. For a random sample Xl, ... , X n , .•. from a population
with mean 11 = E(XJ (i = 1, ... ,) the WLLN holds. This follows by the con-
tinuity theorem for characteristic function(s) (see Chapter 6) and relation (ii)
of (8.4) since
q>- (t) ----+ q> (t) = eil't.
Xn n-co Il

The classical Poisson LLN: If the Xi are Bernoulli with


i = 1, ... ,
then
1 n
where Pn = -
n
L Pi'
i=l

is a special case of (8.7).

Strong laws of large numbers (SLLN). E. Borel (1909) first showed the SLLN
for Bernoulli cases (as above)
Xn --->
a.s.
p.

This means that for every e > 0 and b > 0 there exists an integer N = N(e, b)
such that
P[IX n - pi < e for every n :2: N] :2: 1 - b.

Later Kolmogorov showed that under the conditions of (8.7)

Moreover for a random sample


Xn ~ 11 ~ E(XJ = 11, (8.8)*
i.e., the SLLN holds if and only if the population mean exists.
The proofs of these (see, e.g., Gnedenko (1962)) rest on the following
ineq uali ties.
66 8. Limit Theorems. Laws of Large Numbers. Central Limit Theorems

Kolmogorov's inequality. Let X I, ... , Xn be independent and let Sk =


XI+ ... + X k with E(Sk) = mk, Var(Sd = sf (k = 1,2, ... ). Then for every
e>O
1
P[ISk - mkl < es n, k = 1,2, ... , n] z 1 - 2' (8.9)
e

The Hajek-Renyi inequality. Under the conditions of (8.9), for every non-
decreasing sequence of positive constants Cn and any integers m, n, 0 < m < n,

p[ m<;k<;n
max cklSk - mkl Z eJ ~ -i(c;s; + f cf var(x »). k (8.10)
e k=m+1

Central limit theorems (CLT). They refer to the convergence of sums of


random variables (or equivalently of sample means) in law. The limiting law
is usually the normal. The proofs of CL T's, as a rule, rely on the continuity
theorem for characteristic functions (see Chapter 6):
L
Xn ~ X ¢:> <f>n(t) ~ <f>(t) ¢:> Fn(x) ~ F(x), (8.11)
where <f>n denotes the characteristic function of Xn and Fn the distribution
function of Xw

The Levy-Lindeberg CLT. If XI' ... , X n, ... is a random sample with


E(X;) = 11 and Var(X;) = (J2, then
n
r,; X _ .L (Xi - 11)
S* = v' f/( n 11) = 1=1 L N(O 1)
n- r:. - .,'
(J v n(J

i.e., for every x,


lim PES: ~ x] = <I>(x) = _1_ IX e- u2 / 2 duo (8.12)
n~oo fo -00

Lyapunov's CLT. Let X I, ... , X n, ... be a sequence of independent random


variables with
E(X;) = Ili,
and set
n )1/3
Bn = (
.L Pi
1=1
;
if
as n~ 00,

then
n
L (Xi - Ili)
S: = i=1
sn
~ N(O, 1).
n-+co
(8.13)
Exercises 67

Necessary and sufficient conditions for the validity of the CLT are given
by the

Lindeberg-Feller CLT. Let X n, )-tn' (Tn' Sn, S: be defined as above, and let
Fn be the distribution function of X n. Then for every e > 0,
i
max - - - 0 and S:.!+ N(O, 1)
1 ~i ~n Sn n-oo

The classical De Moivre- Laplace CLT for Bernoulli variables (the normal
approximation to the binomial distribution) is a special case of (8.12).

Exercises
251. If the independent random variables Xl, ... , X n, ... satisfy the condition
V(X;) ~ C < 00, i = 1,2, .. _,
then the SLLN holds.
252. In a sequence of random variables Xl' _.. , X n, ... suppose that X k
depends only on X k - 1, Xk+1' but that it is independent of all the other random
variables (k = 2, 3, ... ). Show that if V(XJ ~ N < 00 (i = 1, 2, .. _), then the
WLLN holds_
253.* If for every n, V(XJ
then the WLLN holds.
~ C < 00 and Cov(X;, X) < °
(i,j = 1,2, ... , n),

(i = 1, 2, __ .) and Cov(X;, Xj)


(Theorem of Barnstein).
-+ °
254.* Let {Xn} be a sequence of random variables so that V(XJ ~ C < 00
when Ii - jl -+ 00 then the WLLN holds

255.* If Xn .!+ X, Yn ~ c, then


L
(a) X. + L¥" -+ X + c;
(b) Xn Yn -+ cX;
L
(c) X./¥" -+ X/c.
256.* Let ~n = (X 1n , X 2n , ... , X k .)' (n = 1,2, ... ) be a sequence of random
vectors and ~ = (X l ' X 2, ... , X k )' a random vector with distribution function
F(x l ' .. - , x k ). If

for all constants c 1, ... , Ck , then the joint distribution function F~n of X In' _ •• ,
X k•
68 8. Limit Theorems. Laws of Large Numbers. Central Limit Theorems

has a limit and


F~JXI' ... , Xk ) --+ F(XI' ... , x k)·
257. Let {Xn} be a sequence of independent random variables uniformly
bounded, that is, there is a constant M such that P[Xk ~ M] = 1 for all k = 1,
2, ... , and suppose that Var(Xd oF 0 for every k.1f s; --+ 00, then the CLT holds
(cf. (8.13)).
258. Show that
n 1
L ~=-
. -n n k
hm e
n~CX) k=Ok! 2·
259. If X and Yare independent Poisson variables with parameters Al and
)'2' respectively, show that
X - Y - (AI - A2 ) L
(X + y)I/2 ~ N(O, 1).
A2 -'00

260. Show that for the sequences {Xn} of independent random variables
with
1- rn 1
(a) P[Xn = ± 1] = 2 ' P[Xn = ±2 ]
n
= 2n+ 1 ' n = 1,2, ... ,
(b) P[Xn = ±nA] = 1/2,
the CLT holds.
261. Compare the results given by Chebyshev's inequality and the CLT
for the probabilities
P[ -k < S: ~ k] for k = 1,2, 3.
262. A variable X has the Pareto distribution with density
x ~ 2.
Give the graph of the function g«(j) = P[IX -Ill> (j] where 11 = E(X), and
compare this with the upper bound given by Chebyshev's inequality.
263. Given that Var(X) = 9, find the number n of observations (the sample
size) required in order that with probability less than 5% the mean of the
sample differs from the unknown mean 11 of X more, (a) than 5% of the
standard deviation of X, (b) than 5% of 11 given that 11 > 5. Compare the
answers obtained by using Chebyshev's inequality and the CLT.
264. In a poll designed to estimate the percentage p of men who support
a certain bill, how many men should be questioned in order that, with
probability at least 95%, the percentage of the sample differs from p less, (i)
than 1%, (ii) than 5%, given that, (a) p < 30%, and (b) p is completely unknown.
265. Each of the 300 workers of a factory takes his lunch in one of three
Exercises 69

competing restaurants. How many seats should each restaurant have so that,
on average, at most one in 20 customers will remain unseated?
266. The round-off error to the second decimal place has the uniform
distribution on the interval (- 0.05,0.05). What is the probability that the
absolute error in the sum of 1,000 numbers is less than 2?
267. The daily income of a card player has uniform distribution in the
interval ( - 40, 50).
(a) What is the probability that he wins more than 500 drachmas in 60
days?
(b) What amount c (gain or loss) is such that at most once in 10 times on
average the player will have income less than c during 60 days?
268. In the game of roulette the probability of winning is 18/37. Suppose
that in each game a player earns 1 drachma or loses 1 drachma. How many
games must be played daily in a casino in order that the casino earns, with
probability 1/2, at least 1,000 drachmas daily? Then find the percentage of
days on which the casino has a loss.
269. Show that the r distribution of Exercise 202 tends to the normal
distribution as s ~ 00.
CHAPTER 9

Special Topics: Inequalities,


Geometrical Pro babili ties,
Difference Equations

Elements of Theory

A. Inequalities

In Chapter 8 several inequalities such as the Chebyshev, Markov, and


Kolmogorov inequalities were given. Here we give some inequalities con-
cerning expectations.

1. A function g, defined over the interval ((l(, [3), is called convex in ((l(, [3) if
for each A (0 :;:; )" :;:; 1), and every pair Xl' X 2 E ((l(, [3), we have
(9.1)

Remark. (a) If 9 is continuous in ((l(, [3) then the validity of (9.1) for A = 1/2
implies the convexity of g.
(b) If the second derivative g"(x) exists, then (9.1) is equivalent to
g"(x) 2 o. (9.2)
The function 9 is called concave if - 9 is convex.

2. Jensen's inequality. Let g(x) be a convex function in ((l(, [3) and P[X E
((l(, [3)] = 1. If E[g(X)] < 00, then

E[g(X)] 2 g(EX). (9.3)

The proof is based on the observation that there exists a line of support of the
curve y = g(x) through the point (11, g(l1» with equation y = g(l1) + A(X - 11)
Elements of Theory 71

such that

3. Cauchy-Schwarz inequality. If E(X2) and E(y2) exist then


[E(XY)J2 :s;; E(X2)E(y2); (9.4)
equality holds only if Y = AX (A constant). See Exercise 103.

4. Holder's inequality. Let p > 0, q > 0, such that p-l + q-l = 1 and X, Y
positive random variables. Then
E(XY) :s;; [E(XP)JI/P[E(yq)Jl/q. (9.5)
This gives (9.4) when p = q = 1/2.
Proof The function
g(X) = log x
is concave for x > 0, i.e., for each A (0 :s;; A :s;; 1)
), log Xl + (1 - A) log X2 :s;; IOg(AXI + (1 - A)X 2)·
Hence
xtx1-). :s;; AX + (1
I - A)X 2,
and setting A = p-l, 1 - ), = q-l, Xl = XP/E(XP), X2 = yq/E(P), we obtain
X Y XP yq
[E(XP)] lip + [E(yq)Jllq :s;; AE(XP) + (1 - A)E(yq)'
Taking expectations gives (9.5).

5. Minkowski's inequality. For every integer k ;::::: 1 and any random vari-
ables X and Y with E(Xk) < 00, E(yk) < 00,
[EIX + Ylkr lk :s;; [EIXn l/k + [EI YlkJl/k. (9.6)
If we define the norm of X by IIXII = [EIXlkJl/k, then (9.6) becomes the
triangle inequality
IIX + YII :s;; IIXII + I YII.
Proof For k = 1, obvious. For k > 1, replacing Y by IX + Yl k - l in (9.5),
we obtain
EIX + Ylk:s;; E(lXIIX + Yl k - l ) + E(IYIIX + Yl k - l )
(9.7)
:s;; [(E IXlk)l/k + (E IYlk)l/kJ [E IX + YI(k-l)'r /',
where k- l + S-l = 1 and therefore (k - l)s = k. Dividing both sides of (9.7)
by (E IX + YI)I/' we obtain (9.6).
72 9. Special Topics: Inequalities, Geometrical Probabilities, Difference Equations

B. Geometrical Probabilities

Geometrical probabilities refer to events corresponding to subsets of a bounded


set S of the Euclidean n-space En under the assumption that the distribution
over S is uniform. Thus the probability of an event
A = {x: x E SA C S}
is given by

P(A) = /1(SA)
/1(S) ,

where /1(R) denotes the Lebesgue measure (geometric size) of the set R, i.e., its
length, area, volume, etc., depending on the dimensionality of R. The solution
of problems in geometrical probability contributed a lot to the understanding
of the role played by the specification of the random experiment related to a
certain event. This is demonstrated, e.g., in the classical Bertrand paradox (see
Exercise 169).

C. Difference Equations

In many cases a problem in probability can be solved more easily by con-


structing and solving a related difference equation, i.e., an equation of the form
g(x, f(x), 8f(x), ... , 8 rf(x» = 0,
where f is the unknown function and 8 denotes the forward difference operator
defined by
N(x) = f(x + h) - f(x) for some fixed h > 0,
and 8 rf(x) = 8(8 r- 1f(x» for r ~ 1 (8 Of == f).
If x = 0, 1,2, ... , then we set
Yn = f(n), n = 0,1,2, ... ,
and we have, e.g.,

8Yn = Yn+1 - Yn'


Of special interest in applications are the linear difference equations with
constant coefficients lXi' i.e., of the form
Yn+r + 1X1Yn+r-1 + ... + IXr- 1 Yn+1 + IXrYn = Zn' (9.8)

where n usually ranges over a set of positive integers. If IXr '# the difference °
equation is said to be of order r. The difference equation is called non-
homogeneous (complete) or homogeneous according to whether Zn '# or Zn = 0. °
Elements of Theory 73

The characteristic polynomial tp(A) of (9.8) is defined by


tp(A) = Ar + 1X1 Ar - 1 + ... + IXr - 1 A + IXr •

The solution of the homogeneous equation


Yn+r + 1X 1Yn+r-1 + ... + IXr- 1Yn+1 + IXrYn = 0 (9.9)
is based on the

Proposition. (i) With each simple root A of the characteristic equation


tp(A.) = 0 let us associate the special solution cA. n (c any constant) of the
homogeneous difference equation (9.9).
(ii) With each real root A of multiplicity k of tp(A) = 0 associate the special
solution
(co + c1 n + C2n2 + ... + Ck_l nk - 1 )A n,

where co, c 1, •.. , C k - 1 are arbitrary constants.


(iii) With each pair of (conjugate) complex roots oflength (absolute value)
p and argument () associate the special solution
pn[Ao cos(n() + Bo) + ... + Ak_ 1 nk- 1 (cos(n() + Bk-dJ,
where k is the mUltiplicity of the complex root and Ao, ... , A k - 1 , Bo, ... , Bk - 1
arbitrary constants.
The sum of the above special solutions of (9.9) determine the general
solution of (9.9) containing r arbitrary constants whic~ are determined by the
initial or boundary conditions satisfied by (9.9).

General solution ofthe complete equation (9.8). This can be found by adding
a special solution of (9.8) to the general solution of the homogeneous (9.9).

Example. Solve the nonhomogeneous equation of the second order

Yn+2 - Yn+l - 6Yn = 2n, n = 0,1, .... (9.10)


The roots of the characteristic equation
tp(A.) = A2 - A. - 6 = 0
are Al = 3, A2 = - 2, and by the preceding proposition the general solution
of the homogeneous equation
Yn+2 - Yn+1 - 6Yn = 0,
IS

Y: = c 1 3n + c 2 (-2)",
with c 1 , C 2 arbitrary constants.
For the general solution of (9.10) we must first find a special solution y~
74 9. Special Topics: Inequalities, Geometrical Probabilities, Difference Equations

which is to be added to y:. We consider the special solution


(9.11 )
where e has to be determined to satisfy (9.10); substituting (9.11) into (9.10)
gives
i.e., y" = -(t)2".
Thus the general solution of (9.10) is
y" = y: + y~ = e 1 3" + e2 (-2)" - (t)2".

Exercises
A. Inequalities

270. If g(x) ~ 0 for every x and g(x) ~ e for x E (a, P), then
P[X E (a, P)] s e- l E[g(x)].
271. (Continuation). Show that for every constant t > 0
1
P[X > t] S )2 E(X + e)2.
(t +e
272. If Xl and X 2 . are independent and identically distributed random
variables then for every t > 0
P[IX l - X21 > t] s 2P[IX l l > !t].
273. Cantelli's inequality. One tail Chebyshev's inequality. If E(X) = 11,
Var(X) = (J2, then
(J2

P[X - 11 s e] S
(J
2
+e
2' e < 0,

e ~ 0,
(see Exercise 271).
274. * Gauss inequality. If Xo is the mode of a continuous unimodal distribu-
tion and ,2
= E(X - xof then
4
P[IX - xol ~ .h] s 9A. 2 '

275. If g(x) ~ 0 and even, i.e., g(x) = g( -x) and in addition g(x) is non-
decreasing for x > 0, show that for every e > 0

P[IXI > ] < Eg(x)


- e - gee) .
Exercises 75

276. (Continuation). If g(x) of Exercise 275 satisfies Ig(x)1 ::;; M < 00, then

P[IXI :;::.: e] :;::.: Eg(X~ g(e).

277. (Continuation). Let g be as in Exercise 275 and P[IXI ::;; M] = 1 then

P[IXI > c] > Eg(X) - g(c).


- - geM)
278.* Berge's inequality. Let E(Xi ) = Ili' Var(X;) = (1? (i = 1,2) and
p(x l , x 2 ) = p, then

p[max (IXI - (11


Ilil , IX 2 -
(12
] 1 + Ji=P2
1l21) > c::;; 2 •
C

279.* Jensen's inequality for symmetric matrix functions (Cacoullos and


Olkin, Biometrika, 1965). The symmetric matrix function G(X) of the matrix
X of order m x n is called convex if, for every pair X I' X 2 of m x n matrices
and 0 ::;; ). ::;; 1, by analogy to (9.1), it satisfies
G(..1.XI + (1 - ).)X2 ) ::;; ..1.G(Xd + (1 - ..1.)G(X2 ),
where for symmetric matrices X, Y we write X :;::.: Y if X - Y is a nonnegative
definite matrix. Let X be a random matrix, i.e., a matrix whose elements are
random variables and G a convex matrix function. Then G(EX) ::;; EG(X).
280. For a positive random matrix show that
E(X- I ) :;::.: (E(X)fI.
281. Show that if E(Y) < 00 and Y> 0 then
E 10g(Y) ::;; log E(Y).
282. For independent and identically distributed random variables Xl' X 2'
... , Xn with P[Xi > 0] = 1 and Var(log X;) = (12 show that for every 8> 0

Hence deduce

B. Geometrical Probabilities

283. A point is chosen at random on each of two consecutive sides of a


rectangle. What is the probability that the area of the triangle bounded by the
two sides of the rectangle and the line segment connecting the two points is
less than 3/8 of the area of the rectangle?
76 9. Special Topics: Inequalities, Geometrical Probabilities, Difference Equations

284. Two points are selected at random on segment AB which is thus


divided into three parts. What is the probability that the three line segments
can form a triangle?
285. What is the probability that three points selected at random on the
circumference of a circle lie on a semicircle?
286. A line segment of length 1 is divided into five parts by four points
selected at random on it. What is the probability that each part is less than 1/2?
287.* A line segment AB is divided by a point C into two parts AC = a
and CB = b. Two points X and Yare taken on A C and CB, respectively. What
is the probability that AX, XY, BY can form a triangle?
288.* BufJon's needle problem. A smooth table is given with equidistant
parallel lines at distance 2a. A needle of length 2J1. (J1. < a) is dropped onto the
table. What is the probability that the needle crosses a line?
289. * A closed convex curve with diameter less than 2a is dropped onto the
table of Exercise 288. Find the probability that a line is crossed by the curve.

C. Difference Equations
290. A has a 1O-drachma coins and B has b 5-drachma coins. Each of them
takes a coin out of his pocket and gives it to the other. This is repeated n times.
What is the probability that A gives a to-drachma coin to B on the nth
exchange?
291. A tosses a die until an ace or a six appears twice.
(a) What is the probability that the game terminates with the appearance
of an ace after no more than n tosses?
(b) What is the probability that the game ends with the appearance of an
ace?
292. A professor has three lecture rooms A, B, and C available each year
for teaching a course. Since he wishes to use a different classroom every year,
at the end of the year he chooses one of the other two rooms for the next
academic year as follows. The first year he teaches in A and selects between
Band C by tossing a coin. If heads appears he chooses B, otherwise he selects
C. At the end of each year he makes his selection in the same way. Find the
probability Pn that he teaches in A during the nth year. Similarly for Band C.
293. Find the probability P. that in a sequence of n Bernoulli trials, with
probability of success (S) p the pattern SF does not appear at all (F == failure).
294. Let Yk be the expected number of occupied cells when placing (at
random) k balls into n cells. Find Yk:
(a) by using difference equations;
(b) by referring to the expected value of a binomial.
Exercises 77

295. A pair of dice is thrown n times; at each throw we observe whether a


double six appears or not. Show that the probability Po of an even number of
double six's satisfies the difference equation
17 1
Po -18 Po - 1 = 36'

Hence find Po'


296. * Each of n urns contains w white balls and b black balls. One ball is
transferred from the first urn to the second urn, then another ball from the
second urn is transferred to the third one, and so on. Finally, we draw a ball
from the nth urn. What is the probability that it will be white?
297. A and B play the following game. A throws a coin I with probability
of heads P until he gets tails; then B begins throwing another coin II with
probability of heads pi until he gets tails. Then A begins again and so on.
Given that A begins the game, what is the probability of heads at the nth
throw?
298. A throws two dice and he gets r points if r aces appear (r = 0, 1,2);
the game stops when no ace appears for the first time. Find the probability Po
that he has n points at stopping. If the game continues indefinitely, what is
the probability that A has n points at some point of the game?
299. A box contains a good lamps and b bad lamps. At each draw of a
lamp, if it is good it is kept in the box, otherwise it is replaced by a good one
(from another box). What is the expected number of good lamps in the box
after n trials? Hence calculate the probability of selecting a good lamp at the
nth trial.
300. Let Pk denote the probability that no cell takes more than two balls
when placing k balls in n cells at random. Find, (a) Pk (b) limk _ oo Pk and
n = constant, (c) lim Pk for k = constant and n ~ 00.
CHAPTER 10

General Exercises

301.* Using the Cauchy-Schwarz inequality show that


get) = log E(IXlt)
is a convex function. Then show that [E(lxlt)]l/t is an increasing function of t.
302. * Every real characteristic function qJo(t) satisfies the inequality
1 - qJo(2t) :s; 4(1 - qJo(t)).

303. * Every real characteristic function CJJo(t) satisfies


1 + qJo(2t):2: 2 {qJo (t)} 2.
304. If Xl' X 2 , X 3 , X 4 are jointly normal with COV(Xi' X) = E(XiX) = (1ij'
show by the use of moment generating functions, that
E(X 1 X 2 X 3 X 4 ) = (112(134 + (114(123 + (113(124·
305. Let Xl' X 2 , ••• , Xn be a random sample from the uniform distribution
in (0, 1) and let y" = n min(X 1, X 2 , •.• , Xn). Using characteristic functions,
show that the asymptotic distribution (n --> 00) of Yn is the exponential e- Y,
y > 0.

306. Let (Xl' Yd, ... , (Xn' y") be a random sample from the bivariate
normal with means 0, variances 1, and correlation p. Let the random variable

Z. = {I if (Xj - ~)(lj - ~) > 0,


J ° if (Xj - X)( lj - Y) < 0.
Find E(2).
307.* n tickets are numbered 1 through n. Of these, r tickets drawn at
random are the lucky ones. A lucky ticket receives an amount equal to the
10. General Exercises 79

number it bears. What is the expected amount given on every draw? What is
the variance of this amount?
308. In the generalized binomial with P[Xi = 1] = Pi (i = 1,2, ... , n), show
that as n --+ 00 the distribution of the number of successes X = L7=1 Xi
tends to the Poisson with parameter A where A = lim n_ oo L7=1 Pi assuming
maxi Ai --+ 0.
309. Let X be the number offailures preceding the nth success in a sequence
of independent Bernoulli trials with probability of success p. Show that if as
n --+ 00, q = 1 - P --+ 0, so that nq --+ A, then

310. If the continuous random variable X is such that for some integer
k > 0, kX has the Gamma distribution and the conditional distribution of Y
given X is Poisson, then Y has a negative binomial distribution.
311. Let Xl' X 2 , .•• , Xn be a random sample from the Laplace distribution

-00 < x< 00.

By using characteristic functions show that the random variable

is distributed as (0"/2n)X~n' Hence deduce that E(Y) = 0".

E(X i ) = °and Var(X;)


312. Let Xl' X 2, ... ,
=
Xn be a random sample from a distribution with
1. Show that

j~sn
Y" = -n-- ~ N(O, 1),
L Xl

JFi
i=l

Sn
Zn = n ---+
n-a) N (0, 1),
L i=l
xl
where Sn = Xl + X2 + ... + X n.
313. Let X(1)' ... , X(n) be the ordered sample from a uniform distribution
in (0, 1) and Y,. = X(1) + X(2) + '" + X(r)' Show that the conditional distribu-
tion of W = y"/l(r+1) is the distribution of the sum of r observations of a
random sample from the same uniform distribution.
314.* N points Xl' X 2 , ... , Xn are selected at random in (0,1). Show that
80 10. General Exercises

the distribution of the sum Sn = Xl + X 2 + ... + Xn has density


f,,(X) = 1 {xn-I _ (n)(X _ l)n-1 + ... }
(n - I)! 1
= n {(n - x)n-I _ (n - 1) (n - 1- x)n-I + ... }
(n - 1)1 n 1 n- 1 '
where the sum extends as long as x-I, x - 2, ... and (n - 1 - x), (n - 2 - x),
... are positive.
315. Let X I, X 2, ... , Xn be independent discrete variables with the same
uniform distribution on the set of integers 0, 1, ... , m - 1, i.e.,
1
P[XI =j] = - for j = 0, ... , m - 1.
m
Find the probability generating function of the sum Sn = Xl + ... + Xn and
hence the probability function of Sn (Feller, 1957, Exercise 18, page 266).
316. (Continuation). A die is thrown three times. What is the probability
that the sum of the outcomes is 1O?
317.* Generalized Banach match box problem. A chain smoker has m + 1
match boxes; each time he lights a cigarette he chooses one of the boxes at
random. Suppose that (to begin with) every box contains N matches. Let Xl'
X 2 , ••• , Xrn be the numbers of matches left in the remaining m boxes, when:
(a) a box is found empty for the first time;
(b) any box is emptied first.
Note: The case m = 1 is known as the Banach match box problem (Feller,
ibid, p. 157).
318. (Continuation). Suppose that box Ck (k = 1, ... , m) is chosen with
probability Pk. What is the probability that Ck empties first?
319. Show that Xl' X 2, X3 with ajoint discrete distribution are independent
if and only if
P[X3 = x31X2 = X2' XI = Xl] = P[X3 = X3]

for all (Xl' X2) with P[XI = XI' X 2 = x 2] > 0 and


P[X2 = x 2 1X I = Xl] = P[X2 = x 2] for all Xl with P[XI = Xl] > o.
320. Let X I' X 2, ... , Xn be independent and identically distributed positive
random variables with E(X;) = /l, E(Xi-l) = r. Let Sn = XI + X 2 + ... + X n.
Show that E(S;;I) exists and

_ -1,
E ( -Xi) - i = 1, ... , n.
Sn n
(Feller, ibid, Exercise 36, page 226.)
10. General Exercises 81

321. (Continuation). Show that

E (Sm) m
Sn = -;;- for m ::;; n,

322. A caterpillar is moving on the edges of a tetrahedron ABCD on whose


top there is glue. In a unit of time the caterpillar goes from any vertex (except
D) to any other vertex with the same probability 1/3. Suppose that the
caterpillar at time t = 0 is on the vertex A. What is the probability that
(a) the caterpillar finally gets stuck;
(b) the caterpillar finally gets stuck coming from vertex B?
323. A flea moves at random on a plane with leaps of constant length a and
in a random direction at each step. If it starts from the origin, let (Xn' y"f be
the position of the flea after n jumps. Where
n n
Xn = L a cos ()i'
i=l
Y" = L a sin ()i'
i=l

The angles ()i are independently uniform in (0, 2n).


(a) Find, (i) E(Xn ), E(Y,,), Var(Xn ), Var(y"), (ii) E(R;) where R; = X; + y"2.
(b) Show that the X n , Y" are uncorrelated but not independent.
(c) Find the distribution of (Xn' y") for large n (n -+ 00) and the density of
Rn. What is the expected distance of the flea from the origin for large n?
324. * A non normal distribution with all marginals normal. Let the random
variables XI' ... , Xn have the joint density

f(x I ' ... , Xn) = (2 1 (1 ~ 2) [On


n)
nl2 +.
exp --2.L...
,=1
Xi 1
,=1
(Xi 12»].
exp( -2"Xi

Show that any n - 1 of the XI' ... , Xn are independent normal with mean 0
and variance t. Are the X I, ... , Xn jointly normal?
325.* In Exercise 288 suppose the plane is divided into squares by equi-
distant lines parallel to the axes. Let 2a be the side of each square and 21l the
length of the needle. Find the average number of lines crossed by the needle,
(a) Il > 0(, (b) when Il is arbitrary.
326. * Let X, Y be independent X2 variables each with n degrees of freedom.
Show that

z=JnX-Y
2 JXY
has the Student distribution with n degrees of freedom. Hence deduce that:

(a) tn=f(ft:- k).


where Fm,n denotes an F variable with m and n degrees of freedom.
82 10. General Exercises

(b) If tn(a) and Fn,n(a) are the upper percentile points of tn and Fn,n, respec-
tively, express one in terms of the other.
327. If for a given value A. of A the random variable X(A) is Poisson with
parameter A. and A is a random variable with a gamma density
aP
r(A.) = r(p) A.P-l e -a..,
show that X(A) has the negative binomial distribution

Pk = P[X(A) = kJ = ( ~p) ( _ p)kq P, k = 0,1, ... ,

where p = (1 + a)-l, q = a(a + 1)-1.


328. * Show that for any two random variable(s) X and Y with Var(X) < 00,

Var(X) = E[Var(XI Y)] + Var[E(XI Y)].


329.* Using the distribution of X(/) where Xli)' ••• , X(n) is the ordered
sample from the uniform in (0, 1), show the following relation between the
incomplete B function lx(nl' n2) and the binomial distribution function:

t
k=r
(n)pkqn-k = Ip(r, n - r
k
+ 1)
= 1 fp xr-1(1 _ x)n-r dx.
B(r, n - r + 1) Jo
SUPPLEMENTS
SUPPLEMENT I

Miscellaneous Exercises

I-I. For any two events A, B show that the following relationships are
equivalent: A c B, A' :=J B', A u B = B, and A n B' = 0.
1-2. Show that (A.6.. BY = (AB) u (A' B') and P(A.6.. B) = P(A) + P(B) -
2P(AB), where .6.. is the symmetric difference, defined in Exercise 34.
1-3. Let n be the number of elements in the sample space Q and N(E)
the number of elements in the event E. Show that the data: N = 1,000, N(A) =
525, N(B) = 312, N(C) = 470, N(AB) = 42, N(AC) = 147, N(BC) = 86, and
N(ABC) = 25 are inconsistent.
Hint: Check N(A u B u C).
1-4. (Lewis Caroll, A Tangled Tale, 1881). In a fierce battle, at least 70% of
the soldiers lost an eye, at least 74% lost an ear, at least 80% lost an arm, and
at least 85% lost a leg. What is the smallest possible percentage of soldiers
that each lost an eye, an ear, an arm, and a leg?
Answer: 10%.
1-5. In a market research survey, from a sample of 1,000 smokers, 811 said
that they prefer brand "A", 752 prefer "B", and 418 prefer "C". Furthermore,
570 said that they like both "A" and "B", 356 that they like both "A" and "C",
348 that they like both "B" and "C", and 297 that they like all three brands.
How did the researcher discover that the results were inconsistent?
Hint: How many prefer at least one of the three brands?
1-6. We throw two dice once. Let A: doubles, B: at least one die shows 5
or 6, C: the sum of the outcomes of the two dice is less than 6.
(a) Find P(A), P(B), P(C), P(AB), and P(AC).
(b) Using only the results of (a) calculate: P(A u B), P(B u C), and P(A u C).
Answer: (a) 1/6,5/9,5/18, 1/18, 1/18. (b) 2/3,5/6, 7/18.
86 Supplement I. Miscellaneous Exercises

1-7. Three athletes are participating in the Marathon. Their chances of


finishing under 2 hours are 1/3, 1/5, and 1/12, respectively. Their times for the
duration of the course are independent. What is the probability that at least
one finishes in less than 2 hours?
Answer: 23/45.
1-8. A horse has probability p of jumping over a specific obstacle. Given
that in 5 trials it succeeded 3 times, what is the conditional probability that
it had succeeded in the first trial?
Answer: 3/5.
1-9. In a train car 3 seats are facing the front and 3 are facing the rear. Two
women and three men enter the car and sit at random. What is the probability:
(a) that the two women are facing each other?
(b) that two men are facing each other?
Answer: (a) 1/10. (b) 3/10.
1-10. On the basis of the following data, examine the relationship between
the father having brown eyes (event A) and the son having brown eyes (event
B). The percentages observed were: AB: 5%, AB': 7.9%, A'B: 8.9%, and A'B':
78.2%.
Answer: P(B/A) = 0.39 and P(B/A') = 0.10.
1-11. We throw three dice. What is the probability that at least one 6
appears given that the three outcomes were completely different. Generalize

G)IG)'
the result for the case of a regular polyhedral die.

Answer:

1-12. Ninety percent ofa population have a television set (event A), while
80% have a car (event B). What is the minimum percentage of the population
with a car that also have a television?
Answer: P(A/B) ~ 87.5%.
1-13. Let P(A) = (I. and P(B) = 1 - I: where I: is a small number. Give the
upper and lower bound of P(A/B).
Answer: ((I. - 1:)/(1 - 1:) ~ P(A/B) ~ (1 - 1:).
1-14. Let Xl' X 2 be independent random Bernoulli variables with the same
parameter p and let Y = 0 if Xl + X 2 = even and Y = 1 if Xl + X 2 = odd.
(a) For what values of p are Xl and Y independent?
(b) Using (a), define three events A, B, and C such that they are pairwise
independent but not completely independent.
Answer: (a) p = 1/2. (b) A = {Xl = O}, B = {X2 = O}, C = {Y = O}.
1-15. We throw three dice twice. What is the probability of having the same
result the second time when the dice are, (a) not distinguishable? (b) distin-
guishable?
Answer: (a) 1/56. (b) (6 + 90 x 3 + 120 X 6)/6 6 •
Supplement I. Miscellaneous Exercises 87

1-16. At a party, A is introduced to 6 women and 4 men. What is the


probability that he identifies all the couples correctly if he knows that there
are, (a) 4 couples, (b) 3 couples?
Answer: (a) 1/360. (b) 1/480.
1-17. A young person driving a motorcycle on a highway with 4 lanes
changes lane every 15 seconds. What is the probability p of finding himself in
the lane in which he started 1 minute later?
Answer: The answer depends on whether the initial lane is an interior or
an exterior one. In the first case p = 5/8, and in the second case p = 1/5.
1-18. In city A it rains half of the days of the year and the weather forcast
is correct with probability 2/3. Mr. Sugar worrying about getting wet always
takes his umbrella when the forcast calls for rain and with probability 1/3
otherwise. Find the probability that:
(a) he is caught in the rain without his umbrella;
(b) he has his umbrella with him when there is no rain.
Answer: (a) 2/9. (b) 5/9.
1-19. The telegram signals (.) and (-) are sent with proportion 3:4. Due
to faults in transmission a dot (.) becomes (-) with probability 1/3. If a signal
has arrived as ( . ) what is the probability that it was transmitted correctly?
Answer: 43/84 (Bayes formula).
1-20. A die is thrown 10 times and let the maximum outcome be M and
the minimum m. Find the P(m = 2, M = 5).
Hint: Consider the probability P(m ~ 2, M ~ 5).
Answer: (4/6)10 - 2(3/6)10 + (2/6)10.
1-21.* A theorem of Poisson. An urn contains b black balls and w white
balls. n balls are drawn from the urn at random, their color unnoticed, then
m additional balls are drawn. Show that the probability that there are k black
balls among the m balls is the same as if the m balls were chosen from the
beginning (i.e., before choosing the n balls).
In other words, in sampling from a finite population the final (hypergeo-
metric) probabilities are not affected by the content of an initial sample when
we do not know that content. Obviously, if we know the composition of the
first sample (i.e., the colors of the n balls), then, in general, the probabilities
are affected.
Hint: Verify first the case m = 1, calculating the probability P(An) of
choosing a black ball on the n + 1 trial; show that P(An) = b/(b + w) with
O~n~b+w-l.

1-22. After an earthquake the books of a large library fall on the floor
and the servant, who does not know how to read, res helves them in any
order.
(a) What is the probability that exactly k books are placed back in their
initial position?
88 Supplement I. Miscellaneous Exercises

(b) How many books are expected to be reshelved in their initial position?
Answer: (a) e-1/kL (b) 1.

1-23. John Smith's problem. John Smith in 1693 posed the following prob-
lem: Is the probability for each of the three players to win the same if: the first
must obtain at least one ace throwing 6 dice, the second at least two aces
throwing 12 dice, and the third at least three aces throwing 18 dice? Newton
and Toilet found that the first player has a better chance than the second one,
and the second player has a better chance than the third one.

1-24. A homogeneous and regular die with v faces is thrown n times. Let
p(n, k) be the probability that a given face appears less than k times. Show
that:
(a) p(vn, n) is a decreasing function of v for given n;
(b) p(vn, n) < 1/2;
(c) p(2n, n) --+ 1/2 as n --+ 00 (De Moivre central limit theorem).

1-25. The Massachusetts State Lottery issues 1 million tickets. The first
winning ticket wins $50,000, the next 9 winning tickets win $2,500 each, the
next 90 tickets win $250 each, and 900 tickets win $25 each. What is the
expected gain of a person buying, (a) one ticket? (b) ten tickets?
Answer: E(I?=l Xi) = I?=l E(XJ (a) $0.1175. (b) $1.175.

1-26. (Continuation). What is the answer to (a) and (b) if:


(i) the state issues 2 million tickets?
(ii) the winning tickets win twice as much?

1-27. In a survey on the housing problem, 2 apartments are chosen from


each of 4 buildings (having both owner-occupied as well as rented apartments).
If we know that:

Building No. 1 234


Owner-occupied 5 482
Rented 8 10 9 10

What is the expected number of owner-occupied apartments in the above


sample?
Answer: 2(5/13 + 4/14 + 8/17 + 2/10). Use additivity of expectations.

1-28. Out of 365 people:


(a) What is the probability that at least two have their birthday on 2 April?
(b) What is the expected number of people having their birthday on 2
April?
(c) What is the expected number of days in a year that are birthdays: (i) for
at least one person? (ii) for at least two persons?
How can you approximate the answers to the above questions?
Supplement I. Miscellaneous Exercises 89

Answer:
(a) 1 - (364/365)365 - 365(364)364/365 365 ~ 1 - 2e- 1 = 0.26.
(b) 1.

(c) (i) 365 {1 - G~:r65} ~ 365(1 - e- 1 ) = 231. .


(ii) 365p where p was given in (a).
1-29. The duration T of a telephone call satisfies the following equation:
P[T> t] = p·exp[ -At] + (1 - p)·exp[ -,ut] for t > 0,
where 0 :s; p :s; 1, A > 0, ,u > O. Find E(T) and Var(T).
Answer: E(n = piA + (1 - p)/,u. Var(n = 2p/A 2 + 2(1 - p)/,u2 - [E(n]2.
1-30.* The generalized problem of points by Pascal. Two gamblers A and
B playa series of independent games in each of which A has probability p of
winning and B has probability q = 1 - p of winning. Suppose that at a given
moment the series of games is interrupted and A needs m additional points
and B needs n additional points to be the winner. The bet should be split
between A and B in the ratio P(A):P(B), where P(A) = 1 - P(B) is the prob-
ability that A eventually wins. Show that

PA = L
m+n-l (m + n- 1) pk - qm+n-l-k = L
n-l (m + k - 1) pmqk.
k=m k m=O k

Note: The above solution was first obtained by Montmort (1675-1719).


Explain the idea behind each term in the expressions for PA • Try to show the
equality between those two expressions for PA using the properties of binomial
coefficients (see Chapter 1).
1-31. An employee leaves between 7:30 a.m. and 8:00 a.m. to go to his
office, and he needs 20-30 minutes to get to the railway station. The departure
time and the duration of travel are distributed independently and uniformly
in the corresponding intervals. He can take either of the following two trains:
the first leaves at 8: 05 a.m. and takes 35 minutes; and the second leaves at
8: 25 a.m. and takes 30 minutes.
(a) What time, on average, does he arrive at his office?
(b) What is the probability that he misses both trains?
(c) How many times is he expected to miss both trains in 240 working days?
Answer: (a) 8: 50. (b) 1/24. (c) 10.
1-32. Show that if X and Yare independent, then (cf. Exercise 328)
Var(XY) = Var(X)·Var(Y) + Var(X)·[E(y)]2 + Var(Y)·[E(X)Y
Conclude that if Xl, ... , Xn are completely independent with means zero, then

Var (11 X) = 11 Var(X;).


90 Supplement I. Miscellaneous Exercises

1-33. We toss a fair coin until two consecutive heads or tails appear. What
is the probability that an even number of tosses will be required?
Answer: 2/3.
1-34.* Maxwell-Boltzmann statistics. s distinguishable balls are placed
randomly into n different cells. Find the probability that exactly r cells will be
occupied.
Answer:

Hint: The probability that exactly k events occur among n events A 1 ,


... , An is given by (see, e.g., Feller, 1957),

Sr - (r: 1)Sr+1 + ... + (-l y G)Sn


(cf. (Ll)).
1-35. A large board is divided into equal squares of side a. A coin of
diameter 2r < a is thrown randomly onto the board. Find the probability that
the coin overlaps, (i) with exactly one square, (ii) with exactly two squares.
Answer: (i) (1 - 2r/af. (ii) 1 - 4(r/a)2.
1-36. A square of side a is inscribed into a circle. Find the probability that:
(i) any randomly selected point of the circle will be an interior point of the
square.
(ii) of eight randomly selected points in the circle, three fall inside the
square, two in one of the four remaining parts of the circle, and one in each
of the other three parts of the circle.
Answer: (i) 2/n. (ii) (8! 4/3! 2! 1! 1! 1!)(2/n)3(1 - 2/n)SCl/4)5.
1-37. The distance between two successive cars on a highway follows the
exponential distribution with mean 20m. Find the probability that there are
90 to 110 cars on a 2-km stretch of the highway.
Answer: e- 100 It280 100k/k! ~ 0.68.
1-38. The number of cars crossing a toll bridge follows the Poisson distri-
bution with an average of 30 cars per minute. Find the probability that in 200
seconds no more than 100 cars will pass through.
Answer: The sum of independent exponentials is Erlang (i.e., gamma with
s = 100, see (2.11));

1
"'--'-"""'-;1'""0"""0
fOO 99 -t/2
e
_ ~
dt - L.. e
-100 lOok .
99! 2 200 t k=O k!
1-39. (Continuation). Establish the relation

An f.00 e -AXx n-1 dx = n~l


L.. e
-At (At)k
--,
(n - I)! t k=O k!
Supplement I. Miscellaneous Exercises 91

between the Erlang distribution function and the Poisson distribution func-
tion.
Hint: The probability that less than n events occur in time t, according to a
Poisson distribution (process) X(t) with

is equal to the probability that the waiting time w,. until the nth event exceeds
t. Hence, also conclude that the density of w,. is the Erlang density on the left
side of (*) above.
1-40. (Continuation). Show that
1 fOO e- x x"-1 dx =
"-1
L Ak
e- A _ .
(n - I)! A k=O k!
°
1-41. Define <I>(a; b) = <I>(b) - <I>(a) for < a < b with <I> the distribution
function of N(O, 1). Show that <1>(0; 1) > <1>(1; 2). Generalize this result for any
equal intervals.
Answer: <I>(a + x) - <I>(a) > <I>(b + x) - <I>(b); x > 0.
1-42. Using the fact that if X follows the normal distribution N(O, 1), then

P[X2 ::s; x] = (2/n)I/2 I I


x e- u2/ 2 du,

conclude that

r(1j2) = I() x(i/2)-le- x dx = In.


1-43. The probability of hitting a target with one shot is PI and the
probability of destroying the target with k(k ~ 1) shots is 1 - p~. Find the
probability of destroying the target after n shots.
Answer: 1 - [1 - Pl(1 - P2)]".
1-44. Let X follow a Poisson distribution with parameter A. We select N
random points from the interval [0, 1]. Let Xi be the number of points in the
interval [(i - 1)/n, i/n] (i = 1, ... , n). Show that the random variables Xi are
independent and identically distributed.
1-45. If X and Yare independent with P[X = 0] = P[X = 1] = 1/2 and
P[y::s; y] = y (0 ::s; y ::s; 1), find the distribution ofthefollowing random vari-
ables W, Z where:
(i) W = X + Y;
(ii) Z = XY.
Answer:
(i) few) = 0.5, 0< w < 2;
z = 0,
(ii) fez) = {0.5,
0.5, o<z<1.
92 Supplement I. Miscellaneous Exercises

1-46. Let X be a uniform random variable on the interval (-a, a) and Ya


random variable independent of X, with distribution function F(y). Show that
the distribution function of G = X + Y is

G(z) = -
1 fa F(z - x) dx.
2a -a

1-47. Let X be uniformly distributed on the interval (0, 1). Find the prob-
ability density function of:
(i) Y = aX + b; a, b constants (a > 0);
(ii) Z = I/X;
(iii) W = g(X), where g is a continuous and monotone function of x in (0, 1).
Answer:
(i) Uniform;
(ii) fez) = l/z2, 1 < z < 00;
(iii) few) = 1/lg'(w)l·
In addition to Problems 1-23 (John Smith, 1963) and 1-30 (points of Pascal),
it is worth giving here some other problems which are of some historical
interest, and show the difficulty in solving probability problems encountered
by even well-known figures in the history of mathematics.
1-48. Luca dal Bargo or Pacciali, 1494. Each of three players A, B, and C
has probability 1/3 of hitting a target better than the other two. At each round
of the game each shoots at the target once. The winner is the first player to
win six rounds. They bet 10 ducats each. At a certain point, however, when A
has 4 victories, B has 3 victories, and C has 2 victories, they have to terminate
the game. How should they split their bet? (cf. Problem 30).
1-49. Huyghens,* 1657. A and B playa game in which each of them throws
two ordinary dice. A wins if he scores 6 and B wins if he scores 7. First, A
throws the dice once, then B throws the dice twice, and they continue alter-
nating after each throw until one of them wins. What is the probability of
winning for each of them?
Note: Huyghens and Spinoza gave the same answer:

P(A)/P(B) = 10355/1227 ~ 0.84.


(cf. Exercise 67).
I-50. Huyghens, 1657. A deck of forty cards consists of 10 red, 10 black, 10
blue, and 10 green cards. Four cards are drawn at random and without
replacement. What is the probability that all the colors will show up?
(Huyghens gave the answer 1,000/9,129 = 0.109; is it correct?)
1-51. Huyghens, 1657. An urn contains 8 black balls and 4 white balls. Each
of three players A, B, and C (and in that order) draws a ball without replace-

* He is the author of the first book on probability, De Ratiociniis in Ludo Aleae, 1654.
Supplement I. Miscellaneous Exercises 93

ment and the first one to draw a white ball wins. What is the probability of
winning for each of them?
Hint: A can win on the first, fourth or seventh draw, etc.
1-52.* Montmort, 1708. Three players A, B, and C play the following game.
They play three rounds for each of which, each of them has probability 1/3 of
winning. A is the winner of the game if he wins a round before B or C wins
two rounds; B is the winner if he wins two rounds before player A wins one
round, or C wins two rounds; C wins like B. What is the probability of winning
for each of them?
(Montmort gave P(A) = 15/27, P(B) = P(C) = 5/27).
1-53.* Daniel Bernoulli, 1760. A container has 2N cards, numbered in such
a way that each of the numbers 1, 2, ... , N appears twice. 2N - n cards are
drawn at random. What is the expected number of pairs of cards remaining
in the container?
(Bernoulli found n(n - 1)/(4N - 2)).
I-54. Euler, 1763. N lottery tickets are numbered 1 to N. n tickets are
selected at random. What is the probability that r consecutive numbers are
selected (r = 1,2, ... , n)? /(10)
Example: For N = 10, n = 4, r = 4, the probability is 7 4 = 1/30.

I-55. * Condorcet, 1785. The probability of event A is p. What is the prob-


ability that:
(a) in n trials event A occurs k times consecutively?
(b) in n trials event A occurs k times consecutively and then A also occurs
C

k times consecutively?
(c) in n + m trials event A occurs n times given that in r + k trials it
occurred r times?
Condorcet gave the following answer for (c):
(r + k + I)! (r + n)! (k + m)!jr! k(r + k + m + n + I)!.
Laplace and G. Polya disagreed with this solution. What do you say?
1-56.* Laplace, 1812. In a lottery with N tickets, n numbers are chosen as
winners. What is the probability p that in k lottery games all the N numbers
will appear?
(Laplace for N = 90, n = 5, and k = 85 gave p = 1/2).
SUPPLEMENT II

Complements and Problems

1. Multivariate Distributions
1.1. The density of the impact point (X, Y) of a circular target is given by

f(x, y) = c(R - Jx 2 + y2)


Find, (a) the constant c, (b) the probability p that the impact point falls in a
circle with radius a < R centered at the origin.
Answer: (a) c = 3/R2. (b) p = 3a 2/R2(1 - 2a/3R).
1.2. The random variables X, Y satisfy the linear relation
aX - bY = c,
where a, b, c are constants. Find, (a) the correlation p(X, Y), and (b) the ratio
ux/uy of the standard deviations.
Answer: (a) p = sg(ab). (b) ux/uy = Ib/al.
1.3. The dispersion (variance~covariance) matrix of a three-dimensional
normal distribution is
2
6 -2] 3 .
3 8

If /lx = /ly = /lz = 0, find the den~ f(x, y, z) and its maximum.
Answer: f(x, y, z) = 1/(2nj230n) exp{ - 2jo(39x 2 + 36y2 + 26z 2
- 44xy + 36xz - 38yz)}
fmox = f(O, 0, 0) = (2nJ230n)-1 = 0.00595.
1. Multivariate Distributions 95

1.4. The joint density of X and Y is given by


f(x, y) = ye- y (x+l), x> 0, y > 0.

Find, (a) the marginal densities of X and Y, and (b) the conditional distribution
function Fx(xIY) of X given Y = y.
Answer: (a) fx(x) = l/(x + 1)z, fy(y) = e- Y • (b) 1 - e- YX •
1.5. Suppose that the random variable X coincides with the random vari-
able Xi with probability Pi' that is,
P[X = XJ = Pi and E(X;) = m i , i = 1,2, ....

Show that
V(X) = I Pi V(X i ) + V(M),
i

where P[M = mJ = Pi (i = 1,2, ... ).


1.6. The random vector (X, Y, Z) has density f(x, y, z). X, Y, Z can only be
observed simultaneously. An observation gave the values u, v, w without
knowing the correspondence between them and X, Y, Z. Find the probability
that X = u, Y = v, and Z = w.
Answer:
f(u, v, w)
{j(u, v, w) + f(u, w, v) + f(v, u, w) + f(v, w, u) + f(w, u, v) + f(w, v, u)} .
1.7. The joint distribution function F(x, y) of the random variables X and
Y is given by
F(x, y) = sin x sin y, °s x S n/2, °s y s n/2.
Find, (a) the density function f(x, y), and (b) the dispersion matrix.
Answer: (a) f(x, y) = cos x cos y. (b) (J'; = (J'; = n - 3, (J'xy = 0.
1.8. The normal density ofa point A = (Xl' Xz, X 3) is given by

What is the probability that A will fall into an ellipsoid, with principal
semiaxes E 1 , E z , E3 along the axes Ox, Oy, Oz, where E 1 , E z , E3 are the
probable errors of X, Y, Z, respectively, i.e.,
P[IXI < E 1 ] = P[I YI < E z ] = P[IZI < E 3] = 1/2.
Answer: (1/2) - (J2Eo/Jn)e-l/ZE6, where Eo is the probable error of
N(O, 1).
1.9. The vector (X, Y) has the uniform distribution in the ellipse
XZ yZ
aZ + bZ S 1.
96 Supplement II. Complements and Problems

Find:
(a) the marginal distributions of X and Y;
(b) the conditional distributions of X given that Y = y, and the conditional
distributions of Y given that X = x;
(c) the Cov(X, Y) = axy .
Are X, Y independent?~--:--::;-,-::;-:-
Answer: (a)fx(x) = 2J1 - (x 2/a 2)/na,fy(y) = 2J1 - (y2/b 2)/nb (Ixl < a,
Iyl < b).
(b) fr(ylx) = (2bJ1 - (x2/a 2 »-1for Ixl < a, Iyl < bJ1 - (x 2/a 2);fx(xIY)
similar.
(c) axy = 0 but they are dependent since f(x, y) "# fx(x)fy(y).

1.10. The point (X, Y, Z) is uniformly distributed in the sphere x 2 + y2 +


Z2 ~ R2. Find the density of Z and the conditional density f(x, Ylz). Generalize
for an n-dimensional sphere.
Answer: fz(z) = 3(R2 - z2)/4R 3 , Izl < R, f(x, ylz) = 1/n(R2 - Z2).
1.11.* Given fy(y), E[Xly], V(Xly), find E(X) and V(X).
Answers:

E(X) = f: E(Xly)fy(y) dy = Ey[E(XI Y = y)],

+ V[E(XI Y)]

t:
V(X) = E[V(XI Y)] (Exercise 328)

= f: V(Xly)fy(y) dy + [E(Xly) - E(X)]2fy(y) dy.

1.12. The vector (X, Y) has normal density


f(x, y) = ce-4X2_6xy-9y2.
Find the densities fx(x), fy(y), fx(xly), fy(Ylx) and the constant c.
Answer:

c=--,
3)3
n
3)3
fy(y) = - - exp {27
-- y 2} ,
2Jn 4

fx(xly) = }.e-<2X+1.5Y )"

Note that they are all normal.


1.13.* Suppose that X has a spherically symmetric n-dimensional distribu-
tion, i.e., with density f(xI + ... + x;). Prove that the density g of the "general-
ized X2 variable" U = X'X is given by
1. Multivariate Distributions 97

where Cn is the "surface" area of the n-dimensional sphere, i.e.,


2nn/2
Cn = r(n/2)·

Hint: Make use of spherical (polar) coordinates r, 81 , ... , 8n - l : XI = r sin 81 ,


k-I n-l
Xk = r sin 8k [] cos 8i , k = 1, ... , n - 1, Xn = r [] cos 8i ,
i=l i=l

where -tn < 8i :::;; tn (i = 1, ... , n - 2 and - n < 8n - 1 :::;; n). Prove that the
Jacobian is r [];;=f (r cOSn-k- 1 8k ) and make use of J':i~/2 COS k - 1 8 d8 = B(t, k/2).
1.14.* (Continuation). For n = 2, show that the polar coordinates (R, 8),
where
x = R cos 8, Y = R sin 8,
are independent, 8 is uniformly distributed on [0, 2n), and R has density (cf.
Exercise 221)
h(r) = 2nrf(r 2 ).
1.15. * (Continuation). Show that the correlation coefficient based on the
single observation (X, y), i.e.,

has density
1 1
f(ro) = - - I 2 ' -1 < ro < 1.
n vi 1 - ro
Hint: ro = sin 28 has the same distribution as under a spherical normal
(centered at the origin). Hence conclude that tan 28 = ro/J1 - r6 has the
Cauchy distribution (t with one degree of freedom).
1.16. * If X = (X I ' ... , Xn)' is N (0, In) and X is partitioned into k subvec-
tors X(l)' ... , X(k) with VI' V2 , ... , Vk components, respectively, show that
(X(l)X(1), ... ,X(k)X(k))/IXI 2 has the Dirichlet distribution (see (5.11)), with
density
r - (n)
f(t I , · · · , tk-I
2
) ---k-~() (1
nr -
Vk
- t
1-
...
-
t
k-I
)(v k /2)-1
k-l
[]
.
,=1
t(v;/2)-1
i ,

i=1 2

Hence, for VI = V2 = ... = Vs = 1, vs + I = n - s (k = s + 1),


(lXII, IX2 1,·· ., IXkl)
IXI
98 Supplement II. Complements and Problems

has the density

Vi ~ 0,

k
if L u'f < 1.
i=l

This is the distribution of any k-dimensional subvector of an n-dimensional


vector
U=(U1 ,···,U.)
uniformly distributed on the (surface of the) unit sphere
xi + ... + x; = 1.
1.17. * (Continuation). In general, if X has a spherical distribution (see Prob-
lem 1.13), since this is invariant under rotations in n-space, the distribution of
the direction X/IXI is the same as that of V and is independent of its length
IXI (for any spherical distribution with P[IXI = 0] = 0).
Hence conclude that the results of Problem 1.16 hold if X is any spherically
symmetric random vector.
1.18. Let X, Y be random variables with
E(X) =}1, E(Y) = '1, VeX) = (Jf, V(Y) = (Ji, Cov(X, Y) = (J12·
Find an approximate formula for
E[g(X, Y)], V[g(X, Y)],
using the linear expansion of g(x, y) around (}1, '1) (up to second-order mo-
ments). Generalize for n random variables.
Answer:

E[g(X, Y)] ~ g(}1, '1)


1 [02g 2 02g
+ 2 ox2 (J1 + 2 oxoy (J12 + oy2 (J2 ,
02g 2J

Og)2 2 og og (Og)2 2
V[g(X, Y)] ~ ( ox (J1 + 2 ox oy (J12 + oy (J2'

where the partial derivatives are evaluated at the point (}1, '1).
Let X = (Xl' ... , X.)" E(X) = (P1' ... , }1.)' = p, Cov(X) = 1: = «(Ji)' the
covariance matrix of X (see (5.7)), and

Vg = (::1' ... '::.)'.


1. Multivariate Distributions 99

where the derivatives of g(x 1, ... , x n ) are evaluated at p. Then


1 n n
E[g(X)] ~ g(p) + "2 i~ j~ gij(lij'
V[g(X)] ~ (Vg)'~(Vg).

1.19. X and Yare independent and identically distributed random vari-


ables with density given by
2
f(x) = r.--::i' O~x~1.
lt y 1 - x

Using Problem 1.18 find the E(Z) and V(Z) where


X
Z=arctan Y'

Answer:
E(X) It
E(Z) = arc tan - - ~ -
E(Y) 4'
1.20. The fundamental frequency of a chord is given by the formula

Q=~ fF
2 '-l;;L'
where F is the tension, m is the mass of the chord, and L is its length. If the
mass is assumed to be constant and F, L are random variables with E(F) = f,
E(L) = I, V(F) = (lJ, V(L) = (I,z, and Cov(F, L) = (III' find an approximation
for the variance of Q.
Answer:

20 J
p +r - fl
f [(lJ (I,z jl
(see Problem 1.13).
V(Q) ~ 16ml

1.21. Let the vector (X, Y) have density f(x, y). We define the complex
variables
Z=X + iY,
and Zt = Zeit. Prove that, in order for all Zt to have the same distribution, it
is necessary that f(x, y) = g(X2 + y2) for some density g (see Problem 1.13).
Hint: The distribution of Zt has to be invariant under orthogonal trans-
formations (rotations).
1.22. If X has density f as in Problem 1.13, show that
y =p + AX (a nonsingular n x n constant matrix)
has density of the form
cIAr I/2f((y - p)'A -l(y - p», A=AA',
100 Supplement II. Complements and Problems

and characteristic function of the form

e;t'''I/I(t' At) for some function 1/1.

1.23. Let Pij = p(X;, XJ be the correlation coefficient between X;, Xj for i,
j = 1, ... , n. For n = 3 and Pl2 =
P13 = P23 = p, what are the possible values
that P can take?

°
Answer: - 1/2 ::s; P ::s; 1. Prove that the correlation matrix P
negative definite, i.e., t' Pt 2 for all t' = (t I ' ... , tn).
= (p;J is non-

1.24. If the p vector X is N(J1, A), show that the quadratic form

has the X~ distribution, with p degrees of freedom, and density given by (cf.
Problem 1.13)

1.25. * The entropy H of a multivariate distribution with density f(x) is


defined by the formula

H(f) = - E[log f(X)] = - f. . f f(x) log f(x) dx.

Prove that for the normal distribution of Problem 1.24

1.26. If X is N(J1, A) with density f, find the density g of the random variable

Y = f(X).

Hint: See Problem 1.24.


1 ]( PI2 1-1
g(y) = 2Jc [ log cy2 ' c = (2nVIAI.

1.27. If X = (X I ' ... , Xn) is uniformly distributed in the simplex T:

Xl + X2 + ... + Xn ::s; 1, X; 2 0, i = 1, ... , n,

show that the density of X is


f(x) = n! for xE T

Show that the (n - I)-dimensional marginals are also special cases of the
Dirichlet distribution (5.11).
2. Generating Functions 101

2. Generating Functions
2.1. Two persons shoot at a target, each has n shots. Using generating
functions, find the probability P that they have the same number of successes,
given that each of them has a probability 0.5 of success (hitting the target) at
each shot.
Answer: The coefficient of the constant term in the expansion of the gener-
ating function
1 1( l)n (1 + U )2n
G(t) = 2n (t + 1)" 2n 1 + t = 4nu n '

that is,

P
= ~(2n).
4n n

2.2.* To obtain the title of champion in chess, an opponent has to win at


least 12.5 points out of the maximum of 24. In the case of a tie (12: 12), the
defending champion maintains the title. The probability that each of the two
players wins a game is the same, and equals half the probability of a tie. Find:
(a) the probability Pch that the defending champion will maintain his title,
as well as the probability Pop that the opponent will become the champion;
(b) the probability P that 20 games will be played in the contest.
Answer: (a) The probability Pch equals the sum of the coefficients of the
nonnegative powers of t in the expansion of
1 1 1)24 (1 + t)48
G(t) = ( 4t + 4t + "2 = 448t24 '

Pch = 4;4 k~4 (:8) = ~ 4;4 [2 48 + G!) ] = 0.5577, Pop = 1- Pch·

(b) The probability of the complementary event equals the sum of the
coefficients of t, with exponents between -4 up to 3 in the expansion of
1 (1 + t)20
G(t) = 4 20 t 20 ;
1
P = 1 - 20
4
L
23
k=16
(40)
k = 0.22.

2.3. In a lottery, tickets are numbered from 000000 to 999999 and they are
all equally probable. What is the probability P that the sum of the digits of a
chosen number be 21?
Answer: The probability P is the coefficient of t 21 in the expansion of
1 1 (1 - t 1 0)6
G(t) = -(1 + t + t 2 + ... + t 9 )6 = - --~
10 6 10 6 (1 - t)6
102 Supplement II. Complements and Problems

where we made use of the equality

-~~~ =
(1 ~
1tt 1+ (n n ) t + (nn + 1) t
~ 1 ~ 1
2 + ....
So

2.4. Prove that the Pascal distribution, for r -+ 00 and rq -+ A, converges


to the Poisson with parameter )..
2.5. Let X, Y, Z be independent random variables, each taking the values
1,2, ... , n with the same probability (uniformly). Find P[X + Y = 2Z] from
their joint generating function G(t1' t 2 , t3)'
Answer: Equal to the constant term 1/2n of the generating function Gw of
W=X + Y~2Z.
-2 (1 ~ S·)2(1 ~ S2.)
Gw(s) = G(s, s, s ) = n3(1 ~ sf(1 ~ S2)S2' 2'

2.6. In Exercise 315, let


q.,k = peS. ::;; k].
Find the generating function Q(t) of q•. k for a given n and hence q•. k'
Answer: 1 ~ P(t) = (1 ~ t)Q(t) (where P(·) is the generating function of S.).
So
q.,k = 1 ~ n1 "L.... (~1)J. . (n) (k + n~ mj ) .
m j<k/m ] n
2.7. (Continuation). Suppose k and m tend to infinity so that kim -+ x. Find
the limit of q•. k'
Answer:
1 ~~
n.
~ (~l)j(~)(X~j)'= 1 ~F(x),
J<X ]

where F is the distribution function of the sum of n independent random


variables from the uniform distribution (0,1) (see Exercise 314).
2.8. Find the densities of the distributions with characteristic functions
1 + it 1 ~ it
<P1(t)=1+t2' <P2(t) = ~1-~2'
+t
Answer: 11 (x) = e- x , x > 0, 12 (x) = eX, x < 0.
2.9. Find the moments of the Laplace distribution with characteristic
function
2. Generating Functions 103

Answer: Using the Taylor series expansion


1 OC!
" Ct)2k
1 + t2 = k-f-O I ,

conclude that
I
J1k =
{k! for even k,
o for odd k.
2.10. Determine the distribution with characteristic function
1
cp(t) = 2e-il _ 1.

Answer: Discrete random variable with probability function


k = 1,2, ....
Make use of the expansion of cp(t) in powers of !e il and the expression of the
Dirac function J

J(x) = - 1 fOC! e i1x dt.


2n -00

2.11. Find the characteristic function of Y = aF(X) + b, where X is a


continuous random variable and F is its distribution function.
Hint: Y is uniform on (a, b); see Exercise 227.
2.12. A vector I; = (X l ' ... , X.) follows the n-dimensional normal distribu-
tion with E(XJ = a, V(XJ = (J2 (i = 1, ... , n) and covariances
i = 1,2, ... , n - 1,
COV(Xi' Xj) = 0 for li-jl>l.
Find the characteristic function of 1;.
Answer:

cp(t l ' ... , t.) = exp { ia L



tk -
(J2.
- L
tf - p(J2
.-1}
L tktk+1 .
k=1 2 k=1 k=1

2.13. Let cp(t) be the characteristic function of a random variable X with


E(X2) = J1~.
(a) Prove that
1
t/!(t) = ---,- cp"(t)
J12
is a characteristic function.
(b) Conclude that

is a characteristic function.
104 Supplement II. Complements and Problems

Answer: (a) !/I(t) is the characteristic function of the random variable with
distribution function

1
G(x) = ---;- IX u 2 dF(u).
J12 -co

(b) Application of (a) with X normal N(O, 1).


2.14. Show that
I - (Itlln) for It I < n,
aCt ) = {
° forltl>n,
is the characteristic function of a random variable X, and determine the
corresponding distribution.
Answer: Since la(t)1 is integrable, X is continuous with density

f (x) -- ~
2
n
In
-n
e
-iIX( 1 -l!l)d _
n
t -
1 - cos(nx)
2 2
n x
.

2.15. Let Xl' X 2 , X3 be independent N(O, 1). Find the characteristic function
of the pair Zl' Z2' where

Hence conclude the characteristic function of Zl' Are Zl' Z2 independent?


Answer:
(f1(tl' t 2) = (1 + d + tn- 1/2 ,
(f1z,(t) = (f1(t, 0) = (1 + t 2 )-1/2 = (f1z,(t).
No, because (f1(tl' t 2) =F (f1z,(t l )(f1Z,(t 2)·
2.16. Find the distributions corresponding to the characteristic functions
I I I
(f1l(t) = - ht' (f12(t) = -'-h-' (f13(t) = - h 2 •
cos sm t cos t

Answer:
. 1
fl(X) = , f3(X) = - - - - .
nx 2 . h nx
2 cosh 2 sm 2
2.17. Bivariate negative binomial. Show that
P(Sl' S2) = p~(1 - P1Sl - P2 S2)-"
with positive parameters and Po + Pl + P2 = 1 is the probability generating
function of a pair (X, Y), so that the marginal distributions of X and Yand
X + Yare negative binomials.

2.18. Problems related to a game of billiards. Let a "round" consist of a


2. Generating Functions 105

sequence of Bernoulli trials before the first failure. Find the generating function
and the probability function of the total number Sk of successes in k rounds.
Answer: Pascal distribution.
2.19. (Continuation). Let G be the number of successive rounds up to the
nth success (i.e., the nth success occurs during the Gth round). Find the dis-
tribution of G and the
E(G), V(G).

Answer:
"-I
P[G = k] = I P[Sk-1 = np[Xk ~ n - k]
j=O

"~I
= L... q
k-I .(k+ -2) _.(k+n-2)
pl
j
. p" 1 ,
j=O ] n- 1
nq
E(G) = 1 + nq , V(G) = 2".
p P
2.20.* (Continuation). Consider two independent sequences of Bernoulli
variables with parameters PI and P2' respectively, or two players with "skills"
PI and P2' respectively. Prove that each player will need the same number of
rounds (trials) until the Nth success (a tie) with probability

(PIP2)N l (N : ~ ~ 2}qlq2r l
=
N
(PIP2) (1 - qlqz)
I_ZN N- I
k~O
(N-1)
k
k
(qlqz) .

2.21. Show that the characteristic function of the f3 density (2.1 0) is


r(p + q) 00 r(p + j)(it)j
r(p) j~O r(p + q + j)ru + 1)
2.22. * Show that the characteristic function of the density in Problem 1.13
is of the form cp(ti + t~ + ... + t~) (see Problem 1.21). If

cp(d "
+ ... + t~) = Il cp(t;),
i=l

that is, X I ' X z , ... , X" are independent, prove that X is N(O, 1). In other words,
a spherically symmetric random vector has independent components if and only
if it is multivariate normal.
Hint: Equation (*) is known as Hamel's equation and its only continuous
solution is
cp(t) = eel

for some constant c (which for a characteristic function must be < 0).
106 Supplement II. Complements and Problems

3. Transformation of Random Variables


3.1. Find the density of
y= +j[XT,
where X - N(O, 1).
Answer:

fy(y) =
4y
.j2n exp -2 '
{y4} y ~ 0.

3.2. X has a uniform distribution on (0,1). Find the density of the random
variable Y defined by

X = !{1 + _2 IY exp {-~} dt}.


2 fo 0 2
Answer: Y is N(O, 1) (see Exercise 227).
3.3. The random variables X and Yare related by

X = f:oo f(t) dt,


°
t:
where f(t) ~ satisfies the

f(t) dt = 1.

Prove that if X is uniform on (0, 1), then f(t) is the density of Y (see Exercise
227).
3.4. Suppose X and Yare independent random variables. Prove that the
product XY has the same distribution if:
(a) X and Yare N(O, 1); or
(b) X is N(O, 1) and Y has density
(X2 density; see (2.11)).
°
3.5. The roots of x 2 + Yx + Z = follow the normal distribution (-1, 1).
Find the density of the coefficients Y and Z (cf. Exercise 222).
Answer:
fy(y) = ±(2 - Iyl) for Iyl s 2,
fz(z) = -t log Izl for Izl s 1.
3.6. Let f(x, y, z) be the density of a point (X, Y, Z) in the three-dimensional
space and
8 . Y
= arc sm R'

What is the density of the pair (R, 8)?


3. Transformation of Random Variables 107

Answer: f(r, e) = r2 cos eJ5" f(r cos ecos cp, r cos e sin cp, r sin e) dcp (cf.
Problem 1.11).
3.7. X and Yare independent normal N(O, 0'2). Find the distribution of (a)
Z = X/I YI; (b) W = IXIII YI.
Answer: (a) Cauchy Distribution (cf. Exercise 220). (b) f(w) = (2/n) x
[1/(1 + w 2 )] (folded Cauchy).
3.8. X and Yare independent random variables with uniform distribution
on (0, a). Find the distribution of Z = X/Y and examine the existence of
moments of Z.
Answer:
.l z ~ 1,
2'

f(z) = {
_1
2Z2'
z ~ 1.

There are no ordinary moments.


3.9. The joint density of X, Y is
f(x, y) = e- Y for ° ~ x~y < 00.

(a) Find the marginal distributions of X, Y and the conditional distribution


of Y for given X.
(b) Ofthe variables X, Y, Y - X, and X/Ywhich are pairwise independent?
(c) In the conditional distribution of Y, given X = x, determine the interval
(y, y + a) so that

Q(a) = P[y ~ Y ~ Y + a]
can be maximized. These intervals with variable x define a zone B. What is
the probability of the zone, i.e.,
P[(X, Y) E B] = P(B)?
Answer: (a)

fx(x) = Loo e- Y dy = e- x ,

fy(Ylx) = eX-Yo
(b)

(X, Y - X) and (Y, X/y)o


(c)

Q(a) = sup f u +a
e X - Y dy = sup ex-u(l - e- a ) (for u = x).
u~x u u~x

P(B) = Q(a) as also concluded from the independence of X and Y - X.


108 Supplement II. Complements and Problems

3.10. * X, Yare independent exponentials each with density


t ~ O. (1)

(a) Find the joint conditional density of X, Y, given that X ::;; Y (ordered
sample of two observations).
(b) Let Xl ::;; Xl::;; X3 ::;; X 4 be an ordered sample from (1). What is the
density fl of(Xb Xl) and what is the conditional density fl of (X 3, X 4 ) for a
given (Xl' Xl)? What do you conclude?
Answer: (a)
f(x, y) = 20 1 e- 8(x+ y ),
O<x<y (see Exercise 238),
fy(Ylx) = Oe- 8(y-X\
i.e., Y - X, given that X x, has the density (1).

f
=

m(x) = E[YIX = x] = x
oo
yfy(ylx) dy = x + o·1
This also follows from the fact that Y - X, given X = x, follows (1), whose
mean is I/O.
(b)
fl (Xl' X2) = 120 1 e- 8 (x 1 +3x 2 ),

f2(X 3 , x 4 ) = 201e-8(x3+x4-lx2),

We observe that, for Xl = Xl, the pair (X 3 - Xl' X 4 - Xl) is distributed as


(X, Y) in (a), i.e., the distribution is not affected by the knowledge that X
exceeds a given value, because of the "lack of memory" of the exponential.
3.11.* If the random variables Xi are independent normal N(a + bt i, (Jl)
(i = 1, ... , n), where ti are constants with 2:7=1 ti = 0, find:
(a) the joint distribution of X I ' ... , Xn;

I
(b) the joint distribution of
_ 1 n
X =- 2: Xi and b =
n
i~ tiXi i~ tl.
n

n i=I
Answer: X is N(a, (Jl/n) and b is also N(b, (Jl/2:7=I t?).
3.12. * (Continuation). By a proper orthogonal transformation of X =
(X I ' ... , Xn)', say Y = OX (0 n x n orthogonal matrix), prove that the random
variables X, band 2:7=1 (Xi - X - btY are completely independent. Con-
clude that 2:7=1 (Xi - X - btY/(Jl has the Xl distribution with n - 2 degrees
of freedom.
3.13. X and Yarejointly normal with E(X) = E(Y) = 0, V(X) = V(Y) = 1,
and p(X, Y) = p, Ipi < 1. Consider the polar coordinates R = JXl + yl
4. Convergence of Random Variables 109

and () = tan- 1 (y/X). Show that () has density (cf. Exercises 220, 221)

f(()) =.JI=P2
2n( 1 - P sin 2())'
0 < () < 2n.

3.14. An elliptically symmetric t distribution. The random vector X =


(X 1,... , X k ) has the density

f(x) = r((k + n)/2) [1 + (x _ )'A -1( _ )]-(n+k)/2


r(k/2)(kn)kI2 JI x JI .

If A = I and k = 1 this is the Student distribution with n degrees of freedom.


If A = h, then X has a spherically symmetric t distribution (see Problem 1.13).
Show that:
(a) E(X) = JI.
(b) X has the same distribution as
JI + A 1/2 Z/X~,
where Z is N(O, Id independent of the X~. Hence conclude that (cf. Problem
1.24)
1
F = - 2 (X - JI)' A-I (X - JI) is Fk,n'
kXn
(c) As n -+ 00,

X~Z.
3.15. Let f(x) denote the density of a random variable X. If f( .) is monotone
and bounded, then f(X) has a uniform distribution if and only if X has the
(negative) exponential distribution.
Hint: See (7.1) or Exercise 227.

4. Convergence of Random Variables


4.1. A Geiger-M iiller counter registers a particle emitted by a radioactive
source with probability 10- 4 .
(a) Assuming that during a certain period, the source emitted 3 x 104 , what
is the probability that more than two particles will be registered.
(b) How many particles have to be emitted so that at least four particles
will be registered with probability 0.99?
Answer: (a) 1 - e-J.[l - ), - (A,2/2)] with A, = 3 X 104 X 10- 4 = 3.
(b)

X ::= 10.7, n= X' 104 ::= 107,000.


110 Supplement II. Complements and Problems

4.2. In some of the countries of western Europe, from the seventeenth


century up to World War I, the following state lottery was played: 5 numbers
out of 90 were drawn and a player had the right to bet, say, an amount a, on
one or more numbers; if all the numbers he had chosen were among the 5
numbers drawn, then he would win an amount determined as follows: 15a if
he had bet on one number, 270a if he had bet on 2 numbers, 500a if he had
bet on 3 numbers, 75,OOOa if he had bet on 4 numbers, and 1,OOO,000a if he
had bet on 5 numbers.
(a) What is the expected gain of a player who is betting on 1,2, 3,4, and
5 numbers?
(b) Suppose that 100,000 players bet on 3 numbers; what is the probability
that at least 10 players win?
Answer: (a) Let Pk be the probability that a player who bets on k numbers
will win, and that Ek will be his expected gain. Then

( 90 - k)
5-k
;
1 2 1
Pk = C~) P1 = 18' P2 = 801' P3 = 11 ,748'

1 1
P4 = 511,038' Ps = 43,949,268'
1 1
E1 = 15a 18 - a = -(ja,

(b) 0.24.
4.3. * Making use of the Stirling formula prove that, as A -+ 00 and for a
constant k, we have the local limit theorem for the approximation of the
Poisson probability function by the normal

I1IAk;.
v)- -e- -
k!
1 {I
- - exp --(k - A)
foi 2A.
2}1 -+ O.
4.4.* If Xn.!. c and f is continuous at c, then f(Xn) .!. f(c).
4.5.* Suppose {X n, n;::: I} is a sequence of random variables with E(Xn) =
o and V(Xn) = u 2 (O) where the parameter 0 takes values in some interval.
Prove that if u;(O) -+ 0, then

for any bounded and continuous function g.


Answer: Examine the

f: Ig(x) - g(O)1 dFn(x)


4. Convergence of Random Variables 111

for Ix - 01 < b where Ig(x) - g(O)1 < e, and for Ix - 01 ~ b where Ig(x) - g(O)1 :$;
M < 00, and make use ofthe Chebyshev inequality (Fn(x) denotes the distribu-
tion function of Xn).
4.6. * (Continuation). If Fn is the "binomial" with

6;(0) = 0(1 - 0),


n
and g is defined in [0, 1], then the Bernstein polynomial

f g (~)n (n)k Ok(1 -


k=O
Or k -----:::-> g(O)
n 00
(1)

°
uniformly in :$; 0 :$; 1. So we have a simple proof of the Weierstrass Approxi-
mation Theorem, according to which such a function g can be uniformly
approximated by a polynomial. Moreover, (1) results in approximating poly-
nomials.
4.7.* (Continuation). If nXn is Poisson with parameter nO, then

e- n8 I
k=O
g (~) (n~)k ---. g(O)
n k.
(2)

and the convergence is uniform in every finite interval of values of O. Equation


(2) also holds true for nonintegral values of n.

°
4.8. Suppose XI' X 2 , ... is a sequence of independent normal random
variables with E(X;) = and V(Xi ) = 1 and let

¢n = Inn XXI ++ ...... ++ XXn ,


2 2
(= X I +"'+Xn
I n n JXf+"'+X;'
Prove that ¢n and (n ---. N(O, 1).
Hint: Xf + ... + X; !. n; use (3.4)(i).
4.9. (Continuation). Find the asymptotic (n ---. 00) distribution of

and !' = nXn + 1


'on 2'
Xn
Answer: (X; - n)/fo and ¢n are asymptotically N(O, 1).

° °
4.10. n numbers are chosen at random from the integers 1,2, ... , N, say
kl' k2' ... , kn' We set Xi = if k i = modulo 3, Xi = 1 if ki = 1 modulo 3,
and Xi = - 1 if k i = 2 modulo 3. Set Sn = I?=l Xi' Prove that when nand
N ---. 00 so that n/N ---. c > 0, Sn/Jn ~ N(O, 2/3).
4.11.* Suppose SN = Xl + X 2 + ... X N is the sum ofa random number N
of random variables Xi' where Xi and N are independent, IXil < c, E(X;) = 11,
V(X i ) = 6 2 , E(N) = n, and V(N) :$; n l -<, e > 0. Make use of Exercise 328 and
112 Supplement II. Complements and Problems

the CLT, and prove that as n -+ 00

SN - nil
r:. -+ N(O, 1).
(Jy n
4.12. Let {Xn' n ;: :.: 1} be a sequence of independent random variables
and

n = 1,2, ....

Prove that the sum


Sn = Xl + ... + Xn
(a) converges in law, and (b) find its asymptotic distribution.
Answer: (a) The characteristic function of Sn

<fJn(t) = Jl Uk).
cos

because cOS(t/2k) - 1 behaves in the same way as - t 2 /22k+1.

(b)

sin t t/2 n sin t


<fJn(t) = - t- sin(t/2 n ) ~ -t-'
i.e., <fJn(t) converges to the characteristic function of the uniform in [ -1,1].
4.13. If a sequence of normal random variable(s) Xn converges in law to
the random variable X, then X is also normal or degenerate.
Answer: The characteristic function of Xm <fJn(t) = exp(il1nt - J:(J;t 2) -+ <fJx(t)
uniformly for It I < <5, <5 > 0. So
l<fJn(t)1 2 = e-O'~t -+ I<fJx(tW,

where, if {(J;} is bounded, then (J; -+ (J2 ;:::.: 0; hence

eil'nt = <fJn(t)e(I/2)0'~t2 -+ <fJ(t)e(1/2)0'2 t2.

Therefore I1n converges, say, to 11 and finally


<fJn(t) -+ eil't-(1/2)0'2t2.

If lim (J; = (J2 = 0, then X degenerates (shrinks) to the point 11.


4.14. Prove that the negative binomial

P[X = kJ = (
r + kk - 1) prq\ k = 0,1,2, ... , (1)

with r -+
ter A..
00 and q -+ ° so that rq -+ A. > 0, converges to a Poisson with parame-
4. Convergence of Random Variables 113

Answer: The generating function of (1),

(1 - q)' ).(5-1)
-----e .
(1 - qs)' rq~).

4.15. If the stochastic sequence {Xn} converges in probability to the ran-


dom variable X, where P[X i= 0] = 1, then the sequence {1/Xn } ~ 1/X.
4.16. Consider the sequence {Xn} of independent random variables with
n = 1,2, ....
If the series La; converges, prove that the sequence of partial sums
Sn = Xl + ... + X n ,
also converges in each of the four modes of convergence (see Chapter 8).
Answer: The Chebyshev inequality for ISn - Sml and the Cauchy criterion
imply the convergence of Sn' It should also be noted that the Xn are uniformly
bounded, since an --+ 0, and then the modes of convergence are equivalent.
4.17. Let {Xd be a sequence ofrandom variable(s) with
P[Xk = ±k a ] = 1/2.
(a) Find the values of a for which

in probability, almost surely, and in quadratic mean.


(b) Examine the convergence of Xn in law for a = 1/2.
Answer: (a) E(Xk ) = 0, V(Xd = at
= k 2a ,

_ 1
2:
n
V(Xn) = 2 k 2a = O(n 2a - l ) --+ 0.
n k=l

Therefore 2a - 1 < 0, i.e., a < 1/2. This also suffices for


X- n ---+
Lm.
. °
For the Strong Law of Large Numbers (SLLN) it is enough that
n a2
k~l k~ < 00 => a < 1/2.

(b) Let <Pn(t) be the characteristic function of Xn. We have


114 Supplement II. Complements and Problems

because a = 1/2. For It I ~ b, log <Pn(t) -+ - t4 /4 uniformly. Therefore


- (.)2)
0'2 .
Xn-+ N

4.18.* By the so-called Monte Carlo method, how many trials are needed
to estimate the integral
("/2
I = J 0 sin x dx,

so that the absolute error of the esimtate will not exceed 0.1 % of I, with
probability p ~ 0.99?
Answer: The integral

-2 I = - 21"/2 sin x dx,


71: 71: 0

can be considered as the mean value of the sin x, where the random variable
X is uniform on (0, 71:/2). So an approximate value of I is

where Xn are (pseudo) random numbers in (0,71:/2). Now, using


I - I
~( ) '" N(O, 1),
(1 In
where

we find:
n ~ 1.55 x 10 6 = 1,550,000.

4.19. Suppose Vis a region ofa plane with area one andf defined in V with
If(x, y)1 ~ c. For the calculation of 1= Hvf(x, y) dx dy, by the Monte Carlo
method, we randomly choose n points (Xl' yd, (X2' Y2)' ... , (Xn' Yn) in V and
we calculate I according to the formula
1
L f(x;, yJ
n
In =-
n ;=1

Prove that
5. Miscellaneous Complements and Problems 115

5. Miscellaneous Complements and Problems


5.1. Bore/-Cantelli Lemma. In the study of sequences of events A l , A z , ...
with Pk = peAk]' a significant role is played by the Borel-Cantelli lemma:
(a) If the series LPk converges, then a finite number of the events Ak occurs
with probability 1.
(b) If the events are (completely) independent and the series L Pk diverges,
then an infinite number of the events Ak occur with probability 1.
In the study of the divergence of series of random variables the following
theorem is very helpful.
5.2. Kolmogorov's Three-Series Theorem. Suppose {Xm n ~ 1} is a sequence
of independent random variables, and for any c > 0, consider the truncated
random variables

X'
n
= {Xn
° if IXnl ~ c,
otherwise.
Then the series
00

S = L Xn
n=l

converges with probability 1 if and only if the following three series converge:
L P(IXnl > c) < 00, and L E(X~).
Otherwise, S converges with probability 0.
The following "laws" are also worth mentioning:
5.3. The Law of the Iterated Logarithm. This law concerns the frequency of
appearance of large values of the standardized number of successes,

S* = Sn - np
n fiN'
in an infinite sequence of Bernoulli trials. According to the De Moivre-Laplace
Central Limit Theorem (CL T), we have
peS: > x] "" 1 - <l>(x),
where "" means that the ratio of the two sides tends to one.
Therefore, for a particular n, large values (> 4) of S:
are improbable, but,
obviously, for large n, it is possible for S:
to exceed, sooner or later, any large
value. How soon this may occur is given by the Law of the Iterated Logarithm
(Khintchine; see, e.g., Feller, 1957). With probability 1
. sup
I1m S: 1, (1)
n-oo J2 log log n =
116 Supplement II. Complements and Problems

i.e., for A > 1, only a finite number of the events

Sn> np + ;..FnMJ210g log n (2)

occurs with probability 1, while for )0 < 1, (2) holds for infinitely many n with
probability 1.
For reasons of symmetry (of the distribution of Sn, (1) implies
S*
lim inf = - 1.
J2 log log n
n
n~OC!

The behavior of Sn is further illustrated by the following:


(a) There exists a constant c > 0 that depends on p, but not on n, so that
for all n
P[Sn > np] > c.

Hint: The probability, according to the binomial distribution, is always


positive and, according to the CL T of Laplace, it approaches 1/2.
(b) Suppose x (0:::; x < 1), with decimal expansion

(3)

where each ai is one of the digits 0, 1, ... , 9. Let ai = 0 (instead of 0 we could


have chosen any other digit) define a success with p = 1/10. Therefore, (3)
corresponds to an infinite sequence of Bernoulli trials with p = 1/10, and all
the limit theorems concerning Bernoulli trials with p = 1/10 can be translated
into theorems on decimal expansions. Sn(x), the number of zeros among the
first n decimal digits of x, takes the place of Sn. Show that
(i) Sn(x)lx --+ 0.1 in measure (Lebesgue) (in probability).
(ii) Sn(x)!x --+ 0.1 almost everywhere (with probability 1).
.... Sn - nllO
(111) hm sup I 1/2·
n~OC! (n log og n)
Answer: (i) WLLN, (ii) SLLN, (iii) Application of (1).
5.4. The Zero-One Law of Kolmogorov. Let {Xn , n ;?: I} be a sequence of
independent random variables and A an event independent of (any event
defined in terms of) Xl' ... , X k for every k. Then, either P(A) = 0 or P(A) = 1.
5.5.* The tail of the normal N(O, 1). For large x(x --+ 00), the approximation

1
1 - <1>(x) '" - o/(x) (4)
x

is valid; more precisely, for every x > 0, the double inequality

(1 1) <
o/(x) - - -
X x3
1 - <1>(x) < o/(x)- 1
X
(5)
5. Miscellaneous Complements and Problems 117

is valid. Moreover, for every constant a > 0 show that, as x -+ 00,

Hint: Verify that

-1 cp(x) = foo cp(y)(1 + y-2) dy > foo cp(y) dy = 1 - <I>(x)


x x x

and, similarily, make use of the integral of

5.6. Consider a sequence {An' n ~ I} of independent events. Then a finite


or infinite number of events Ak occur with probability 1, according to whether
the series Ln P(An) converges or diverges, respectively (see the above Borel-
Cantelli Lemma). Show this by using the Three-Series Theorem.
5.7. Let {Xn' n ~ I} be a sequence of random variables with lin = E(Xn) <
00 and X a random variable with V(X) < 00. If, for each k, Xl' ... , X k and
X - (Xl + ... + X k ) are independent, prove that V(Xk ) < 00 for every k, and
that the series

converges with probability 1 (almost everywhere or almost surely).


5.8. In a sequence of (independent) Bernoulli trials with probability of
success p, the event Ak is realized if k consecutive successes appear between
the 2kth and 2k + 1 th trial. Prove that if p ~ 1/2, then an infinite number of the
events Ak occur with probability 1, whereas for p < 1/2 with probability 1 a
finite number of the events Ak occur.
5.9.* Bochner-Khintchine Theorem. A continuous function cp(t) with cp(O) =
1 is a characteristic function if and only if it is nonnegative definite, i.e., if for
each n ~ 1 and for each n-tuple of real numbers t l , ... , tn and complex numbers
Zl' ... , Zn it satisfies the
v v
LL
j=l k=l
cp(tj - tk)ZjZk ~ O.

Prove that (*) is necessary.


Hint: The left side of(*) is equal to

where F is some distribution function. For a proof of the sufficiency of (*) see,
e.g., Gnedenko (1962).
118 Supplement II. Complements and Problems

5.10. Prove that the functions, (a) e- i1tl , (b) 1/(1 - iltl), (c) cos t 2 , are not
characteristic functions.
5.11. If tp(t) is a characteristic function, prove that g(t) = e'P(t)-1 is also a
characteristic function.
Hint: See (6.6).
5.12. Show that, for a real characteristic function tp(t), the following in-
equalities are valid:
(a)
1 - tp(nt) :s;; n 2 (1 - tp(t)), n = 0,1,2, ...
(b)
1 + tp(2t) ~ 2[tp(t)Y
5.13. Let X be the number of Bernoulli trials required until we have
observed r successive successes. Find the generating function of X and E(X).
Answer:
p't'(l - pt)
I t Lt
00 • 00 •

P(t) = .;, P[X = n] = .;, P." = 1- t + p ,qt ,+1'


1 '
E(X) = P (l)I
= ~.
p'q'

5.14.* Let M. be the maximum number of consecutive successes (the


maximum length of a run) observed in n Bernoulli trials. If
p." = P[M. :s;; r],
show that
p." = 1 - Pl., - P2" - ... - P.,r>
where P." was defined in the preceding problem, and therefore, the generating
function of P." is
00 • 1 - P(t) 1 _ p't'
"\' P t = = ~~--=--~cc-
.';:, .,' 1- t 1 - t + p'qt,+1 .

Also, show that


E(M) = log n + 0(1),
-logp
V(M) = 0(1).

5.15. Find the distributions with characteristic functions, (a) cos t, (b)
Ir;o
cos 2 t, (c) Pk cos kt where Pk ~ 0, Pk = 1. I
Answer: (a) cos t = Heit(-l) + eit(l)]. Therefore

if x < -1,
if -1 :s;; x < 1,
if x ~ 1.
5. Miscellaneous Complements and Problems 119

o if x < -2,
{ 1/4 if -2.:s; x < 0,
F(x) = ~/4 if O.:s; x < 2,
if x ~ 2.

(c) LPk cos kt = tLPke-ikt + tLPkeikt.


Therefore this is a discrete distribution with jumps tPk at the points ±k
(k = 0, 1, ... ).
5.16. Find the discrete distributions with generating functions, (a) i(1 + S)3,
(b) t(1 - tS)-l, (c) e(S-l), (d) (is + i)lOO.
Answer: (a) Discrete with probabilities 1/8, 3/8, 3/8, 1/8 at the points 0, 1,
2, 3, respectively.
(b) The probability of the value k is r k - 1 (k = 0,1,2, ... ).
(c) Poisson with A = 1.
(d) Binomial with P = 1/4 and n = 100.
5.17. Using the Kolmogorov inequality (8.9), show that if the series

L V(Xd/k2 < 00,


00

k=l

then the SLLN holds, i.e.,

5.18. A fair but loss-incurring game (a probability "paradox"). The prob-


ability that, in each play of a game, the player receives 2k dollars is

k = 1,2, ... , (1)


Pk = 2kk(k + 1)'

(average) gain in each game is


°
and that he receives dollars is Po = 1 - (PI + P2 + .. '). So, the expected

If, in each game, the player pays a I-dollar fee, the net profit ofthe player after
n games is equal to
n
L Xk - n = Sn - n, with E(Sn - n) = 0,
k=I

i.e., the game is fair (Xk is a random variable with distribution (1». For every
e > 0, however, the probability that, in n games, the player will lose more than
(1 - e)n/log2 n dollars, approaches 1, i.e., it can be proved (truncating the
120 Supplement II. Complements and Problems

random variables X k , see Feller (1957) or Gnedenko (1962» that

. [ (1 - e)n]
hm P Sn - n < 1 = 1.
n~CX) og2 n
5.19. In a sequence {Xn' n ~ l} of Bernoulli random variables, suppose
that
y" = 0 if XnXn+l = 1 or Xn = 0 and Xn + 1 = 0,
y" = 1 if Xn = 1 or X n + 1 = 1.
Find E(Zn) and V(Zn) where Zn = I?=l 1';.
Answer:
E(Zn) = nE(Y1 ) = 2npq,
V(Zn) = 2npq(1 - 2pq) + 2(n - l)pq(p ~ q)2.

5.20. We randomly choose two numbers in (0, 1). What is the probability
p that their sum be smaller than 1 and their product smaller than 3/16?
Answer: p = area S, where S = {(x, y): x ~ 0, y ~ 0, x + y ~ 1, xy ~ 3/16}.
1 3
p = 4 + 16 log 3 = 0.456.

5.21. A bus on line A arrives at a bus station every 4 minutes and a bus on
line B every 6 minutes. The time interval between an arrival of a bus for line
A and a bus for line B is uniformly distributed between 0 and 4 minutes. Find
the probability:
(a) that the first bus that arrives will be for line A;
(b) that a bus will arrive within 2 minutes (for line A or B).
Answer: Suppose x is an instant of time, where 0 ~ x ~ 12 minutes. The
times of arrival of the buses for line A are x = 0, 4, 8, and of buses for line B
the arrival times are y, y + 6 with 0 ~ y ~ 4.
(a) The favorable cases are: for 0 < y ~ 2 we have y < x ~ 4 or 6 + y ~
x ~ 12; and for y > 2 we have y < x < 8 or y + 6 < x < 12. Therefore p = 2/3.
(b) Favorable cases: 2 ~ x ~ 4, 6 ~ x ~ 8, 10 ~ x ~ 12,4 + Y ~ x ~ 6 +
y. For y < 2 we have 0 < x < y, and for y > 2 we have y - 2 ~ x ~ y. There-
fore p = 2/3.
5.22. N stars are randomly scattered, independently of each other, in a
sphere of radius R.
(a) What is the probability that the star nearest to the center is at a distance
at least r?
(b) Find the limit of the probability in (a) if
and N/R 3 -> 4nA/3.
Answer: (a) (l - r3 / R 3)N.
(b) exp{ -41!Ar 3 /3} (close to the sun A ~ 0.0063, when R is measured in
parsecs).
5. Miscellaneous Complements and Problems 121

5.23. A satellite moving on an orbit between two parallels 60° north and
60° south (latitude) is equally likely to land at any point between the two
parallels. What is the probability P that the satellite will land in the north
above 30°?
Answer:

P = {R2 I21r de 1~:3 cos <p d<P} I {2R2 I21r de I 1r13 cos <p d<P} = 0.21.
5.24. In the equation A3 + 3X A + Y = 0, the coefficients X, Yare uni-
formly distributed in the rectangle IXI s a, IYI s b. What is the probability
P that the equation has real roots?
Answer: It is required that y2 + X 3 sO, i.e., for X sO, when y2 s _X3.
If a 3 S b 2 , we have

1 fa a 312
P = 2ab 0 x 312 dx = 5b'

P= 21 - 1
2ab
fb y213 dy = 21 ( 1 -
0
3b 213 )
-a- .

5.25. * The binomial distribution via difference equations. Let Pk,n denote the
probability of k successes in n independent Bernoulli trials. By using the
generating function Gn(t) of PII • k (k = 0, 1,2, ... ), deduce the binomial distribu-
tion.
Hint: Pk,n satisfies the partial-difference equation
Pk,n = pPk-I,n-1 + qPk,n-I'
from which we can find the difference equation for Gn :
Gn(t) = (pt + q)Gn- 1(t),
which has the solution

i.e., the probability generating function of the binomial.


5.26. A and B play the following game. They flip a coin, if the outcome is
a head A gets 1 dollar from B, otherwise A pays 1 dollar to B. Initially, each
of them has 3 dollars. The game terminates when either A or B loses all his
money. What is the probability Pn that n flips are needed?
Answer: Pn > 0 only for n = 2k + 1 (k = 1, 2, ... ). Suppose qk is the prob-
ability that the game will not finish after 2k + 1 flips. Then
. _ ~(~)(n-3)12
I.e., Pn - 4 4

5.27. Let y" = max{XI' ... , Xn} where Xl' ... , Xn are independent and
identically distributed random variables with a uniform distribution on (0, 1).
122 Supplement II. Complements and Problems

Show that the distribution of


Zn = n(1 - y,,)
converges (as n --+ OC!) to the exponential distribution with distribution function
F(z) = 1 - e- z •
Answer: Examine the limit of the sequence {Fn(z)} of the distribution
functions of {Zn}'
5.28. A discrete random variable Xn that appears in the theory of extremes
has the distribution function

Fn(n) = 1 _ (n)r , 1 ::;; r ::;; n,


(n + nX)r
and Xn takes the values l/n, 2/n, .... Show that the sequence {Xn} converges
in law to a continuous distribution with distribution function
O, x::;; 0,
{
F(x) = 1 _ (1 + xrr, x> 0.
p
5.29. If Xn --+ X and E(Xn - y"f --+ 0, show that the sequence {Y,,} also
converges to X in probability.
PART III

SOLUTIONS
Solutions

The set of female students in the freshman and


sophomore years.
C' F = F - C: The set of non-Cypriot female students.
A 1 F' C = A 1 C - F: The set of Cypriot male freshmen students.
A3FC' = A3F - C: The set of non-Cypriot junior female students.
(A 1 u A 2 )CF: The set of Cypriot female students in the freshman
and sophomore years.

2. (a) (A u B)(A u C) = A u (BC).


(b) (A u B)(A' u B) = B u (AA') = B u 0 = B.
(c) (A u B)(A' u B)(A' u B') = B( A' u B') = B(AB)' = B - A (by de Mor-
gan's laws and (b».

3.
B
B

(a) The shaded area represents (b) The shaded area represents
AuBuC. (AB'C') u (A' BC') u (A' B'C)
u(A'B'C').
126 Solutions

(c) The shaded area represents (d) The shaded area represents
(A u B u C)' = A'B'C. ABC.

A B B

(e) The shaded area represents (f) The shaded area represents
(AB'C) u (A'BC) u ABC = AB - C.
(A'B'C).

(g) The shaded area represents


AuA'B'.
4. (a) A = {(HHT), (THH), (HTH)},

B = {(HHT), (THH), (HTH), (HHH)},


C = {(THH), (THT), (TTH)}.
(b) (i) A' B = {(HHH)}: exactly three heads.
(ii) A' B' = (A u B)' = B' = {(HTT), (TTT). (TTH), (THT)}: at least two
tails.
(iii) AC = {(THH)}. The first is tails, and the second and third are heads.
5. Denoting by M and F a male and a female child, respectively, we have
A = {MFMF, FMFM},
B = {MMMM, MMFM, MFMM, MFFM},
C = {MMFF, MFFM, MFMF, FFMM, FMFM, FMMF},
D = {MMMM, MMMF, FMMM, FFFF, FFFM, MFFF}.
Solutions 127

6. A = {abebea. aebeba}.
7. 44.
A = {aaaa, bbbb, ecce, dddd};

B = {aabe, aaeb, abbe, ... }, that is, G) G) G) G) = 144 points,


where a, b, e, and d denote the four types of cellular disorders, and the position
of the letter denotes the patient.
A = {ab*, ba*, a*b, b*a, *ab, *ba},
B = {ab*, a*b, ba*, b*a},
where a, b denote the two persons, * means "none", and the position of a, b, *
indicates the floor, e.g., a*b means: a at first floow and b at third floor.

9. (\0) G) G) = 100 (by the multiplication rule).

10. 4!. (3!)2(4!f (i.e., permutations of the four works times permutations of
volumes within each of the four works.
11. Identify the n persons with n cells and the r objects with r balls to be
placed in the n cells:
(i) if the objects are distinguishable there are n' ways;
(ii) if the objects are indistinguishable there are (n + ~ - 1) ways.
12. The 5 boys define consecutive gaps (positions) between any two of
them; the 5 girls can be placed in the positions in as many ways as the number
of permutations of 5 things taken all at a time, i.e., 5! = 120.
The 5 boys can be seated at a round table in as many ways as the number
of cyclical permutations of 5, i.e., (5 - I)! = 4! = 24. Therefore, the total
number of ways is equal to 4! 5! = 2,880.
13. As many chords as the number of combinations (~), i.e., 28. As many
tr!angles as the number of combinations (~), i.e., 56. As many hexagons as
(6) = 28.

14. Cyclical permutations of n, i.e., (n - 1)!. The probability that any fixed
person sits right or left of any given person is the same for every person; and
hence is equal to l/(n - 1). Therefore, the probability sought is 2/(n - 1). This
also follows from the fact that out of the (n - I)! total cases there are 2(n - 2)!
favorable cases.
15. By virtue of (1.5) the 20 recruits can be distributed into 4 camps in 20!
(5!)4 ways. However, to each division in 4 groups, there correspond 4! distribu-
128 Solutions

tions to 4 camps. Hence, there are


20!
(5!)44!
distributions into 4 groups of 5 each.
16. According to the multiplication formula, we can form

G)·G)·7!
words (every permutation of the seven letters is supposed to be a word).
17. (i) Pascal triangle. (ii) Use (i) repeatedly.
18. The n things are divided into two groups of m and n - m. Any r of the
n will contain k of the m and r - k of the n - m where k = 0, 1,2, ... , r. Hence

the expression for (~).


19. (i) The expansion of (1 - 1)" = O.
(ii) Differentiate (1 + x)" once and put x = 1.
(iii) Differentiate (1 + x)" twice and put x = 1.
20. (i) Expand the right-hand side. (ii) Use 17(i).

21.

25. Examine the inequality ak+tiak ~ 1. It gives k ::; (nx - l)/(x + 1) and
the required maximum value of ak is the ak*+1 with

k*=[~J
x+1 '
where [a] denotes the integer part of a.lf(nx - l)/(x + 1) is an integer, r say,
then there are two maxima, namely, the ar and ar + 1 • On the other hand, if
x < lin then ak+1 < ak for every k and max ak = ao = 1. For e = 2.72, n = 100
we have

k*=[lOOe-lJ.
e+ 1
26. peA') = 1 - peA) = 1 - t = i,
peA' u B) = peA') + PCB) - peA' B)
= 1- peA) + PCB) - {PCB) - P(AB)}
= 1- peA) + P(AB) = 1 - t + i = i,
Solutions 129

peA u B') = peA) + PCB') - P(AB')


= peA) + 1 - PCB) - {peA) - P(AB)}

= 1 - PCB) + P(AB) = 1 - t+i = g,


P(A'B') = P[(A u B)'] = 1 - peA u B)

= 1 - peA) - PCB) + P(AB) = 1- t - t +i = 172'

peA' u B') = P[(AB)'] = 1 - P(AB) = 1 - i = i.


27. From A ~ A u B, B ~ A u B, we conclude that peA) ~ peA u B),
PCB) ~ peA u B). Hence

peA u B) ~ max{P(A),P(B)}, (1)

and as peA) = 3/4 > PCB) = 3/8 it follows that peA u B) ~ 3/4. Because of
A ;2 AB, B ;2 AB we have P(AB) ~ peA), P(AB) ~ PCB) and so

P(AB) ~ min(P(A), PCB)). (2)

Consequently, P(AB) ~ 3/8.


By the addition theorem we have P(AB) + peA u B) = peA) + PCB) and,
because of peA u B) ~ 1, P(AB) ~ peA) + PCB) - 1, but P(AB) ~ and
hence
°
P(AB) ~ max{O, peA) + PCB) - I}.
(3)
P(AB) ~ i + i-I = l
From (1), (2), and (3) for peA) = 1/3, PCB) = 1/4 we have the inequalities
peA u B) ~ t and °~ P(AB) ~ l
28. We have P(A'B) = PCB) - P(AB) and peA) = 1 - peA'). Hence
P(AB) - P(A)P(B) = P(AB) - [1 - P(A')]P(B)

= P(A')P(B) - [PCB) - P(AB)]

= P(A')P(B) - P(A'B). (1)

P(AB) - P(A)P(B) = P(AB) - P(A)[1 - PCB')]

= P(A)P(B') - [peA) - P(AB)]

= P(A)P(B') - P(AB'). (2)


The required relation follows from (1) and (2).
29. By the addition theorem, we have
P(A 1 A 2 ) = peAl) + P(A 2 ) - peAl u A 2 ) ~ peAl) + P(A 2 ) - 1. (1)
That is, for n = 2 the inequality holds. Suppose that it holds for n = k, i.e.,
k
P(A 1 A 2 ••• Ad ~ L P(A;), -
i~l
(k - 1).
130 Solutions

It is sufficient to show that it also holds for n = k + 1. Using (1) and (2) we get
P(A 1A 2 ···Ak+1) = P[(A 1A 2 · •• A k)Ak+1]:::;; P(A 1A 2 • .. A k) + P(Ak+d-l
k k+l
~ L: P(Ad -
i=l
(k - 1) + P(Ak+l) - 1= L:
i=l
P(AJ - k.

30. We have
PEA u B u C] = PEA u (B u C)] and by the addition theorem
= P(A) + P(B u C) - P[A(B u C)]

+ P(B u C) - P[(AB) u (AC)]


= P(A)

= P(A) + P(B) + P(C) - P(BC) - P(AB) - P(AC)

+ P(ABC).
PEat least one] = P(A u B u C) = 0.5 + 0.4 + 0.3 - 0.35 - 0.25 - 0.2 +
0.15 = 0.55, that is, 55%.
31. (a) Let Pn(k) denote the probability that the integer n is divisible by k.
Then Pn(k) = (l/n) [n/k] where [x] denotes the integer part of x. Hence the
required probability equals

(b) For n = 100 we get from (*)

100 3
1 ([100J 4 - [100J)
+ [100J 12 = 100
1 (33 + 25 - 8) = 2:'
1

(c) Taking the limits as n --+ 00 in (*) we obtain


1 1 1 1
3 + 4 -12 = 2:'
32. (i) Let Pk = P[exactly k of the events A, B, C occur] (k = 0, 1,2,3), then
Po = P[A'B'C'] = P[(A u B u C)'] = 1 - P(A u B u C) = 1 - P(A) - P(B)

- P(C) + P(AB) + P(AC) + P(BC) - P(ABC),

P1 = P[AB'C' u A'BC' u A'B'C] = P[A(B u C)'] + P[B(C u A)']

+ P[C(A u B)']

= P(A) - P[AB u AC] + P(B) - P(BC u BA) + P(C) - P[CA u CB]


= P(A) - P(AB) - P(AC) + P(ABC) + P(B) - P(BC) - P(BA)

+ P(ABC) + P(C) - P(CA) - P(CB) + P(ABC)


= [P(A) + P(B) + P(C)] - 2[P(AB) + P(AC) + P(BC)] + 3P(ABC),
Solutions 131

P2 = P[ABC u = P(ABC) + P(ACB') + P[BCA']


AB'C u A' BC]
= P(AB) - P(ABC) + P(AC) - P(ABC) + P(BC) - P(ABC)
= [P(AB) + P(AC) + P(BC)] - 3P(ABC),
P3 = P(ABC).

(ii) PEat least k of the events A, B, C occur] = L~=kPn the Pi as in (i); see
also Exercise 30.
33. By the addition theorem, for n = 2, (1.1) holds. Suppose that it holds
for n = r. We shall show that it also holds for n = r + 1. By the addition
theorem we have

By hypothesis for n = r it follows that

p[~ (A iA'+1)] = ktl (_I)k- lS:


where S: = Ll~il
P(Ai,Ai2···AikA'+I)·
<i2< ... <ik5:r

Let
Sr =
1 ~il
L
< ... <i k 5:r+l
P(Ai, Ai2 ··· Ai)' k= 1, ... ,r+ 1. (2)

Then
, ,+1
SI + P(A,+d = L P(A i ) + P(A,+d = i=1
i=1
L P(A;) = st*, (3)

and, in general,

Sk + S:_I = L
1 ~il < .-. <i k 5:r
P(A 1 A 2 ••• A iJ

+ 15:il < ...L<i P(Ai,Ai2···Aik_,A'+1)


k - I $r

= S:*, k = 2, ... , r, (4)


(5)

By virtue of (2)-( 5), (1) becomes


132 Solutions

r
= SI + P(Ar+t> + L (_l)k-l [Sk + S:-I] + (-I)'S:
k=2
r+l
= L (_I)k- 1S:*,
k=1

i.e., (1) holds for n = r + 1.


34. For any two events A and B we have:
(i) d(A, B) = P(A t::,. B) = P[AB' u BA'] ~ 0 and d(A, B) = 0 if A = B, that
is, d is a nonnegative function.
(ii) d(A, B) = P(A t::,. B) = P(B t::,. A) = d(B, A) (symmetric property).
To complete the proof we have to show that for any events A, B, and C the
triangle inequality holds, i.e.,
(iii) d(A, C) ::;; d(A, B) + d(B, C),
d(A, B) + d(B, C) - d(A, C) = P(AB' u BA') + P(BC' u CB')
- P(AC' u CA')
= P(AB') + P(BA') + P(BC') + P(CB')
- P(AC') - P(CA')
= P(AB'C u AB'C') + P(A'BC u A'BC')
+ P(ABC' u A'BC') + P(AB'C u A'B'C)
- P(ABC' u AB'C') - P(A'BC u A'B'C)
= P(AB'C) + P(AB'C') + P(A'BC) + P(A'BC')
+ P(ABC') + P(A'BC') + P(AB'C)
+ P(A'B'C) - P(ABC') - P(AB'C')
- P(A'BC) - P(A'B'C)
= 2[P(AB'C) + P(A'BC')]
~O.

35. (a) The probability Po of no lucky ticket is

and the probability of winning is 1 - Po.


(b) The probability of winning is
Solutions 133

37. Let Xj denote the number of persons born on the jth day of the week
(among the 7 persons). Then (Xl' ... , X 7) has the multinomial distribution
7
L
j=l
nj = 7.

(a) P[Xj = 1, j = 1,2, ... , 7J = 7! (1/7)7.


(b) peat least two on the same dayJ = 1 - P[Xj = 1, j = 1,2, ... , 7J =
1 - 7! (lj7r.

7 7 .
7' 3! (1)4(5)3
(c) P[two on Sunday and two on TuesdayJ = 2! 2;

38.

39. For real roots we must have


b2
ac<-.
- 4 (1)

From the values of the product ac we observe that inequality (1) holds as
follows:

Value of b: 234 5 6
Favorable cases: 3 8 14 17

Thus we have 43 favorable cases out of a total of 6 3 = 216. Hence the


probability of real roots is 43/216 and of complex roots is 173/216.
4
40. (a) C52) (a favorable case for each of the 4 suits).

(b) 13 x 48 (to each choice of 4 cards of the same face value (ace, 2,

C52) 3, etc.) there correspond 48 ways of choosing the fifth


card).
5 X 45
(there are 5 quintuplets with successive face values, e.g.,
(c) C52) (2, 3, 4, 5, 6), (3, 4, 5, 6, 7), etc., and each of the 5 cards
can be chosen in 4 ways corresponding to the 4 suits).
134 Solutions

41. (i) Multiple hypergeometric:

C~)C:)C:)(13 - VI 1~ V 2 - vJ IGD·
(ii) (~)(134~ V) G~)' I V= 0,1,2,3,4.

(iii) (~)(~)(13 ~ v~ ~ vJ G~)' I


(a) Let the event Aj be the jth player who gets an ace (j = 1,2,3,4). Then
4! 48!/(12!)4
P[AIA2 A 3 A 4] = 52!/(13!)4
(b) Let Bj be the event when the jth player gets all the aces (j = 1,2,3,4).
Then

Pea player gets all the aces] = 4P[Bj ] = 4 (:8) IG~)'


(c) Pea player gets VI aces and his partner Vaces] 2

= 2 X
(VI4)( 13 48- VI )(4-VI)(35+VI)
V 13 - V 2 2

G~)G~)
(i.e., of the two hands one has VI
aces and the other has Vaces), if VI -:f.
2 V2 ,
VI V 2
when = 2 the factor above should be deleted.
42. (a) The ten boys define ten successive intervals (arcs). The six girls can

choose six of the intervals in C60) ways. The event A that a certain boy
remains between two girls can be realized in (!) ways. Hence

(b) The six girls can enter in (10)6 ways. The event B that certain girl stands
by certain boy can be realized in 2 x (9)5 ways. Hence

P[ ] = 2 (9)5 = !
(10)6 5'
X
B
This is immediate since the girl can choose 2 favorable positions out of the
10. We cannot conclude that her choice is not random.
Solutions 135

43. (a) (i) Spq25 with p = 1 - q = 1/26.


(ii) (3/1W.
(iii) 1/26 5 •

(b) (i) C:) Ie:) = ;6·

(ii) (D ICs6).
(iii) 1/(26k
44. Let us represent the r balls by r stars, and the n cells by n spaces between
n + 1 bars. Each distribution starts and ends with a bar and between these
extreme bars there are r stars and n - 1 bars. To each selection, either of the
r places for the stars or of the n - 1 places for the bars, there corresponds a
distribution. Consequently, there are

such distributions (Feller, 19S7, p. 36). An alternative proof is by induction on


n as follows: Suppose (*) holds for n = k, i.e., the number of solutions Xi ~ 0,
Xi integers, of the equation

is given by
k+r -
s(k, r) = ( k _ 1
1) .

Then clearly the solutions of


Xl + ... + Xk+l = r,
or equivalently of

are the solutions of


XI + ... + Xk = r - i, i = 0, 1, ... , r,
that is,
s(k + 1, r) = s(k, r) + s(k, r - 1) + ... + s(k, 0)
= (k +r - 1) + (k+ r
k-l
- 2) + ... + (k - 1) ,
k-l
which, by Exercise 17(ii), gives

s(k + 1, r) = (k +
k
r) '
i.e., (*) holds for n = k + 1.
136 Solutions

45. The number of distributions with exactly m empty cells is (:).

( n -r -m-1
1 ). Indeed, if we represent the r balls with r stars and the n cells

by spaces between n + 1 bars, then we can select m cells (empty cells) from the

n in (:) ways. Without any loss of generality, let the empty cells be the first
m, that is, the spaces between the first m + 1 bars. The r stars leave r - 1
spaces of which n - m - 1 are to be occupied by bars and hence we have

( r - 1 ) choices. Using the result of Exercise 44, we conclude that the


n - m-l
required probability equals

46. (a) There are (52). ordered samples of n cards and of these 4· (48).-1
have an ace as the nth card. Thus the required probability is

( 52 - n)
P(A.) =
4·(48).-1
(52). = C:) ,
3
n = 1, 2, ... , 49.

(b) Let E. the event that the first ace will appear after the nth card. This is
equivalent to the event that the first n cards will contain no ace

n = 0, 1, ... ,48.

Another proof Let An the event in (a). Then E. = An+1 u A.+2 U··· A49 and,
as the Aj are mutually exclusive,

where for the sum

i=~1
49 (52 -
3
i)
the result of Problem 17(ii) was used.
Solutions 137

47. (a) Ignoring leap years, there are 365 v ways of distributing v persons in
365 days. The event Av that no two persons have the same birthday can be
realized in (365)v ways ( = 0 for v > 365). Hence

[ A ] = (365)v = nV (1 _ k - 1) ~ 1 _ v(v - 1)
P v 365 v k=l 365 730 '
where the approximation ~ holds for small v. Thus
v(v - 1)
P[A~] = 1 - P[Av] ~ 730 .

(b) Solving the inequality P[A~] ;? 1/2 gives v = 23 with P[A;3] = 0.507.
For v = 60, it turns out that P[A~o] = 0.994, that is, in a class of60 it is almost
certain that at least two have a common birthday.
48. Let Ak be the event that the kth letter is placed in the kth envelope
(k = 1,2, ... , N). Then
p = P[each letter is placed in a wrong envelope]
= 1 - P[at least one letter is placed in the right envelope]

= I-P[U AkJ.
k=l

By Poincare's theorem we have

p[ UAkJ = Sl -
k=l
S2 + ... + (_l)N- 1 SN' where

Sk = L
1 Sit < ... <ikSN
P[A i1 ... AiJ.

But
[ ] (N - I)!
P Ak =-N"!'
(N - 2)!
P[AiAj] = N.
"
(N - 3)!
P(AiAjAd = N! '

so that, in general,

_ (N)(N - k)! _~.


Sk - k N! - k!'

the last term is


138 Solutions

and so

P=I-P[U Ak]=I-[I-~+~- ... +(-lt-l~]= f (_I)k~.


k~l 21 31 N. k~2 k.

This probability, even for moderate values of N (N > 4), approaches 1 - e- 1 =


0.632.
Note: The problem of at least one match (here in terms of envelops and
letters) is referred to as the problem of rencontres.
49. (a) Let Ak be the event that the number k does not appear (k = 1,
2, ... , n). The probability Pm that exactly m among the n events AI' ... , An
occur simultaneously is given by

Pm = Sm - (mm+ 1) Sm+l + (m m+ 2) Sm+2 - ... + (_I)n- m(n)


m Sn'

where Sk is defined as in Problem 48* and is here given by

The required probability is given by


p! = Pn-m·
(b) P[each of the numbers 1,2, ... , n will appear at least once]
= 1 - Pn[at least one of the numbers 1,2, ... , n will not appear]
= 1- p[ UAk]
k~l
= 1- ±
k~l
(-I)k- 1 Sk = 1_ ±
k~l
(_l)k-l(n)([n - k]r)N
k (nr)N

= ±
k~O
(_l)k(n)([n - k]r)N.
k (nr)N

50. P[m balls will be needed] = (;)p[n - 1 among the numbers 1,2,
... , n will appear at least once after m - 1 trials, and at the mth trial the nth
number will appear]

= G) [ :t:1- (_I)k-l] Sk nr - ~+1


= (n)[nf t
1 k~O
(_I (n - 1)[([n - k - l]r)m-l].
k (nr)m-l
r
nr - m +1
= ±
k~l
(_I)k-l (n
k
=11) ([n(nr-_k]r)m-l
1)m-l
.
Solutions 139

51. (a) Let Ai be the event that the ith man takes the ith coat and the ith
hat. Then
P = P[no one takes both his coat and hat]

N
= 1- L (_l)k- 1 Sk'
k=l

where
Sk = L P[A i ,··· AiJ (Poincare's theorem), k = 1, ... , 2.
lSi 1 <"'<i k s,N

But

PEAk] = [
(N - 1)!J2
N! ' Sl = LN
P(Ad =N
[(N - 1)!J2 = (N - 1)! ,
k=l N! N! 1!

P(A.A.) = [
(N - 2)!J2
N! ' S2 = L P(AiAj) = (N)[(N - 2)!J2
I J l";i<j";N 2 N!
(N - 2)!
N!2! '
and in general
(N - k)!J2 S = (N)[(N - k)!J2 = (N - k)!.
P[Ai, A i2 ··· AiJ = [ N! ' k k N! N! k! '
the last term is

Therefore

P
= 1- f
k=l
(-1t- 1 (N - k)! =
N! k!
f (-1t(NN!- k!k)! .
k=O
(b) P[each man takes the wrong coat and the wrong hat]
= P[each takes a wrong coat]· P[each takes a wrong hat]

= [t
k-2
(-1)kk\J
. k-2
[t
(-1)kk1,J
.
= [t
k-2
(-1tk\J2 (see Exercise 48).
.
52. (a) Let Pk be the probability that the housewife gets k free packets. Then
(1)
with Si (i = 1,2, 3,4), defined as in Problem 48*, and let Ai be the event
140 Solutions

that the ith letter of the word TIDE does not appear, (i = 1,2,3,4). We
have

Substituting into (1) we get

On the other hand,

P2 = (~~4(~y (multinomial distribution). (2)

53. Let A be the event that both the selected balls are white. We
have:

(1)

and because of
N1 N 1 - l --,--
> -,---=-__ for N2 > 0,
Nl + N2 Nl + N2 - 1
equation (1) gives the inequalities

(
Nl
Nl
+ N2
)2 > "21 > ( Nl
Nl - 1
+ N2 - 1
)2'
the first of which gives
1
Nl > j2 (Nl + N2), that is, Nl > (j2 + I)N2'
and the second

Hence, it follows that


(1 + j2)N2 < Nl < (1 + j2)N2 + 1. (2)
For N2 = 1 we get 2.41 < Nl < 3.14, that is, Nl = 3. Then we observe that

P(A)=~'~=~
4 3 2
and the required minimum value of Nl is 3.
Solutions 141

(b) If N2 is even, then, from (1) and (2), we have

Nl between possible Nl P(A)


2 4.8 and 5.8 5 10/21
4 9.7 and 10.7 10 45/91
6 14.5 and 15.5 15 1/2

Thus the minimum value of Nl is 15.


(c) The minimum value of N corresponds to the minimum value of Nl since
Nl is an increasing function of N 2 • Thus in (a) N = 3 + 1 = 4, and in (b)
N = 15 + 6 = 21.
54. Since A and B are independent we have
P(AB) = P(A)P(B),
and hence
P(AB') = P(A) - P(AB) = P(A) - P(A)P(B) = P(A)[1 - P(B)]
(1)
= P(A)P(B'),
which shows that A and B' are independent. Similarly, it is shown that
P(A' B) = P(A')P(B), (2)
P(A' B') = P(A u B)' = 1 - P(A u B) = 1 - P(A) - P(B) + P(AB)
= 1 - P(A) - P(B) + P(A)P(B) = [1 - P(A)] [1 - P(B)]
= P(A')P(B').

55. Since the events A, B, and C are completely independent, the following
relations hold:
P(AB) = P(A)' P(B), (1)
P(BC) = P(B)P(C), (2)
P(CA) = P(C)P(A), (3)
P(ABC) = P(A)' P(B)P(C). (4)
From (2) and (4) we obtain
P(AA*) = P(ABC) = P(A)'P(BC) = P(A)'P(A*),
which shows that the events A and A* = BC are independent. The proof of
the remainder is similar.
56. x = P(A'B'C) = P(A u B)'C = P(C) - P(A u B)C = P(C)
- P(AC u BC)
= P(C) - P(AC) - P(BC) + P(ABC)
= P(C) - P(A)P(C) - P(B)P(C) + P(A)P(B)P(C).
142 Solutions

Letting P(A) = a we obtain


x = (1 - a)(1 - P(B»P(C), (I)
b = I - P(A u B u C) = P(A'B'C') = P(A')P(B')P(C')
= (I - P(A»(I - P(B»(I - P(C».
b = (1 - a)(1 - P(B»(I - P(C», (2)
c = I - P[ABC] = I - P(A)P(B)P(C) = I - aP(B)P(C). (3)
From (1) and (2) we get
x P(C)
- ::;. P[C] = ~b. (4)
b 1 - P(C) x+
Substituting (4) into (3) we obtain

P(B) = (I - c)(x + b). (5)


ax
Combining (I), (4), and (5) we get

x = (1 _ a) (I _ (I - c)(x + b»). _x_ = I - a . ax - (1 - c)(x + b),


ax x+b x+b a
ax(x + b) = (1 - a)ax - (I - a)(1 - c)(x + b),
ax 2 + abx = (1 - a)ax - (I - a)(1 - c)x - (I - a)(1 - c)b,
ax 2 + ab - (I - a)(a +c- I)x + (I - a)(1 - c)b = O. (6)
As x represents probability, both roots of (6) must be positive and therefore
their sum as well, i.e.,
ab (1 - a)2 + ab
(1 - a)(a +c- I) - ab > 0, a-I+c>--
I - a'
c>--,----
I-a
57. Associate the events Ai with independent Bernoulli trials; hence use the
binomial distribution.
58. Let Ai be the event that an ace appears at the ith throw and let Bi be
the event that a six appears at the ith throw. Then the required probability
equals

_(~Y _(4)2 _
-GY - 5 -
16
25·

59. (a) We have


P[X2 > 0] = I - P[X2 = 0].
Solutions· 143

By the theorem of total probability


P[X2 = 0] = P[Xl = 0]P[X2 = 0IX 1 = 0] + P[Xl = I]P[X 2 = 0IX 1 = 1]
1 1 1 1 1 25
+ P[Xl = 2]P[X2 = 0IX 1 = 2] =:r 1 + 2'4 + 4· 16 = 64'
and hence

(b) Using Bayes's formula, the required probability equals


P[Xl = 21X2 = 1]
= 2]P[X2 = IIX 1 = 2]
P[Xl
P[X 1 = I]P[X2 = IIX 1 = 1] + P[Xl = 2]P[X2 = IIX 1 = 2]

1 1
4 4
-
1 1 1 1 5
2'2 + 4·4
60. Let Am denote the event Xmax:::; m (m = 1,2, ... , n). Then
P[An] = P[Xl :::; m, X 2 :::; m, ... , X,:::; m]

m'
= P[Xl :::; m]···P[X,:::; m] = ,.
n
Clearly, Am- 1 Am and the event [Xmax = m] = Am - Am- 1 . Consequently,
(a) P[Xmax = m] = P[Am] - P[Am-1Am] = P[Am] - P[A m- i ]
m' - (m - 1)'
n'

m = r, r + 1, ... , n.

61. The number X of successes (wins) ofa team in n games obeys a binomial
distribution with probability p = 1/2 for success, that is,

P[X = k] = G)Gr k = 0, 1, 2, ... , n.

(a) P[the series will end in at most 6 games]


= P[in exactly 4 games] + P[in exactly 5 games]
+ P[in exactly 6 games].
144 Solutions

Now we have
P[in exactly 4 games] = PEA wins the 4 games] + P[B wins the 4 games]

P[in exactly 5 games] = 2· PEA wins 3of the first 4 games and also wins
the 5th (last) game] G) GY =~.
= 2·

P[in exactly 6 games] = 2·G)GY = 156'


Hence,
.
[ h senes
Pte Wl
'11 end'In at most 6 ]
games ="81 + 41 + 5
16
11
= 16'
(b) The required probability p equals
p = P[B will win the games 3 to 6]
+ P [A will win 2 of the games 3 to 5 and the sixth game]
=(t)4 + 3(t)4 =t.
62. Let Ai be the event when the patient has illness Ai (i = 1,2,3), and let
B be the event when the result of the test is positive twice. By hypothesis, the
a priori probabilities are
1 1
P(Ad = 2' P(A 2 ) = 4'
After the three independent repetitions of the test the probabilities that the
result will be positive are

P[BIA 1 ] = (23)(1)23 9
4 4 64' =

P[BIA 2 ] = (D GY =~,
From Bayes's formula we get the required probabilities
2 9
4 64 18 18
PEA liB] = -2-9--1-24--1-2-7 18 + 24 + 27 69'
4"64 +4'64 +4"64
24
P[A 2 IB] = 68'
Solutions 145

63. Let G be the event that a person is Greek, let T be the event that a
person is Turkish, and let E be the event that a person speaks English. Then
from Bayes's formula we have

P GE _ P[GJP[EIGJ 0.75 x 0.20


[ I J- P[GJP[EIGJ + P[TJP[EI TJ 0.75 x 0.20 + 0.25 x 0.10
= 0.857,
that is, of the English speaking population of Nicosia, 85.7% are Greeks.
64. Let Ai be the event that A forgets his umbrella in the ith shop, let Bi
be the event that B forgets his umbrella in the ith shop, and let Bo be the event
that B has left his umbrella at home. Then:
(a) P(they have both umbrellas) = P(A'lA~A~)[P(Bo) + P(BbB~B;B3)]

27 64 + 27 2457
64 2·64 8192
(b) P(they have only one umbrella)
+ P(BbB~B2) + P(BbB~B;B3)]
+ + +
= P(A'lA~A~)[P(BbBd

+ [P(Ad P(A'lA 2) P(A'lA~)][P(Bo) P(BbB~B;B3)J

= (~)3[~~~~~
4
+ ~(~)2 ~J
24244 2 4 4

+[~+~~+(~y ~lG+~GYJ
999 6279 7278
= 8192 + 8192 = 8192
(c) P(B lost his umbrellalthey have only one umbrella)
P(they have one umbrella and B lost his umbrella)
P(they have only one umbrella)

999
8192 999
7278 7218
8192
65. The condition for A and B to be independent is
P(AB) = P(A)P(B).
146 Solutions

We have
P(A) = 1 - P(A') = 1 - [P(all boys) + P(all girls)]
1 1) 2 - - 1 n 1
= 1 - ( 2n + 2n = -2-n--'-1- ,

P(B) = P(all boys) + P(one girl) = 2" (l)n + n (1)


2" (l)n-l
2" n+1
= ~'

.I n
P(AB) = P(one gu) = 2n '

and by condition (*) we must have


n n + 1 2n - 1 - 1
-- => 2n - 1 = n + 1 => n = 3.
2n 2n 2n - 1

66. Let A be the event that a really able candidate passes the test and let
B be the event that any candidate passes the test. Then we have

P[BIA] = 0.8, P[BIA'] = 0.25, P(A) = 0.4, P(A') = 0.6,

and by Bayes's formula

P[AIB] _ P(A)P(BIA) 0.32 32


- P(A)P(BIA) + P(A')P(BIA') 0.32 + 0.15 47'
that is, about 68%.

L
00

67. PEA] = PEA wins at the k + 1 throw of the dice]


k=O

where

PI = PEA wins in a throw of the dice] = :6'

P2 = P[B wins in a throw of the dice] = 366'

consequently,
5
36 30
P(A) = -----:3---:-1----:3=-=-0
61
1 - 36· 36

68. Let Ai be the event that player Pi wins (i = 1,2,3), let Bj be the event
Solutions 147

that a non-ace appears at thejth throw, and let P(AJ = ai (i = 1,2,3). Then
and

because if PI does not win on the first throw then Pz plays "first" and hence
P(AzIB I ) = P(Ad = a l • Similarly,

25
a 3 = P(A3) = P(A 3B I B z ) = P(BIB2)P(A3IBIBz) = 36al'

and, since a l + az + a3 = 1, we get


36 30
a -- a --
I - 91' z - 91'
69. By an argument similar to Exercise 68 we get
k = 1,2, ... , N, q = 1- p.

and by the condition I,~=I Pk = 1


pqk-I
Pk = -1-- ' k = 1,2, ... , N.
-q N
70. Let W be the event that finally a white ball is selected from A, and WI
be the event that a white ball is selected from A, and let Wz be the event
that a white ball is selected from B. Then by the theorem of total probability
we get

peW] = P[WI WI Wz ]· P[WI W2] + P[WI WI W;JP[WI W;J


+ P[WIW;W2]P[W;W2] + P[WIW;W;JP[W;W;J,
where setting WI + hi = Ni (i = 1,2), we have

, WI - 1
P[WI WI W2] = - - ,
NI
WI - 1
P[WI WI W2] = - - ,
NI

P[WI w.'W'] =
I 2
~
N ' I

71. (a) Let A be the event that a person has the disease, and let Bi be the
148 Solutions

event that the ith test is positive (i = 1, 2). Then


\

PEA] = 0.1, P[B;lA] = 0.9, P[BiIA'] = 1 - P[B;IA'] = 0.1.

P[BIA] = P[B 1 IA]P[B2 IA] = 0.81, P[BIA'] = 0.Q1,


P[A]· P[BIA] 81
P[AIB] = P[A]. P[BIA] + P[A']· P[BIA'] = 81 + 9 = 0.9.
(b) If C denotes the event that only one test is positive then

P[ClA] = G)P[BdA]. P[B;IA] = 0.18,

P[ClA'] = G)P[B1IA'].P[BIIA'] = 0.18.


Thus we find
PAC = P[A]P[ClA] = 0.1.
[ I] P[A]· P[ClA] + P[A']P[ClA']
72. (a) Let A be the event that coin A is selected, let B be the event that coin
B is selected, and let Hk be the event that heads appears k times (k = 0,1,2).
Then by the theorem of total probability

We have

PEA] = P[B] =~, P[HkIA] = G)(~YGy-k,


P[HkIB] = G) Gy(~y-k,
and therefore

(i)

(ii)

(b) P[heads at least once in strategy (a)] = P[H 1 ] + P[H2 ]


11
16'

P[heads at least once in strategy (b)] = G) GY + (D GY


3

Solutions 149

Hence he must follow the second strategy ifhe wants to maximize the probabi-
lity of at least one head.
73. Let A be the event that a white ball is selected from A, let B1 be the
event that two white balls are selected from B, let B2 be the event that one
white ball and one black ball are selected from B, and let B3 be the event that
two black balls are selected from B. Then by the theorem of total probability
we get:

P[BI] =
G)
(I})' P[B 2 ]
G)(~)
= -(122) , P[B3] =
G)
e22)'
8 7 6
P[AIBI ] = j)' peA IB2 ] = 13' P[AI B3] = 13·

Hence
136
peA] = ---.
429
(b) The required probability p say, is given by

Applying Bayes's formula we get

PCB IA] = P[BI]P[AIBIJ = ~ P[B 2 ]P[AIB 2 ] 7


PCB IA] =------------- = -.
1 peA] 34' 2 peA] 34
Hence
13
P= ---.
34
74. Let the event A be that the test is positive and the event B that a woman
has the disease. Then
I 19
--_._-
P(B)P(AIB) 2000 20 19
P[BI A] = --------------------------
P(B)P(AIB) + P(B')P(AIB') 1 19 1999 1 2018'
2000· 20 + 2000·20
that is, less than 1'X. of the women with positive test have the illness. The lady's
fear is rather unjustified.
75. Let A, B, C be the events that the clerk follows routes A, B, C,
respectively, let L be the event that the clerk arrives late, and let S be the event
150 Solutions

that the day is sunny. Then


P[CSL] P[S]P[CLIS]
(a) P[ClSL] = P[SL] = P[S]P[LIS]
P[ClS]P[LICS]
P[AIS]P[LIAS] + P[BIS]P[LIBS] + P[ClS]P[LICs]
Since

P[AIS] = P[BIS] = P[ClS] =~, P[LIAS] = 0.05,

P[LIBS] = 0.10, P[LICS] = 0.15,


we get P[ClSL] = 0.5.
, P[S']P[LIS]
(b) P[S IL] = P[S]P[LIS] + P[S']P[LIS'] ,

P[LIS] = P[AIS]P[LIAS] + P[BIS]P[LIBS]


1 30
+ P[ClS]P[LICS] = 3 100'
P[LIS'] = P[AIS']P[LIAS'] + P[LIBS'] + P[ClS']P[LICS']
1( 6 15 20 ) 1 41
= 3 100 + 100 + 100 = 3" 100'
3 , 1
peS) = 4' P(S) = 4·

Consequently,
41
P[S'IL] =131.
76. Let A be the event that the painting is original, and let B be the event
that the expert says it is an original. Then according to Bayes's formula
5 9
P[A]P[BIA] 6 10 45
(a)P[AIB] = P[A]P[BIA] + P[A']P[BIA'] 5 9 1 1 46
«( 10 +"6·10
(b) Let A be the event that the second choice is original, and let H be the
event that the expert decides correctly that it is an original. Then by the total
probability theorem we have

P[A*] = P[A*IH]P[H] + P[A*IH']P[H'] = ~.1O + ~.~ =~.


10 11 10 11 110
Solutions 151

77. Let A be the event that the 5 cards are clubs, and let B be the event
that the 5 cards are black. Then

C:) IC5 C:) 2

C56) IC52) C56)'


P[AB] )
P[AIB] = PCB] = =

78. Let A 2 denote 2 boys, let B2 denote 2 girls, and let Cn denote n children
in the family. Then

(a) P[C 2 IA 2 ] = :[C 2 ]P[A 2 IC2] •

L P[Cn ]P[A 2 ICn ]


n=2
00

peA B ] ~ P[Cn ]P[A 2 B2 ICn ]


(b) P[B 2 IA 2 ] = 2 2 = _n-_:______
P[A 2 ] L P[Cn ]P[A 2 ICn ]
n=2

(l)n-l
and since

Pn = P[Cn] = (1 - 2a)"2 ' n = 2, 3, ... ,


and

we get

P[A 2 ] = n~2 P[C n ]· P[A 2 ICn ] = (1 - 2a) n~2 G) Gyn-l


_ (1 - 2a) ~ _ (~)n-2 _ (1 - 2a) _ 8(1 - 2a)
1)3 -
2
- 42 n4:2 n(n 1) 4 - 42 ( 27 '
1--
4

L
00

P[A 2 B 2 ] = P[Cn ]· P[A 2 B 2 ICn ] = P[C4 ]· P[A 2 B2 IC4 ]


n::::::4

6(1 - 2a) 3(1 - 2a)


128 64
Finally,
(1 - 2a)
8 27
P[C 2 IA 2 ] = 8(1 - 2a) = 64'
27
152 Solutions

3(1 - 2a)
64 81
P[B2IA2J = 8(1 - 2a) S12·
27
79. (a)

P[A3I B3J = P[A3J· P[B3I A 3J


P[A l JP[B3 IA l J + P[A 2JP[B3IA 2J + P[A3JP[B3I A 3J
6
11
(b)

P[A2I B3J = ;[A 2JP[B3IA 2J 0.2 x 0.1 2


0.11 11
I
i=l
P[A i JP[B3IAJ

(c)

P[A 2IB l uB2 J = ;[A 2J·P[B l uB21A 2J ,


I
i=l
P[AJ· P[B l u B21 AiJ

whereby the addition theorem for conditional probabilities

and the conditional probabilities are given in the table.


80.
(Ia) Pk = G)C70YC30Y-k, k = 0,1, ... ,S (binomial).

(Ib) Pk =
C)(S ~ k)
CSO) , k = 2, 3, 4, S (hypergeometric).

(10 - k)S
(II a) P[Xmin > kJ = lOs '

P[Xmin = kJ = P[Xmin > k - 1J - P[Xmin > kJ


(10 - k + 1)s - (10 - k)S
k = 1, 2, ... , 10.
lOs

( Ib) P[X. > kJ = (10 - k)s


I mm (lO)s
Solutions 153

(IlIa) P[Xmax ~ k] = C~y,


P[Xmax = k]= P[Xmax ~ k] - P[Xmax ~ k - 1]

k = 1,2, ... , 10.

(kh
(IIIb) P[Xmax ~ k] = (lOh k = 5, 6, ... , 10.

(IVa) Pk = (7)( 3
10 10
)k-l
' k = 1,2, ....

(IVb) k = 1,2,3,4.

81. (I) and (IV) as in the preceding problem.

I)J,
(lIa)
P[Xmin ;::: k] = 1[7 -[10 -

(k -
~~k -

10
I)J5
'
k = 1,2,3,

k = 4, 5, 6, 7.

1
10 - 2(k - 1)5
k = 1,2,3,
(10)5 '
(lIb) P[Xmin ;::: k] =
(7 - (k - 1»5
k = 4, 5, 6, 7.
(10)5 '
(2k/IO)5, k = 1,2,3,
(IlIa) P[Xmax ~ k] = { (k + 3)/(10)5, k = 4, 5, 6,
(2k)5/(l0)5' k = 1,2,3,
{
(IIIb) P[Xmax ~ k] = (k + 3)5/(10)5, k = 4, 5, 6, 7.

(IIIc) P[Xmax ~ k] = (2k/IO)5, k = 1,2,3,


P[Xmax ~ k] = (k + 3/10), k = 4, 5, 6, 7.
82. Let X be the number of defective items in a sample of 10. Then X obeys
a binomial law

P(X = k) = CkO) (l/IO)k(9/IO)lo-k, k = 0, 1, ... , 10,

and the required probability is


P(X = 0) = (9/10)10.
83. Let X be the number of persons that incur the accident in a year. Then
X obeys a binomial law with n = 5,000 and P = 1/1,000. Since A. = np = 5, the
154 Solutions

Poisson approximation to the binomial (for large n and small p) applies.


Therefore, the required probability is given by
50 5 52) 31
P(X ::;; 2) = e- 5 ( O! + IT + 2! = 2e- 5 •

84. Let X be the number of persons making reservations on a flight and


not showing up for the flight. Then the random variable X has the binomial
distribution with n = 100 and p = 0.05. Hence

P(X ::;; 95) = 1 - P(X > 95) = 1- if (lOOk)


k;96
(0.05)k(0.95)iOO--k.

85. (i) Let X be the number of accidents per week. Then X obeys a Poisson
probability law with parameter A = 2 and hence
P(X ::;; 2) = e- 2 (1 + 2/l! + 22/2!) = 5e- 2 •
(ii) If Y denotes the number of accidents in 2 weeks, then Y has A Poisson
distribution with ). = 4. Hence
P(Y::;; 2) = e- 4 (1 + 4/l! + 42/2!) = 13e- 4 .
(iii) The required probability equals (P[X ::;; 2])2 = 25e- 4 .
86. The number X of suicides per month obeys a binomial probability law
with parameters n = 5 x 10 5 and p = 4 X 10- 6 • Since np = 2 « 10) the
Poisson probability law with A = 2 provides a satisfactory approximation to
the above binomial law. Hence,
4 2k
Po == P(X ::;; 4) = L e- 2 ,k. =
k;O
7e- 2 .

Let Y be the number of months with more than four suicides. Then

P(Y = k) Ck2) (1 -
= PO)k(pO)i2-k,

P(Y 2 2) = 1- {P(X = 0) + P(X = I)}.


87. Because of the independence of the tosses the required probability is
equal to the probability that 20 tosses result in n heads, that is,

( 20) 120
n 2 '
The required conditional probability equals

C03~ n) GY O
C03~ n)
Jo C03~ k) GYO Jo C03~ k)'
88. Suppose that a family has n children and let X be the number of boys.
Then X obeys a binomial law with parameters nand p = 1/2. The probability
Solutions 155

of at least one boy and at least one girl equals


1 1
P(1 :5; X :5; n - 1) = 1- P(X = 0) - P(X = n) = 1 - 2 2" = 1- 2"-1'

and since this probability must be (at least) 0.95 we get 2"-1 ~ 20 and hence
n = 6.
Ak A Ak- 1 A
89. p(k; A) = e-;' k! = k e-;' (k _ I)! = kP(k - 1; A).

For given A, we conclude from the above relation that p(k; A) is increasing in
k for k < A and decreasing for k > A. If A happens to be an integer then
p(A; A) = p(). - 1; A). Thus p(k; A) attains its maximum when k is the largest
integer not exceeding A.

90. (a) f:oo f(x) dx = I2 (1 - 11 - xl) dx

= Jo
f1 x dx + f2 (2 - 1 x) dx
1 1
="2 +"2 = 1.

}'

~-------L------~~-----.x

Triangular distribution

(b)
1
-;-
foo
-00
Pdx
p2 + (x _ a)2 = -;-
1[
arc tan
(x-p-
- a)Joo -00 = 1.
)'

----~------------~------------------~--.x
IX

Cauchy distribution
156 Solutions

(c) f
OO

-00
f(x) dx = -1
20-
fl'
-00
e(X-I'J/u dx + -1
20-
100

I'
e-(x-I'l/u dx = 1.

------------------~--~---------------------x

Laplace distribution

(d) fOO

-00
f(x) dx = -
4
If 0
00
xe- x/2 dx = 1.

max
1/2e
I
I
I
_____ J. _______ _
I

o~----------~----------~----------------__.x
2 4

X121-distribution
91. (i) We have

f oo
-00
f(x) dx = a f3 x dx + a f6 (6 -
0 3
x) dx = 9a,

and the condition

f: f(x) dx = 1
gives
Solutions 157

(ii) P[X > 3] = ~ L6 (6 - x) dx = ~,


P[1.5 < X < 4.5] =! f3 X dx +! f4.5 (6 - x) dx =~.
91.5 9 3 4
(iii) P(AB) = P[3 < X < 4.5] = 3/8 = P(A)' P(B).
Hence A and B are stochastically independent.

92. (a) P[X > 5] = -1 fOC! e- x /5 dx = e- 1 •


5 5

(b) P[3 < x < 6] =! f6 e- x /5dx = e- 3/5_ e- 6/5.


5 3

(c) P[X < 3] =! f3 e- x /5dx = 1 - e- 3/5.


5 0

(d) P[X < 61X > 3] = P[3 < X < 6] = 1_ e-3/5.


P[X> 3]
93. (a) P[0.1 < X < 0.2] = 0.1.
9
(b) L P[0.k5 < X < 0.k6] = 10 x 0.01 = 0.1.
K=O

(c) P[0.3 s P < 0.4] = P[0.09 s X < 0.16] = 0.07.


94. (I) Let X be the height of a man in centimeters. Then X is N(167.9).

X - 167 J
(a) P[X> 167] = P[ 3 > 0 = P[Z > 0] = 21
(Z'" N(O, 1)).

X - 167 170 - 167J


(b) P[X > 170] = P[ 3 > 3

= P[Z > 1] = 1 - <l>(l) = 16%,


where <l> denotes the cumulative distribution function of the standard normal
N(O, 1).
(II) (i) Let Y be the number of men that have height greater than 170 cm.
Then Y obeys a binomial law with n = 4 and p = P[X > 170] = 0.16. Hence
P(Y = 4) = (0.16)4 = 0.0007.
(ii) If Z denotes the number of men that have height greater than the mean
Jl= 167, then the random variable Z has the binomial distribution with
parameters n = 4 and p = P[X > 167] = 0.5. Thus

P[Z = 2] = G) (0.5)4 = 0.375.


158 Solutions

95. (a) Let X be the length of a bolt. Then X is N(5, (0.2)2),


p = P[X ¢ (4.8,5.2)] = 1- P[4.8 < X < 5.2]
= 1 - P[ -1 < Z < 1] = 2(1 - <1>(1)), p = 0.32.

(b) If Y denotes the number of defective bolts in the sample, then Y has the
binomial distribution with n = 10, p = 0.32. Hence P[Y = 0] = (0.68)10.
96. Let T be the time interval (in minutes) between two successive arrivals.
Then T has the exponential distribution with mean 3, i.e.,
f(t) = te- t /3 , t> o.
(a) PET < 3] = 1 - = 0.37.e- 1
(b) P[T> 4] = e- 4 /3 .
(c) Let X be the number of customers per hour and Y be the number of
customers who buy the object. Then X has a Poisson distribution with A = 20.
On the other hand,

P[Y = k/X = n] = (~}O.l)k(0.9rk, k = 0, 1, ... , n.

Hence

L P[Y =
00

P[Y = k] = klX = n]P[X = n]


n=k

= f (n)(O.l)k(0.9rke-202~nn. = e- 22k.:,
n=k k
k = 0,1, ... ,

that is, Y has a Poisson distribution with parameter A = 2.


97. We have
Fy(y) = P[Y :s; y] = p[X2 :s; y] = P[ -.JY < X < .JY]

= F(.JY) - F( -.JY).

Since F is differentiable so is Fy and Y is a continuous random variable with


density

fy(y) = dd Fy(y) = 1r.: [f(.JY) + f( - .JY)].


Y 2yy
98. Using (*) of the preceding exercise, we obtain:
2
(a) fy(y) = j2;ry
1 exp [ - -y+Jl
- 2-
]
, Y> o.
2a 2ny a

(b) fy(y) = _1_ e -Jy, y > O.


2.JY
1 1
(c) fy(y) = - .JY ' y > O.
n y(l + y)
Solutions 159

99. Fy(y) = P[IXI < y] = P[ -y < X < y] = F(y) - F(-y),


d
fr(y) = dy Fy(y) = f(y) + f( - y), y > O.

1
(a) fr(y) = ;;Cexp[ _(y2 + Jl2], y > O.
UY 2n
(b) y > O.
2 1
(c) fy(y) = ~ 1 + y2 ' y > O.

100. Fx(x) = P[X ::; x] = P[log X ::; log x] = P[Y::; log x]


= Fy(log x),
d 1
fx(x) = -d Fx(x) = - fy(log x)
x x

=
1 [(IOgX- Jl
;;C exp - 2 2
)2] ' X > O.
u y 2nx U

101. I = f: Ix - mlf(x) dx = - f:oo (x - m)f(x) dx

+ too (x - m)f(x) dx.

Applying Leibnitz's rule of differentiation,


d iq(y) iq(y) a
-d f(x, y) dx = ~ f(x, y) dx + f(q(y), y)q'(y) - f(p(y), y)p'(y),
y p(y) p(y) uy
we get

dl
dm =
fm -00 f(x) dx -
foo f(x) dx
m

and setting this equal to zero gives

F(m) = f m
-00 f(x) dx
1
= 2'
that is, m is the median of the distribution.

t:
102. We have

E(X) = xf(x) dx = f:oo (x - a)f(x) dx +a f: f(x) dx

= roo (x - a)f(x) dx + Loo (x - a)f(x) dx + a. (1)


160 Solutions

Since E(X) < 00, the following integrals exist

roo (x - a)f(x) dx = J:oo yf(y + a) dy,


{oo (x _ a)f(x) dx = Loo yf(y + a) dy,
and moreover, since f(a + y) = f(a - y), we have

J:oo yf(y + a) dy = - Loo yf(a + y) dy.


Hence from (1) we obtain E(X) = a.
103. E(tX + y)2 = E(t 2X 2 + 2tXY + y2)
= t 2E(X2) + 2tE(XY) + E(y2).
Since E(tX + y)2 20 we get

t 2E(X2) + 2tE(XY) + E(y2) 2 0 for every t.


Therefore the (constant) coefficients of the quadratic in t must satisfy the
relation
E2(XY) - E(X2)E(y2) ~ O.

104. IE(X)I = Itoooo x dF(x) I~ too", Ixl dF(x) = E(IXI).

105. We have

E[JA(X) = 1· P[X E A] + O· P[X ¢ A] = P[X E A] = P(A) = L dFx(x).

106. Applying Chebyshev's inequality, we get


1
P[IX - E(X)I > ka] ~ k 2 '

since E(X) = 0 and L\(X) = E(X2) - [E(X)]2 = 0 it follows that


1
P[IXI > 0] ~ k 2 for every positive integer k.

Hence for every B > 0 we have


P[IXI > 0] < B.

This implies that X = 0 with probability 1.


107. E(X - C)2 = E(X - jJ. + jJ. - cf = E(X - jJ.)2 + (jJ. - C)2,
so that the minimum is attained when c = jJ. = E(X).
Solutions 161

108. {l = E(X) = L: xf(x) dx = J:oo x dF(x) - Loo x d[I - F(x)]

= [xF(x)]~oo - [x(I - F(x)](joo - J:oo F(x) dx + Loo [1 - F(x)]dx

= Loo [1 - F(x)] dx - J:oo F(x) dx,


where we used
lim xF(x) = lim x[I - F(x)] = 0 (Exercise 113).
x--oo x-+oo

109. E(X - e)k = J: (x - e)kf(x) dx

= foo (x - e)k dF(x) - Loo (x - e)k d[I - F(x)]

= [(x - e)kF(x)J_oo - [(x - e)k(I - F(x))]~

- foo F(x)d(x - e)k

+ 1 00
[1 - F(x)]d(x - e)k

= k 1 00
(x - e)k-l[I - F(x)] dx

- k foo (x - C)k-l F(x) dx,

where we used

lim (x - c)kF(x) = lim (x - enI - F(x)] = O.


x--co

110. The rth central moment is given by

r = 1,2, ... , n.

The last relation shows that the first n moments determine the first n central
162 Solutions

moments. The inverse is also true because

J1~ = E[X'] = E[(X - J1) + J1J' = E [ktO G}X - J1)kJ1r-kJ

= ktO G) J1kJ1,-k

provided the mean J1 is given.


111. Suppose X is continuous with density f(x) (if X is discrete the proof
is analogous). Then for every integer r > 0, we have

t: IxI1(x) dx ~ t: K'f(x) dx = K' < 00,

hence E(X') exists.


112. Since the nth order moment exists, the integral f~oo xnf(x) dx converges

t:
absolutely, that is,

Ixl'i(x) dx < +00.


For every k = 1,2, ... , n - 1, we have

t:
and therefore

f:oo Ixlkf(x) dx < Ixlnf(x) dx + 1 < +00.

113. Since the mean of X exists, the integral f~oo Ix I dF(x) also exists. Thus

o= lim x[1 - F(x)] = lim x foo dF(y) ~ lim foo y dF(y) = 0


X-a) x- 00 x x-co x

= lim x[I - F(x)]


x-+-oo
= o.
Similarly, we can show that
lim xF(x) = O.
x-+-oo

114. We have
n
V[SnJ = L V(X k) + 2 L Cov(Xi, X j) = nu 2 + n(n - I)p0"2, (1)
k=l i<j

where
V(Xd = 0"2, k = 1,2, ... , n, p= 0"2 COV(Xi' X), i -# j,
V(X k) = E[Xf] - (E[Xk])2.
Solutions 163

But, since each of the numbers 1,2, ... , N has probability liN of appearing
in any selection, we have for k = 1,2, ... , n
1 N +1
E(Xk ) = - LN
k = -- (2)
N k=l 2
and
2 1 ~ k 2 (N + 1)(2N + 1)
(3)
E
[
Xd = N L..
k=l
= 6 .

On the other hand, since

V[Jl xkJ = V(constant) = 0 = Nu 2+ N(N - 1)pu 2,


we get
1
p=--- (4)
N-1
and from (1), (2), (3), and (4) we obtain

V[Sn] =
n(n 2 -1)[ n-lJ
12 1- N _ 1 .

Note that if X k (k = 1,2, ... , N) were independent, e.g., when we draw a sample
from an infinite population (that is when N ..... (0) then we would have

V[S] = n(n 2 - 1)
n 12'
The dependence of X k , when we draw a sample from a finite population,
reduces the variance by the factor lOO«n - l)/(N - 1))%, referred to as the
finite population correction factor.
115. Let X be the number of white balls in the sample. Then:
(a) X is binomial b(k, n, p), where p = N 1 /(N1 + N 2 ), and hence

(b) X is hypergeometric and

This expectation is easily computed if we represent X as a sum


n
X = L
r=1
X"
164 Solutions

where

x, = {I 0
if the rth selection is a white ball,
if the rth selection is a black ball,
r = 1,2, ... , n,

are identically distributed and therefore

E(X,) = P(white ball at the r selection] = P = NI , r = 1,2, ... , n.


NI + N2
Hence
n
E(X) = L E(X,) =
,=1
np.

116. The random variable X takes on the values 0, 1, and 2 with probabilities
2 4 8
Po = P[X = 0] = 3"5 = 15'
1 4 2 1 6
PI = P[X = 1] = 3"5 + 3"5 = 15'
1 1 1
P2 = P[X = 2] = 3" 5 = 15'
respectively.
(a) The expected number of correct answers is
6 1 8
J-l = E(X) = O'Po + I'PI + 2'P2 = 15 + 2'15 = 15'
2 6 4 10
(b) E(X )=PI+4P2=T5+15=T5'

Hence

V(X) = E(X2) _ [E(X)]2 = 10 _ ~ = ~.


15 225 225

Another method. Let X k be the number 0 or 1 of correct answers in problem


k (k = 1, 2). Then X = X I+ X 2 and so
1 1 8
E(X) = E(Xd + E(X2) = P[XI = 1] + P[X2 = 1] ="3 + 5 = 15'

V(X) = V(X I ) + V(X 2) = P[XI = O]P[XI = 1] + P[X2 = 0]P[X2 = 1]


2 4 86
="9 + 25 = 225'
117. Let X be the gain from the ticket. Then X takes the values 1,000,500,
Solutions 165

and 100 with probabilities 1/3,000, 1/3,000 and 5/3,000, respectively. Hence
1 1 5 2
E(X) = 1,000· 3,000 + 500· 3,000 + 100· 3,000 = 3·
Since a man pays 1 dollar for the ticket the expected (total) gain is 2/3 - 1 =
- 1/3, i.e., a loss.
118. (a) Let X be the number of throws. Then X obeys a Pascal (negative
binomial) probability law with parameters n = 3 and p = 2/3. Hence the
expected number of throws in a performance of the experiment is
n 9
E(X) = P= 2.
(b) In 10 repetitions the expected number of throws is
10· E(X) = 45.

119. Let X be the total gain of the gambler. The random variable X takes
the values-I, 0,1, and 7 with probabilities

P[X = - 1] = P[no ace] = ( -5)3 = -125


6 216'

P[X = 0] = P[l ace] = G)(DGY = ;156'

P[X = 1] = P[2 aces] = (DGYG) = 2\56 ,

P[X = 2] = P[3 aces] = G)GY 2~6· =


Hence

which shows that the game is not fair. The game becomes fair if the gambler
gets a dollars when three aces appear where a satisfies the relation
125 15 a-I a-Ill
E(X) = - 216 + 216 + 2t6 = 216 =0 = a = 111 dollars.
120. Let p. be the probability that the gambler obtains heads for the first
time at the nth toss and X be the gambler's gain. Then X assumes the values
- 15 and 1 with probabilities

P[X = -15] = P[all4 tosses result in tails] = 116 ,


166 Solutions

1 1 1 1 15
P[X = 1] = PI + pz + P3 + P4 ="2 + 4 + 8 + 16 = 16·

Hence E(X) = 0, that is, on the average, the gambler preserves his capital.

121. Let Ai be the event that book i arrives (reaches its destination), let A
be the event that both books arrive, and let B be the event that at least one
book arrives, and let 11 denote the expected net value; then:
(i) If each book is sent separately, we have:
(a) peA) = 0.9 x 0.9 = 0.81.
(b) PCB) = 1 - P(none reaches its destination) = 1 - 0.1 x 0.1 = 0.99.
(c) 11 = 0 x P(A~A2) + 2 x P(A I A 2 u A~Az) + 4 x P(AIA z) - 0.20
= 0 x 0.Q1 + 2 x 0.18 + 4 x 0.81 - 0.20 = 3.4.
(ii) If the books are sent in a single parcel, then:
(a) peA) = 0.9.
(b) PCB) = 0.9.
(c) J1. = 0 x 0.1 + 4 x 0.9 - 0.15 = 3.45.
Thus, according to criteria (a) and (c), a single parcel is preferable, whereas
according to (b), method (i) is better.

122. (a) X = 2Xo where Xo is a Bernoulli random variable with parameter


p. Hence
E(X) = 2E(Xo)= 2p, Var(X) = 4 Var(X o) = 4pq.

(b) X = XI + X z where XI and X z are independent Bernoulli with para-


meters PI and Pz, respectively. Hence

E(X) = E(X I ) + E(X z) = PI + Pz, Var(X) = Var(Xd + Var(Xz)

123. Y
y =1

o~---------------------------------------.x
Solutions 167

F{x) = {01 - 0.8e- x , x ~ 0,


x < 0,
P[X = OJ = F{O) - F{O-) = 0.2,
f{x) = {0.8e-x, x>O,
0, x <0,

E{X) = 0.8 Loo xe- x dx = 0.8.

Suppose that the lifetime of a certain kind of bulb obeys an exponential law
with parameter ,1= 1, and consider a box containing 80% ofthis kind of bulb
and 20% defective bulbs. Then the lifetime of the bulbs in the box has the
above distribution function F{x).

124. E[Y] = foo 1 - t){X) <p{x - J.l) dx


<p x

tf:oo
-00

2
= [1 - <I>{x)] exp[2J.lX; J.l ]d[2J.lX; J.l2]

[2J.lX-J.l 2]1+00
= p.1 [1 - <I>{x)] exp 2 -00

+ P.1 foo-00 exp


[2J.lX - J.l2]
2 <p{x) dx

= °+ -J.l1 foo
-00
1
<p{x - J.l) d{x - J.l) =-.
J.l
125. We have
1 Ak A Ak- 1
L k- = L -,----
00 00

E{X) = - , , - -,,-
e - 1 k=1 k! e - 1 k=1 (k - I)!
1 00 Ak ,12 00 Ak- 2 A2
E[X{X - I)J = e" _ 1 k~2 k{k - 1) k! = e" - 1 k~2 (k - 2)! = 1 - e ,,'
and so

V(X) = E[X{X - 1)] + E{X) - [E{X)]2 = lA~: A" - (I _A; ")2

A{I - Ae-" - e-")


{1 - e-,,)2

126. The elements (simple events) are the following:


AA, ACC, ACBB, ACBACC, ACBAA, ACBACBB, ... ,
BB, BCC, BCAA, BCABCC, BCABB, BCABCAA, ... ,
168 Solutions

and
ACBACBACB. .. (infinite sequence), BCABCABCA ... (infinite sequence).
If we assume that the games are independent, we conclude that the probability
of a simple event consisting of k games equals 1/2\ because the probability of
each player winning a game is 1/2. Hence
1 1 1 1
L 2k"=-+-+-+···=
00

k:2 2 2 4 8
1.

Thus the last two elements of the sample space have zero probability and
therefore the game terminates with probability 1.
127. The sample space consists of the elements:
HH, HTT, HTHH, HTHTT, HTHTHH, ... ,
TT, THH, THTT, THTHH, THTHTT, ... ,
and
HTHTHTHT. .. } ... ..
up to mfimty, wIth zero probablhty.
THTHTHTH ...
The required probabilities are

P(A) = t 2-;2 =
k:2 1_(~)6
2
= 63,
64

P(B) = f
k:1
(~)2k. 2 = ~.
2 3
128. The number of all possible outcomes are 36 6 = 6 12 . The number
of favorable outcomes are 12!/26 and hence the required probability is
12!/(2 6 . 6 12 ) = 0.0034.
129. Let Ak be the event that the player k has a complete suit. Then the
required probability is p = P CVI Ak) and by the Poincare formula we get

where
Solutions 169

130. Let the event A be that I win the bet. Then, denoting the result "tails"
by T, we have
A = HHHuHHTuTHH.

Tossing the coin C 1 first, that is, playing according to the series C 1 C2 C 1 , we
have
peA) = P1P2P1 + P1P2(1 - pd + (1 - P1)P2P1 = P1P2(2 - P1)'
On the other hand, tossing the coin C2 first we have
peA) = P2P1P2 + P2P1(1 - pz) + (1 - P2)P1P2 = P1pz(2 - P2),
and, since P1 > P2'
P1P2(2 - pd < P1P2(2 - P2),
that is, you must select coin C2 for the first toss.
Remark. If one has to play against two persons, he should play first against
the stronger player in order to maximize his chances of winning.
131. Method 1. If we denote by Ak the appearance of the kth pair, then the
required probability is

Applying Poincare's theorem, since P(A i ! A i2 .. . Ai) = °for k ~ 3, we get

20· 18 . 16· 14 224 99


P = 1 - P(no one pair) = 1 - 20. 19. 18. 17 = 1 - 323 = 323

or

CO} 24
P ~ 1 - Pf«iection of 'hoo< belonging to 4 pai,,) ~ 1- (2;) 99
323

For a random selection of 10 shoes the probability of selecting no pair is less


than 6%.
170 Solutions

132. (a) (l/N)(l - 1/N)n-1 (geometric with P = liN).


(b) (1 - II N)" because the first n balls must fall in the cells 2, 3, ... , N.
133. (a) The n (n :$; N + 1) first balls must fall in different cells. Thus the
required probability is (N)nIN n.
(b) The probability, say Pn' that n throws will be necessary equals the
probability that the first n - 1 balls will fall in different cells and the nth ball
in one of the n - 1 occupied cells. Hence
(N)n-1 n - 1
Pn = N n- 1 ---;;;-'

and the expected number of throws is given by the sum


N~l _ ~ n(n - 1)(N)n-1
L... nPn - L... n .
n=i n=i N
134. (a) Let E1 be the event that the nth card is the first ace. Then

P(E ) = (48)n-1 (4)1


1 (52)n'
(b) Let E2 be the event that the first ace appears among the first n cards

P(E ) = 1 _ (48)n.
2 (52)n
135. The thirteenth diamond may appear at the nth position (26 :$; n :$; 52).
Of the 52! cases (permutations ofthe 52 cards), the number offavorable cases
is equal to
L
52
n=26
(n - 1)!(52 - n)! ( 26 ) .13.
n - 26
136. To each triplet of numbers out of the {I, 2, ... , n} there correspond
six permutations (ordered triplets), and of these, three have the first number
smaller than the second. Since the (nh ordered triplets are equiprobable, the
required probability is 1/2.
This also follows from the fact that the first number is equally likely to be
larger or smaller than the second one.
137. Let p(n, m) denote the probability that in placing n balls in m cells all
cells are occupied. Then the required probability Pn is given by
Pn = p(n, m) - p(n - 1, m).
To find p(n, m), let Aj be the event that thejth cell is empty. By the Poincare
formula (1.1)
p(n, m) = 1- P(A 1 U'" U Am) = 1- Sl + S2 - ... + (-lrSm
= .f (-l)j(~)(l _1)n,
)=0 m )
Solutions 171

since

138. (a) The probability function of X is


-). Ak
ek!
P[X = k] = n Ak k = 0,1, ... , n,
c k '. '
L e-).-
k=O k!
where we set
n Ak
c= k=O
L -k'·

For the random variable Y we have


P[Y = ka - (n - k)b] = P[Y = k(a + b) - nb] = P[X = k]
1 Ak
k=0,1, ... ,n.

Hence we find
(a + b)A n+l
E(Y) = A(a + b) - , - nb.
n.c
(b) To maximize E(Y), it suffices to minimize, with respect to n, the
expression
(a + b)A n + 1
,
n.c
+ nb.

139. Of the six permutations (cf. Exercise 136) only one is such that its first
number is the smallest and the second number the largest. Hence the required
probability is 1/6.
140. By the Poincare formula (1.1) we have
P(AB u AC u BC) = P(AB) + P(AC) + P(BC) - 2P(ABC) = 0.104
using also the independence of A, B, C.
141. (a) Let Xn denote the outcome of the nth coin on the first throw and
y" the outcome of the second throw (n = 1,2,3,4). Then the required prob-
ability is

P[Xn = Y", n = 4
1, 2, 3,4] = ]] P[Xn = y"] = (1)4
2: .
(b) Let X be the number of heads on the first throw and Y be the number
172 Solutions

of heads on the second throw; then


4 4
P[X = Y] = L P[X = k, Y = kJ = k=O
k=O
L P[X = kJP[Y = kJ
_t (4)2
- k=O k
1 _ 35
28 - 128
in virtue also of Exercise 21.

142. (a) P[X = 'J = (b)j_, W j = 1,2, ... , b + 1.


] (n)j'
J'(b)
L jP[X = j] = A ~I
L ~.
b+! p+!
(b) E[XJ =
~I ~1

(c) The required relation follows from


b+1 p+1 (b)
L
j=1
P[X =j] = A L ~ = 1.
j=1 (n)j

143. Obviously, when this occurs 2j balls will have been drawn (j = 1,2,
... , N), and according to the hypergeometric distribution the required prob-
ability is equal to

P(AIc)
P(AC)
=-- =
p[ 0ACBj ] = t P(ACB)
144. }-I <-J-_I_ __
P(C) P(C) P(C)
00

L P(AICBj)P(BjC)
. 1 00

J= P(C) = j~ P(AIBjC)P(Bjlc).

145. The probability of success for Rena is 1/3 and the probability offailure
is 2/3. The probability of success for Nike is: 2/3' 1/2 = 1/3. The probability
of success for Galatea is: 2/3' 1/2' 1 = 1/3. Therefore the process is fair (cf.
random sampling from a finite population).
In the second case, the probability of success for Rena is a (say) and the
probability for Nike is b = a(2/3) and for Galatea is c = a(2/3)' (2/3). Hence

9 b=~ 4 since a + b + c = 1.
a = 19' 19' c = 19

Therefore Nike and Galatea rightly protest.


146. Let Ai be the event that the ith face of the die appears. Let the given
Solutions 173

sides be iI' i2 , ••• , ik • Then the required probability is

where

SI = JI P[A;J = G)G)"
S2 = ~ P[A;nA;m] = G)(~r

Hence

Application. What is the probability that each face will appear at least once?
Applying formula (1) for k = 6 we find

P6 =.to (_1)"(~)(6 ~ n)' =.to (-I)"C)(6 ~ n)"


(cf. Exercise 137).
147. From

fo OO
f(x) dx = c foo _1_ e- x2 / 2 dx = -c = 1
0 fo 2
we have c = 2.
E(X) = f oo xf(x) dx = foo 2x _1_ e- x2/ 2 dx = - -2- foo d(e- x2/ 2 )
o 0 fo fo 0

- - 2- e _x 2/2 1 _2_ =J2


00
= = _
fo 0 fo .;-;c'
148. (a) Let X denote the lifetime in hours of an electric bulb. Then

- 180 ]
q = P[X < 200] =P[X 20 < 1 = <l>(I) = 0.84.
Therefore the required probability is
(1 - q)4 = p4 = (0.16)4.
174 Solutions

(b) Consider the event Ak where the urn contains k bulbs with lifetime
greater than 200 hours, and the event B where the selected bulb has a lifetime
greater than 200 hours.

P(B) = kt1 P[(BIAk)]P(Ak) = kt1 ~(:)pkq4-k


=P t(
k=1
3 )p k - 1 q4-k = p;
k - 1
that is, the probability of selecting a bulb with lifetime greater than 200 hours
is the same whether a bulb is chosen from the population of bulbs or from a
random sample of size n (here n = 4) of the population.
149. Let X denote the outcome (sum) in a throw 2 ~ X ~ 12. Then
P[X = 2] = P[X = 12] = 1/36, P[X = 3] = P[X = 11] = 1/18,
P[X = 4] = P[X = 10] = 1/12, P[X = 5] = P[X = 9] = 1/9,
P[X = 6] = P[X = 8] = 5/36, P[X = 7] = 1/6.
Let Xn denote the outcome of the nth throw, let W be the event the gambler
(finally) wins, let W1 be the event he wins on the first throw, and let L1 be the
event he loses on the first throw, then
P[Wd = P[X = 7] + P[X = 11] = 1/6 + 1/18 = 2/9,
P[L 1] = P[X1 = 2] + P[X1 = 3] + P[X1 = 12] = 1/9,
and therefore
P(more than one throw) = 1 - P(W1) - P(Ld = 2/3.
We now have for case (a)
2 10
P[W] = 9 + i~4 P[WIX1 = jJP[X1 = jJ, (1)
i#7
and from
ao
Pi = P[WIX 1 = jJ = L P[X. = 7]P[Xi ::f: 7, Xi ::f: j, i = 2, ... , v-I]
.=2
we find
Solutions 175

Substituting in (l) we find

PEW] = ~: = 0.618.

Under (b) we similarly obtain


2 10 244
PEW] ="9 + j~4 qjP[Xl = j] = 495 ~ 0.493,
U7

where we set Pj = 1 - qj. We notice that (b) defines an almost fair game. In
fact, this is the most usual way of playing the game.
150. The expected profit for B is by Exercise 149
1 2 3 4 5 6 5 4 188
2· 36 + 3 . 36 + 4· 36 + 5· 36 + 6· 36 + 7· 36 + 8· 36 + 9· 36 = 36'
For A this is
3x 2x x 6x
36 + 36 + 36 = 36 .
The game is fair if 188/36 = 6x/36. Hence x = 31 t.
151. Let Ei be the event that one white ball and one red ball are drawn on
the ith draw. Then by the multiplication rule the required probability is
P[E 1 E2··· En] = P[E l ]P[E2IE l ]··· P[EnIE 1 ••• En- 1 ]

Another method. The number of ways in which 2n balls may be divided into
n pairs is (2n)!/2 n of which (n!)2 are favorable (for pairs of different color)
because to every permutation of n white balls there correspond n! permutations
of red balls which give pairs of balls of different color.
152. Let Ei be the event that the number of the ball drawn from urn A is
i and from B larger than i. Then the required probability is

n-l n-l n- i n- 1
P = PEEl U E2 u··· U En- 1 ] = L
i=l
P(E;) = L -2
i=ln
- = -2-'
n

Another method. It is equally likely that the ball drawn from A will bear a
number larger or smaller than that of B and the probability of equal numbers
is l/n. Therefore
1 n-l
p+p+-=1 and p=--.
n 2n
176 Solutions

153. Let Po be the required probability. Then:


(i) (a) Every person after the second person chooses one of the other
persons, except the first one. Therefore we find

Po -
_ [(N - 1)]n-l -_(1 _~)n-l
N N
(b) Each ofthe n persons chooses one of those who have not been informed.
Hence Po = (N)n/N n.
Similarly, when it is told to k persons at a time, we have

(a) P0--
(N -
Nn
kr l - [1 - -Nk]n-l '
1 - (b)
(N)nk
Po = [(Nh]n
(ii)

(a) n = pN and

(b) lim Po = lim


N~oo N~oo
(1 - ~)(1
N
- ~)
N
... (1 - ~).
N
154. Each gambler may win the game on the kth throw of the game (event

Ad where k = n, n + 1, ... , 2n - 1 with probability (~= ~)GY because


there are (kn -- 1)
1
ways of winning n - 1throws out of k- 1throws and the
kth throw must be a success.
Thus the probability that each player wins the game is

ZIk=n (kn -- 1)1 (~)k


l

2
= ~.
2
The probabilities after interruption are

P(I) = L (v - k -
2n-l m- 1) (l)v-k-m
-,
v=n+k n- k - 1 2

P(II) = L (v - m -
Zn-l k- 1) (l)v-k-m
-.
v=n+k n- m- 1 2
The amount will be divided in parts proportional to P(I), P(I/).
155. Let A/ B be a rational fraction and a an integer. The possible re-
mainders of the division of A by a are 0, 1, ... , a-I. Hence the probability
that A is divisible by a is l/a. Similarly for B. Therefore the probability that
both A and B are divisible by a is l/a 2 • The fraction A/B is an irreducible
fraction if and only if both A and B are not divisible by any of the prime
numbers 2, 3, 5, .... Hence the required probability is given by
Solutions 177

This is obtained by noting that

P
where the summation extends over all nonnegative integers a, b, c, ... , and
taking into account that any positive integer n is of the form n = 2a • 3h • sc· ...
156. Let Ap be the event that the chosen number is divisible by the prime
p. If PI' P2' ... are different prime divisors of n the events ApI' A p2 , .. , are
independent. The probability that the chosen number is prime with respect
to n is, by definition of cp(n), cp(n)/n.
On the other hand, P(Ap) = lip, because there are q numbers out of 1, 2,
3, ... , n which are divisible by p, where n = pq. Thus
q q 1
P(A ) =- =- = -;
P n p'q p
therefore
cp(n)
n
= p[n A~J = fl P(A~) = fl (1 - ~).
~ ~ ~ p
157. The generating function of the number of insects

where N is the number of colonies and lj is the number of insects in the jth
colony, is given by
PSN(t) = exp{A(P(t) - I)},
where P(t) is the generating function of lj. But

P(t) = log(l - pt)


log(l - p)
and therefore

PSN(t) = exp {A [11:~~11 ~ ~: - I]},

that is, the probability generating function of the negative binomial distribu-
tion.
158. Let Ak be the event that the lot contains k defective tubes, and let E,
be the event that in a sample of n tubes r are defective. Then
(a)

P[E,IA k ] = ( : =:)G)/(~)'
(Bayes's formula).
178 Solutions

(b)

df (N =n)j(N)
k=r k r k
1- .
Lm
j=r
(N -
Pj.n)j(N)
] - r ]
.

159. Let Ak be the event where k defective articles are bought, let Bn be the
event where n customers come into the shop, and let E be the event where a
customer buys a defective article. Then we have
peE) = P(of buying)P(buying a defective item Ibuying)
2 3
= 3·4 = 2'

P[AkIBnJ = (~);n'
and

160. Let W denote the weight of an article. Then the proportion of articles
with weight less than Wo is

P = P[W < woJ = -1- f~ dw = -1- f~~ e- w2/2 dw.


fo fo
e-(w-Il)2/2
-00 -00

Then the expected profit is


K = (1 - p) - (a + bJl).
From

we have the value of Jl (Jl > wo ) which maximizes the expected profit.
161. P[the clerk is lateJ = P[the train arrives after 8:45J + P[the train
arrives before 8:45JP[the bus arrives after 9J = 1 - <1>(3/4) + <1>(3/4)[1 -
<I>(2/3)J = 1 - <1>(3/4)<1>(2/3) ~ 0.4.
(a) P[that the clerk is lateJP[that the employer is lateJ = 0.4 x 0.07 =
0.028 because we have P[that the employer is lateJ = 1 - <1>(3/2) = 0.07.
(b) P [that the employer arrives before the clerk J = P [that the train arrives
before 8:45JP[the bus arrives after the employer's carJ + P[that the train
arrives after 8:45J = <I>(3/4)<I>(1/ji3) + 1 - <1>(3/4) ~ 0.5 because the arrival
time X of the bus obeys the normal probability law N(8: 58,3 2 ) and the arrival
Solutions 179

time Y of the private car follows the N(8: 57,22) and therefore

P[X> Y] = P[X _ Y> 0] = p[(X - 8:58) - (Y - 8:57) > __I_J


UX-y UX - y

= $(Jo) ~ 0.61.

162. This is due to Dodge (see Rahman, 1967, p. 159).


(a) Let En be the event that the nth article examined is the first defective
article (n = 1, 2, ... , r), and let A be the event that the sequence is defective.
Then
r r
a = PEA] = P[E 1 U E2 u··· u Er] = L P(E n ) = L pqr-1 = 1- qr. (1)
n=l n=l

a is also the probability that among r articles at least one is defective. Hence (1).
(b) Let X be the number of examined articles of a defective sequence. Then

1 [ ] f kP[X = k] f k q k-1
A =E XIA = L.. = P L...--
k=l PEA] k=l 1 - qr
1 - qr(1 + rp)
p(1 _ qr)

(i) Let Y be the number of defective sequences. Then Y obeys the geometric
probability law with parameter 1 - P(A) = 1 - a (probability of success).
1 _ qr
L
00

Jl = E(Y) = n(1 - a)a n = - r - = q-r - 1.


n=O q
1 _ qr
(ii) AJl + r = -pqr- = r.
(c) Let ({J denote the required percentage. Then
({J = (average of examined articles before a defective article + r)/(r + r*)
= fl[f + qr(1 - f)],
where we put r* as the average of articles which passed before a defective one
was found; and f- 1 x the average number of examined articles before a
defective one was found.

= 1_ = pqr(1 - f)
(d) p p( ((J) f + qr(1 - f).
(e) p is maximized when p = p* as obtained from

(1 _ f)(1 _ p*r) = f[(r + l)p* - 1].


1 - p*
180 Solutions

For a more detailed discussion of Exercises 163-170 we refer to Mosteller


(1965).
163. The error in A's syllogism is due to a mistaken sample space. He thinks
each of the pairs of convicts AB, AC, BC, who are going to be set free, has
probability 1/3. This is correct from the point of view of the Board. But when
we keep in mind the answer of the guard, we have the events:
1. A and B free and the guard answers B (Bl event).
2. A and C free and the guard answers C (C 1 event).
3. Band C free and the guard answers B (B2 event).
4. Band C free and the guard answers C (C 2 event).
Then

PEA free 1·f t he guard says BJ = PEA free and the guard says BJ
[ h d J
Pte guar says B
P(Bd 1/3
1/3 + 1/6 = 2/3.
Similarly, if the guard answers that C is set free.
Another method. It is equally likely that the guard is going to say that B (B
event) or C (C event) is set free. Then, if A is the event that A is going to be
set free, we have

P[AIBJ = P[ABJ = 1/3 = ~


P[BJ 1/2 3·
164. We obtain a number from the first box. The probability of a new
number from the next box is 5/6. Therefore, on average, 1/(5/6) = 6/5 are
required for the next new number because the mean of the geometric distribu-
tion is l/p.
Similarly, 6/4 boxes will be required for the third number, 6/3 for the fourth,
6/2 for the fifth, and 6 for the sixth. Hence the required number of boxes on
average is

For n coupons and large n, the required number is n log n + 0.577n + 1/2
boxes because
1 1 1
1 + - + ... +- ~ log n + - + C,
2 n 2n
where C = 0.57721 denotes Euler's constant.
Solutions 181

165. The probability of a couple at the first two seats is


10 9 91010
19'18+ 19'18= 19'
and the expected number of pairs at the first two seats is also
10 10 9
19 = 19' 1 + 19' O.

The same is true for every pair of successive seats and there are 19 - 1 = 18
such pairs. Therefore the expected mean number of pairs (male-female student
or female-male student) is 18 '10/19 = 9 199 = 9.47. More generally, for a male
students and b female students the mean number of couples is

(a +b- 1)[ ab
(a + b)(a + b - 1)
+ ab
(a + b - l)(a + b)
] = 2ab .
a+ b

Remark. We used the fact that the mean of a sum of random variables is
equal to the sum of the mean values of the corresponding random variables,
no matter whether the variables are (statistically) independent or not.
166. If A shoots and hits C, then B will certainly hit him; therefore he
should not shoot at C. If A shoots at B and misses, then B will shoot at the
more dangerous, i.e., C, and A has a chance to shoot at B and hit him with
probability 0.4.
Naturally, if A fails then he has no chance. On the other hand, suppose
that A hits B. Then C and A shoot alternately at each other until either one
of them is hit. The probability that A will win is
(0.5)(0.4) + (0.5)2(0.6)(0.4) + (0.5)3(0.6)2(0.3) + "', (1)
where the nth term expresses the probability that in 2n trials C will fail n times,
B will fail n - 1 times, and A will hit in the end. The sum in (1) is
(0.5)(0.4) 2
1 _ (0.5)(0.6) = "7 = 0.28 < 0.4.

Thus if A hits B and continues with C, A has a smaller probability of winning


than if the first shot fails. Hence to start, A shoots in the air and tries to hit B
in the second round. Then C has no chance (of surviving).
167. The expected appearances of a 6 in the experiments of A, B, Care 1,
2, 3, respectively, but this does not mean that the corresponding probabilities
of the events are equal. In fact, we have

P(A) = 1 - P(no 6) = 1 - GY ~ 0.665,

5)12
P(B) = 1 - P(no 6) - P(one 6) = 1 - ( 6" - (12)(1)(5)11
1 6" 6" = 0.619,
182 Solutions

P(C) = 1 - P(no 6) - P(one 6) - P(two 6's)

Indeed Newton advised Pepys to choose the throw of six dice.


168. Employing one worker secures 364 working days; employing two
workers, in all probability having different birthdays, we secure 2 x (365 - 2) =
726 working man-days, whereas if the number of workers is large there is a
great probability that every day is a holiday because of somebody's birthday.
Hence there is an optimum number of workers to be employed by the factory.
Consider n workers and let N be the number of days in a year. The
probability that any day is a working day is equal to
p = P[no birthday on a given day] = [(N - 1)/N]n = (1 - l/N)",
and this day contributes, on average, np man-days. This is true of any day.
Hence the total expected number of man-days during the whole period of N
days when n workers are employed is equal to nNp. This is maximized for
some n = no if

and

from which we obtain

(no + 1) (1 - ~) ~ no and

and finally
no < N < no + 1.
no = N - 1 and no = N give the same maximum, i.e., N 2 (1 - (I/N))N; hence
t
take no = N - 1. For large N, (1 - N- 1 ~ e- 1 and hence the expected
number of man-days is N 2 e- 1 , while if every day was a working day it would
be N 2 . Thus in a usual year (365 days) the no = 364 workers will offer about
49,000 man-days. No doubt the wages must be very low in Erehwon.
169. There are several versions of the problem depending on the way in
which a chord is chosen. We consider three versions.
(a) The middle of the chord is chosen at random, i.e., it is uniformly
distributed over the circle. In this case, we have
area C 3 1
P[(AB) < r] = P[M ¢ C] = 1 - C = 1 - -4 = -4'
area
where C is a circle with radius (see Fig. 1)
(KM) = r cos(1t/6) = r.j3/2.
Solutions 183

Figure 1

(b) Since A is some point on the circumference, take it fixed and choose B.
Clearly now
120 1
P[(AB) < r] = 360 = 3'
since if B is chosen 60° on either side of A on the circumference, (AB) will be
smaller than r.
(c) For reasons of symmetry, we suppose that AB has a given direction,
say, of the diameter 00'; in this case, the middle M of the chord is uniformly
distributed on the diameter KK' which is perpendicular to 00' (Fig. 2). We
now have
P[(AB) < r] = P[(TM) > (TL)] = P[(TM) > {} rJ
= 1- 2r}3/2 = 1- =0.13. J3
2r 2

K
A B

0' t - - - - - ' I ' = - - - - - l 0

K'
Figure 2

Remark. The variety of solutions exhibited is due to insufficient description


of the corresponding random experiment which determines the choice of the
184 Solutions

chord. Thus in (a) we essentially choose at random a point M in the circle and
require the probability that M lies in a smaller homocentric cycle. In (b) the
points A and B are randomly chosen on the circumference, whereas in (c) we
can imagine a cyliner rolling on a diameter of the circle (say KK') and the
equally likely events correspond to equal intervals on the diameter (irrespective
of their position on the diameter).
170. This is another version of the beauty contest problem or the dowry
problem (how to select the most beautiful candidate or the highest dowry
without being able to go back and choose a candidate already gone). The
neophyte at a given stage of the race knows the rank (first, second, etc.) of the
horse passing by; only the horse having the highest rank (up to that point)
may be considered for final selection. Such horses will be called "candidates".
Before we consider the general case, let us see some simple cases. For n = 2
the neophyte has a chance of 1/2 of choosing the better horse. For n = 3,
choosing at random, e.g., the first horse he chooses has a probability 1/3 of
winning since of the six cases 123, 132*,231 *,213*,312,321 only the last two
are the favorable ones. The probability gets higher if he ignores the first horse
and chooses the next better one. This procedure gives the three cases marked
with an asterisk and therefore the probability becomes 1/2.
In general, it will be shown that the best strategy is to let s - 1 horses pass
and choose the first candidate after that. Let ~ denote the event that the ith
horse wins. Then the ith horse will be chosen (as the best one) if
k = 1,2, .... (1)

We will show that the right-hand side is a decreasing function of i, whereas


the left-hand side is an increasing function of i, so that at some stage the
process of observing more horses should stop and a decision taken. We
observe that the probability that the ith horse is the best one is equal to the
probability that the highest rank is among the first i ones, i.e., i/n, that is, an
increasing function of i and at some stage it is larger than the probability of
winning later. According to this strategy, the neophyte letting s horses pass
has a probability of winning equal to the probability of coming across a single
candidate after the nth stage. Thus we have
P[~] = P[kth horse has highest rank]
x P[best of the first k - 1 horses is among the first s - 1 horses]
1s - 1
n k - l'
for s ~ k ~ n and hence
1 s- 1
L P[~] = -n L - k -1
n n
n(s, n) = P[to win] =
k=s k=s -

s- 1 n-1 1
=-
n
L-k'
k=s-l
1< s ~ n. (2)
Solutions 185

Certainly the first horse is a candidate and n(1, n) = l/n. The optimum value s*
of s is the smallest integer for which (1) holds, i.e.,

-s > n(s + 1, n)
n
= -s
ns
(1 + -s+l1- + ... + -n-1
- 1-)
or

-1 + -1- + ... + -1- < 1 < -1- + -1 + ... + - -


1
. (3)
s s+1 n-l s-l s n-l
For large n the approximation
1 1
1+- + ... + - ~ log n + C,
2 n
where C = 0.57721 is Euler's constant, gives
s-1 n-l s n
n(s, n) ~ --log - - ~ -log-,
n s-1 n s
and by (3), s* ~ ne- l , i.e., for large n the neophyte must wait until e- l = 37%
of the horses pass by and then choose the first candidate. It can be found that
for n = 10, s* = 4, and n(s, n) = n(4, 10) = 0.399; similarly, n(8.20) = 0.384,
n(38,100) = 0.371 thus approaching e- l as n ~ 00. A random choice ofa horse
as the best would give l/n.
171. (a)
Fx(x) = 0, x < 0, Fx(x) = 2/3, ° ~ x < 1, F(x) = 1, x ~ 1,
Fy(Y) = 0, Y < 0, Fy(Y) = 1/3, ° ~ Y < 1, F(y) = 1, y ~ 1,

o °or y < 0,
°
for x <

F(x, y) = {
1/3 for (0 ~ x < 00, ~ y < 1),
2/3 for (0 ~ x < 1, 1 ~ y < 00),
1 for x ~ 1, y ~ 1.

(b) (- 1/2, 0) is a continuity point. (0, 2) is a discontinuity point of F(x, y)


because for x < 0, F(x, 2) = 0, while F(O, 2) = 2/3.
172. F(x, y) must satisfy F(X2' Y2) - F(Xl' Y2) - F(X2' Yl) + F(Xl' Yl) ~
°for all Xl ~ Yl' x 2 ~ Y2 (why?); for (Xl' yd = (0, 0), (X2' Y2) = (2, 2) we have
F(2, 2) - F(2, 0) - F(O, 2) + F(O, 0) = 1- 1- 1+ ° = -1;

thus it is not a distribution function.


173. P[Xi = 1] = P[Xi = 0] = 1/2, i = 1,2,3;
P[Xi = ai' Xj = aJ = P[Xi = a;]
for all i i= j (i,j = 1,2,3) and ai' aj = 0, 1; then Xl' X 2 , X3 are pairwise
186 Solutions

independent. But the relation

does not hold, for example, for a l = a2 = a3 = 1; then

P[XI = 1, X 2 = 1, X3 = 1] = 0 #- n P[X;
3

;=1
= 1]
1
=-.
8
174. Putting x = yu we have,

fy(y) = ce- Y f: X n1 - 1 (y - x)n 2 -1 dx = ce- Yy n1+n 2-1 fOl un,+n2-1(1 - u)"rl du

= cyn, +n 2 -l e - y r(nl )r(n2)


r(nl + n2 )'

and from the equation

we have c- l = r(ndr(n 2) and Fy(y) is a Gamma distribution with parameter


n l + n 2 • Similarly, we find that X has the Gamma distribution with param-
eter n l :

X> o.

175. For n; ~ 0 and n l + ... + n k - l + nk ~ n, we have

where Xk = n- (Xl + X2 + ... + X k - l ). This is hypergeometric.


176. Similarly as Exercise 175.
177. The marginal distribution of Xl' ... , Xr (r < k) is Dirichlet with
density

f(
Xl'···'Xr
)-
r( nl + n2 + ••• + nk + l )8r"k+Nr- +l
l

-r( n l ) ... r( nr )r( n r + l + ... + nk+l )


n x,!,,-l ,
r
(1)
;=1

where we put
8 r +1 = 1- (Xl + X2 + ... + X r ), Nr = n r +l + ... + nk+l·
Solutions 187

Equation (1) for r = 1 and r = k - 1 gives as density of X k for given Xi = Xi


(i = 1,2, ... , k - 1),

Then the conditional distribution of Xk/R k for Sk = Sk is p(nk, nk+l).


178. The density is
f(x, y, z) = 1/V,
(1)
(x, y, z) E S = {(x, y, z), x ~ 0, y ~ 0, z ~ 0, x + y + z ~ c},

f
where V = volume of (S) = c 3 /6. The joint density of (X, Y) is
6 C
-
X
- Y 6(c - x - y)
f(x, y) = 3" dz = .
c 0 c
Note. For c = 1, (1) and (2) are special cases of Exercise 177.
179. (a) Since the quadraticform (see (5.10)) x 2 - xy + y2 is positive definite,
f(x, y) is the density of the bivariate distribution N(p., I:) where

_~(2 1)
I:- 31 2·

Hence

(b) The ellipses x 2 - xy + y2 = constant.


180. (a) As in Exercise 178, we find
f(x, y) = 1/2, x ~ 0, y ~ 0, x + y ~ 2.
(b)

fx(x) =
1
2
f2-X
0
1
dy = 2(2 - x).

(c) f(x, y) 1
fy(YIX = x) = fx(x) = 2 _ x' 0< y < 2 - x.

181. (a) Multinomial Pi = P2 = ... = P6 = 1/6, n i = n 2 = ... = n6 = 2,


n = 12.
(b)

P[X = i, Y = j] = i!j! (121 = G)i+iGY2-i-i, °


i _ j)! ~ i +j ~ 12.

Trinomial with Pi = P2 = 1/6, P3 = 2/3.


188 Solutions

(c) Cov(X, Y) = E(XY) - E(X)E(Y),

1 1
E(X) = npl = 12. 6 = 2, E(Y) = np2 = 12. 6 = 2,
i j n-i-j
E(XY)= L i" P1P2P3
O$;i+j$;n 'J i! j! (n - i - j)!
(n - 2)! p~-l pt 1p~-i- j
= n(n - l)P1P2 L.
2$;i+j$;n (z -
.
I)! (j - I)! (n -
. .J)!
I -

m! p~p~p~-S-k
= n(n - l)P1P2 L
09+k$;m s! k! (n - s - k)!

= n(n - I)P1P2(Pl + P2 + P3)m = n(n - l)P1P2


(where m = n - 2, S = i-I, k = j - 1). Hence Cov(X, Y) = -nplP2' and in
this case Cov(X, Y) = -1/3.
182. We have
P[XI = n1, •.. , Xi = ni , ... , Xj =
nj , ... , X k = nk] ni + I.l!l
P[XI =n1,···,Xi=n i + I, ... ,Xj=nj-I, ... ,Xk=nk] nj Pi
for i,j = 1,2, ... , k, i =j.
Thus the vector (n~, n~, ... , n~+l) is the most probable, if and only if, for every
pair of indices i, j (i #- j) the inequality

PinJ spin? + 1) (1)

is satisfied. From this, summing over j withj #- i we have


k+l k+l
Pi L
j=l
nJ s (n? + 1)
j=l
L Pj'
Ni Ni

p;{n - n?) s (n? + 1)(1 - Pi), (2)


npi S n? - Pi + 1 < n?,
npi - 1 s n? - Pi < nf.
Similarly, summing over i withj #- i we get
nJ spin + k), j = 1,2, ... , k + 1. (3)

From (2) and (3) the result follows.


183. Putting npi = ).i (i = 1,2, ... , k), in the multinomial probability func-
tion, we have
Solutions 189

Hence from (1) the result follows by taking the limit (n -+ 00).
184. (a)

F(x, y) = 2 LX du J: (1 + u + V)-3 dv = LX [(1 + U)-2 - (1 + u + y)-2] du

1 1 1
=1---+ ---
l+x l+x+y l+y
(b)
fx(x) = J0
OO
f(x, y) dy = 2
Joo (1 + x + yf3 dy = (1 +1 X)2·
0

(c)
}; ( IX = x) = f(x, y) = 2(1 + X)2 .
Y Y fx(x) (1 + x + y)3
185. fx(x) dx '" area of the infinitesimal strip ABB' A' (see figure), so that
1
f(x) dx = -(2~).
11:

_-T-.....:A A'

x x

BB'
190 Solutions

Hence
2
fx(x)=-~, -1 < y < 1,
1t

1 1 ~ .
Fx(x) = - + -(xv 1 - x + arc sm x),
2 1t

= 1 - -1 (area 0 f the cIrcular


. part (AOB)),
1t

Fy(y) = Fx(Y) = 1 - Fx( - y).


X and Yare not independent because, for example,

p[x> f, Y> fJ=o#P[X> fJp[x> fJ


that is, the relation
P[X E A, Y E B] = P[X E A]P[Y E B]
does not hold for all A, B.

186. P[X < Y < Z] = II dz


000
IZ dy IY dx =!.6
This is so, since every ordering of X, Y, Z has the same probability because
of symmetry of the joint density.
187. (a)
Fx(x) = Fy(x) = x 2, F(x, y) = x 2y2, fx(x) = fy(y) = 2x, 0 < x, y < 1.
(1)
4xy
fx(xl Y = y) = 2y = 2x, fy(YIX = x) = 2y. (2)

From (1), the relations (2) follow because of the independence of X, Y.


Solutions 191

188. We have
= E(X) = -0.35 + 0.20 =
I1x -0.15,
E(X2) = 0.35 + 0.20 = 0.55,
Var(X) = E(X2) - 11~ = 0.55 - 0.0225 = 0.5275,
ax = JVar(X) = 0.726,
l1y = E(Y) = -0.3 + 0.1 + 0.8 = 0.6,
2
E(y2) = 0.3 + 0.1 + 1.6 = 2, Var(Y) = E(y2) - - = 2 - 0.36 = 1.64,
Y
ay = JVar( y) = 1.28,
E(X Y) = 0.1 - 0.05 - 0.2 + 0.2 = 0.05,

p=
Cov(X, Y) E(XY) - I1xl1y 0.05 + 0.09 = 0.14 = 0.15.
axay 0.726 + 1.286 0.932
(a) The regression lines of X on Y and of Yon X are given by

(1)

(2)

respectively. Substituting parameters in (1) and (2), we have


x + 0.15 = 0.084(y - 0.6), (3)

y - 0.6 = 0.265(x + 0.15). (4)


Observe that both lines pass through the point (l1x = -0.15, l1y = 0.6); more-
over, line (3) passes through (x = -0.2, y = 0) and line (4) passes through
(x = 0, y = 0.64) (see figure).

-0.2 o x
192 Solutions

(b)

E(YIX = x) = m(x), m( -1) = 0.29, m(O) = 0.78, m(l) = 0.75.


189. If X and Yare independent, then
Pij = P[X = x;]P[Y = Yj] = Piqj' i,j = 1,2, ... ,
and the nth line is (Pnql' Pnq2' ... ) = Pnq (n = 1,2, ... ). So there is only one
linearly independent row. Hence the rank r(p) of the matrix P is 1. Conversely,
if r(p) = 1, then there is a vector a = (aI' a2' ... ) so that every row 1n can be
written as
n = 1,2, ... (An = constant).
But

gives (1)

So a is a multiple of q and there are constants A: so that


n,j = 1,2, ....
Hence X and Yare independent (A: will be equal to Pn necessarily).
190. Consider the random variables X and Y with joint probability function
P[X = Xi' Y = yJ = Pij' i,j = 1,2, (1)

and marginal probability functions


P[X = x;] = Pi' i = 1,2, and P[Y = Yj] = qj' j = 1,2, (2)
respectively. We have
2 2
L L Pij =
i=1 j=1
1, (3a)

2
L Pij =
j=1
Pi' i = 1, 2, (3b)

2
L Pij =
i=1
qj' j = 1,2, (3c)

PI + P2 = 1, ql + q2 = 1. (4)
Since X and Yare supposed to be orthogonal we get
Cov(X, Y) = E(XY) - E(X)E(Y) = 0, (5)
2 2 2 2
E(XY) = L L xiYjPij, E(X) = L XiPi, E(Y) = L Yjqj·
j=1
i=1 j=1 i=1
Hence (5) is written
X1Yl(Pll - Plqd + X1Y2(P12 - Plq2) + X2Yl(P21 - P2qd + X2Y2(P22 - P2q2)
= O. (6)
Solutions 193

From (3) we have


PI2 = PI - PII' P21 = ql - PII' P22 = q2 - P12 = Pl1PI + P2· (7)
Replacing them in (6) we get
XIYI(PII - Plqd - XIYI(PII - PI + Plq2) - X2YI(PII - ql + P2ql)
+ X2Y2(PII - PI - q2 - P2q2) = O.
The above equation, because of (4), is written
(XIYI - XI Y2 - X2YI + x 2Yz)(Pl1 - Plql) = 0,
or
(XI - X2)(YI - Yz)(Pll - Plqd = 0,
and since XI "# Xz, and YI "# Yz, it follows that PII = Plql. Equations (7),
because of the last relation, are written
P12 = PI - Plql = p(l - qd = PlqZ'
P21 = ql - Plql = ql(1 - PI) = P2ql,
P22 = pz - PZI = P2 - P2ql = P2(1 - qd = pzqz,
that is, Pij = Piqj (i,j = 1,2). So X, Yare independent.

191. (a)
P(t) = (pt + q)n, M(t) = P(e t ) = (pet + qt, <p(t) = M(it) = (pe it + qt,
J1 = p'(1) = M'(O) = i-I<p'(O) = np, (Jz = npq.

(b)
M(t) = e).(e'-l),

(c)
pet pe it 1
P(t) = 1 ~t qt ' M(t) = 1
- qe
t' <p(t) = 1
- qe
it' J1 =-,
p

(d)

P(t) = C~ qty, M(t) = C! qety, <p(t) = C !qeity, J1 = p' (J


2 rq
=2.
P
192. The characteristic function of X is

<p(t) = -1 foo eitx


--2 dx = e- 1tl •
TC -00 1+X
Hence
194 Solutions

that is, (6.11) is only a necessary but not sufficient condition for the inde-
pendence of two random variables.
193. Py(t) = E(t Y ) = E(t 3X + 2) = t 2E(t 3X ) = t 2PX (t 3 ).
194. Var(SN) = PsN (1) + PsN (1) - PS)1)2, (1)
where
(see (6.4»,
psJt) = P5c(t)P;'(Px(t»,
PSN(t) = P;(t)P;'(Px(t» + P5c(tf P~(Px(t».
Hence we have
PSN(t) = E[SNJ = P5c(1)P;'(Px (1)) = P5c(1)P;'(1) = E(X)E(N),
PsN(1) = PZ(1)P;'(1) + [P'(1)J2 P~(l)
= E[X(X - 1)JE(N) + E(N2)E[X(X - 1)].
Replacing in (1) we have the result.
195. The number of the blond children is

The number of children N is geometric with parameter Pl = 1/2. The X/s are
independent Bernoulli with parameter p. Because of (6.4) and Exercise 191 we
have as the probability genrating function of SN

Ps (t) = Pl (pt + q) at +b
N 1 - q 1 (pt + q) 1 - et'
where a = P1P2/(1 - qql), b = qql/(1 - qql), e = qlP/(1 - qql)' The prob-
ability Pk of k blond children is given by the coefficient of t k in the expansion
of Ps)t). This gives
Po = b, Pk = ek-1(a + be), k = 1,2, ....
This is a geometric distribution with modified first term.
196. Let

where
N
Xjk = L
j=l
Xjk

and, for every k = 1,2, ... , N, P[Xjk = 1J = Pj = 1 - P[Xjk = OJ (j = 1,2,3).


The probability generating function of ~k = (X lk , X 2k , X 3k ) for k = 1, 2, ... ,
Solutions 195

N, is
k = 1,2, ... , N.

The random variable N is, by assumption, Poisson with parameter A = 150.


It can be shown that the probability generating function of X = (Xl' X 2 , X 3 ) =
~:J=1 ek· Similarly the probability generating function of SN (cf. (6.4» is given by

PX (t1' t 2 , t 3 ) = PN (Pk (t 1 , t 2 , t 3 »= e).(Pk-l)

= e).(P,t, +P2t 2+P,t'+P4- 1 ) = fl3 e).p;(t;-l).


i=l

This is a product of Poisson probability generating functions with parameters


APi· Then Xl' X 2, X 3 are independent Poisson random variables with parame-
ters AP1' AP2' AP3' respectively. This can be directly shown as follows:

197. We have

where

f x,(u) = a - a cos(ula)
2 '
fx 2 (u) =
b - b cos(ulb)
2 .
nu nu

We have

()
CPyt--
_ e foo e itu 1 - cos(ule)
2
d _
u-
{l- eltl, It I :$; lie,
n -00 u 0 for It I > lie,
and

CPx,
(t) = {I -
0,
altl, It I :$; lla,
It I > lla, CPx 2
(t)={I-b lt l,l t l:$;l /b,
0, Itl > lib;
hence, for 2e = a +b
CPx(t) = cPy(t)
Note. The density

f( ) = I - cos (AU) <u<


u 1
II.nu 2 ' -00 00,
196 Solutions

has characteristic function

cp(t) = {I -
0,
Itl/A, It I ~ A,
It I > A.

n
198.
E(Sn) =I E(Xt) = n(1 + 11 2 ),
i=1
(1)
n
Var(Sn) = I Var(xt) = n[E(X:) - E(Xt)2].
k=l

But if Z - N(O, 1), then X k = Z + 11 and


E(X:) = E(Z + 11)4 = E(Z4) + 4IlE(Z3) + 611 2E(Z2) + 611 3E(Z) + 114
(2)

since
E(Zk) = i-kcpt)(O) = {O for k = 2m + 1,
1,3, ... ,(2m-l) fork=2m(m= 1,2, ... ),

where

From (1) and (2) we have


Var(Sn) = n[(3 + 611 2 + 114) - (1 + 1l 2)2J = 2n(1 + 211 2).
On the other hand, we have
E(X~) = v, E(T) = av, Var(T) = 2a 2 v.
Setting
Var(T) = Var(Sn)
we have

199. By an extension of the theorem of total probability for the expected


values, we have

I
00

CPs/t) = E(e itSN ) = E[eitSNIN = nJP[N = nJ


n=O

=I E(eitSn)P[N = nJ = I cpx;(t)P[N = nJ = PN(CPxJt»,


n

because for given N = n the characteristic function of Sn = Xl + '" + Xn is


cpx;(t).
200. The probability generating function of a Poisson distribution is
(1)
Solutions 197

the characteristic function of the Cauchy distribution with density


1 1
f(y) = ~l + y2

IS

(2)

So the characteristic function distribution of SN'


SN = Xl + X 2 + ... + X N,
where Xj(j = 1,2, ... , N), are independent and identically distributed Cauchy
random variables and N is Poisson independent from Xj' is
CPs)t) = P(cp(t)) = e).(e- 1tl -1).

The density of SN is

=-
e-). foo ( L (Ae-t)n)
--,- cos tx dx = -e-). L ,An foo cos txe- nt dt
00 00

n 0 n=O n. n n=O n. 0

=-
e-).f 1 An
costxdx+-e-). L - 2
OO 00 n 2
non n=l n! n +X
1
=-e-).
foo cosxtdt+-e-).
1
L,
00 An
2
n

nOn n=l n. n + X
We observe that this is a mixture of Cauchy distributions with weights w",
i.e., of the form
Ln w"f,,(x),
where
,n
_).11,
w" = e -, n = 1,2, ....
n!
The f,,(x) represent Cauchy densities
1 n
f,,(x) = - 2 + 2' -00 < X < 00, n = 1,2, ... ,
n n X

fo(x) = -1 foo cos tx dt.


n 0

201. We have (see Exercise 191), putting p = Aln,

cpzJt) = (pe't. + q)n = [A(e


1+
it - 1)Jn -+ e).(e it -1),
n n+oo
198 Solutions

that is, the characteristic function of Poisson with parameter A. According to


the continuity theorem for characteristic functions, the FzJx) -+ Fz(x). Hence
the approximation ofa binomial by Poisson, for p -+ 0, n -+ 00, so that np -+ A,
IS

k = 0,1,2, ....

202. We have

(A - it)'

For A = 1/2, S = n/2, the Gamma distribution gives x;. Hence


q>x~(t) = (1 - 2itr n / 2.

203. Show that the characteristic function q>yJt) of Y",


n k n sin(t/k)
q>yJt) = Il cos(t/2 ) = Il 2 sm. (/2k+1)'
k=1 tk=1

tends (as n -+ 00) to sin t/t.

204. q>x(t) = roooo eitX[A dF I (x) + (1 - A) dF2(x)]

= Aq>1(t) + (1 - A)q>2(t),
where q>j is the characteristic function of Fj (j
have
= 1,2). For Fj - N(llj' an we
j = 1,2,
hence
q>x(t) = }, exp(ill1 t - !aft 2) + (1 - A) exp(i1l2t - !ait 2),
E(X) = i-Iq>~(O) = AIl1 + (1 - A)1l2'
Var(X) = - q>;(0) + [q>~(0)]2 = Aaf + (1 - A)ai.
205. The exponetial is a Gamma distribution (see Exercise 202) with pa-
rameters S = 2 and A = 9. Hence

q>(t) = (1 - ~'t)-l
and the generating function of cumulants 'P(t) is given by
it) it (it)2 (it)'
'P(t) = log q>(t) = -log ( 1 - 9 = 9 + 292 + ... + r9' + .. '.
Solutions 199

Then the cumulant of order r is


(r - 1)!
K =---
, 8'
206. By the definition of Y we have
P[Y = (2k + 1)h/2] = e- kh .9[1 - e-.9h]
and hence the probability generating function of Y is

Py(t) = E(t Y ) =
k=O
f t(2k+l)h/2 e -kh.9[1 - e-.9h]

= t h/2(1 _ e-.9h)/(1 _ t he-.9h).

From the above relation we have

Py(1) = E(Y) = ~ + [~h coth Gh8) - ~] > ~ = E(X)


because of 0 < tanh{!8h) < t8h and

Var(Y) = ;2 - ;2 [1 - G82h2/ sinh2G8h)) ] < ;2 = Var(X)

because of sin(h8/2) > t8h.


207. We observe that by the orthogonal transformation
t* = t cos 8 + u sin 8, u* = t sin 8 - u cos 8,
(1)
x* = x cos 8 + y sin 8, y* = x sin 8 - y cos 8,

t: f:
with Jacobian -1 we have

cp(t, u) = e i(IX+UY)g(x 2 + y2) dx dy

= f-: fX '
oo
e i(/*x"+U*Y*)g(x*2 + y*2) dx* dy*.
That is, cp(t, u) is invariant under the orthogonal transformation (1) and so it
must be a function of the invariant (function) of the transformation, i.e., of the
length Jt 2 + u 2 of(t, u). Then we deduce

cp(t, u) = cp*(t 2 + u 2). (2)

For independent random variable(s) X and Y with density g(X2 + y2) we have
CPx,y(t, u) = CPl(t)CP2(U).
Because of (2) we have
(3)

where CPl' CP2 are the characteristic function(s) of X, Y, respectively. We can


200 Solutions

write relation (3) in the form


cp(r) = CP1 (t) CP2 (u), (4)

then putting t = 0 we deduce that cP = CP2 and putting u = 0 we deduce that


cP = CPl' Hence
CP1 (t) = cpt(t 2),
So cP satisfies the functional equation
cp(t 2 + v2) = cp(t 2)cp(U 2)
the only solution of which (see, e.g., Feller, 1957) is cp(x) = efJx • Since cP is a
characteristic function, Icp(t) I :::;; 1, and hence {3 < O. So for some (12 > 0 we
have
cp(t2) = e-a2t2/2,

that is the characteristic function ofthe normal N(O, (12).


208. Let

The characteristic functions of U and V are

CPu(t) = ( t2)2n' cPy(t) = ( t 2 )2n.


1 +- 1 +-
n n
Hence U, V are identically distributed, having the same characteristic function
for every t. The joint characteristic function of U and V is

CP(t1' t 2) = E exp[i(t 1 U + t2 V)] t


= E exp [ i ( _1_ _
+tL
2
n Xj + - _
t_1_ t2 L lj
n )]

n n

yrTI
j=l j=l

= [1 + C1: t2 + C1: t2Yrn.


We observe that
CP(tl' t 2)"# CP(t1' O)cp(O, t 2) = CPU(t1)CPy(t2)'
So U, V are not independent. It is easily shown that Cov(U, V) = 0, that is,
U and V are uncorrelated. This can also be shown from the fact that the
coefficient of t 1 t 2 in the expansion of log cp(t 1, t 2 ) is O.
209. (a) Let Xl' X 2 be independent and identically distributed random
variables with characteristic function cp(t). Then the characteristic function of
Xl - X 2 is cp(t)qJ(t) = Icp(t)12.
(b) Let Xl' X 2 , ••• , Xn be independent and identically distributed random
Solutions 201

variables with characteristic function q>(t) and let N be Poisson with parameter
)., independent ofthe X/so Then, from Exercise 199, the characteristic function
of the sum

210. Similarly, as in Exercise 196, the multinomial random variable X is


written
k
X = (Xl> X 2, ... , X k) = L ~j'
j=l
where the probability generating function of ~j is
Pj(t 1t 2, ... , td = P1 t 1 + P2 t2 + ... + Pktk + Pk+1·
Hence the probability generating function of X is
P(t1' t 2, ... , t k) = (P1 t 1 + P2 t 2 + ... + Pktk + Pk+1)".
From the above relation we have

j = 1,2, ... , k,

E(XjXs) = - - p I a2 ,
atj ats /,=12=---=1.=1

a2 PI
E[Xj(Xj - 1)] = at] /,=1 2=---=1.=1'
j = 1,2, ... , k,

hence the assertion is proved.


211. Putting ).)n instead of Pj into P(t1' ... , t k) of Exercise 210 we have

P(t 1, ... ,tk)=


(1+ ±k(t- _ 1»)n
j =1 J
n
J ----:::;-+ex p
n 00
[f A
j=l
j(tj -1)]=rl e ).j(lrl),
j=l

and hence the assertion is proved.


212. We have

By virtue of (2.13), we find


202 Solutions

213. The conditional characteristic function of X. given Y = y. is

({Jx(t 11Y = y) = E[eil,XI Y = y] = f'' oo ell,Xfx(xl Y = y) dx.


while the joint characteristic function of X and Y is
({J(t 1• t 2) = E[e i(I,X+r2Y)] = E[E(e i(I,X+12 Y)1 Y)]
= E[eiI2YE(ell,xIY)] = E[eiI2Y({Jx(t1IY)]

= f~oo eiI2'({Jx(t 1IY=y)fy(y)dy.


By the inversion formula we have

(1)

and since

the result follows.


214. The joint characteristic function of Y, Z is
({J*(t 1• t 2) = E[exp(i(t1 Y + t2Z))] = E[exp(it1(S1 + S3) + it 2(S2 + S3))]
= E exp(i[t 1S1 + (t 1 + t 2)S3 + t 2S2])
= [({J(tdJ"'-"[({J(t 1 + t 2)]"[({J(t 2)]"2-",

where ((J(t) is the characteristic function of X. By virtue of (1) of Exercise 213.


we have
1
E(YIZ = z)fz(z) = -2 foo e- 1I2. [i- 1-a({J*l
%
a- dt 2• (1)
1t -00 t1 ,=0

(2)

Thus we find
n
E(YIZ = z) = (n1 - n)E(X) + -z.
n2
This can be found more easily. directly (see Exercise 320)
E(S1 + S31S2 + S3 = z) = E(Sd + E(S3IS2 + S3 = z) = (n1 - n)E(X) + n/z.
Solutions 203

215. If q>(t) is the characteristic function, it must be shown that for every
n E N there is a characteristic function q>n(t) such that

This is satisfied by the Pascal, Cauchy, and Gamma distributions as directly


deduced from the corresponding characteristic functions in Exercises 191 and
202. For the Laplace distribution, we observe that the characteristic function
is
r
q>(t) = (1 + t 2 1 = [(1 + t2)-1/n]n,
where (1 + t 2 )-1/n, as implied by Exercise 202, is the characteristic function of
the difference of two independent and identically distributed Gamma variables
with parameters A. = 1 and s = l/n.
216. Let X be the length of the circumference and Y the area of the circle.
We have, because of (7.1),

1 (x)
fx(x) = 2n fR 2n = 2n(p - lX)'
1 2nlX < x < 2np,

1 fx
fy(y) = - -
2J"iY
(if) = -2J"iY- -P-- ,
-
n
1 1
lX
217. Applying (7.10) for the transformation
= x + t, Z w = x/y,
with Jacobian J(z, w) = -z/(w + If, we find the density of z, w:

z (zw z) Z 2 _A
z >0, w>O.
f(z, w) = (w + 1)2 fx,y w + 1 'w + 1 = (w + 1)2 A. e z,

Since it can be written as a product of a function of z and a function of w, it


follows that Z, Ware independent; Z is Gamma and W is F with 2 and 2
degrees of freedom.
218. X 2 - Y must be ~ O. Hence the required probability is

p[X2 - Y ~ 0] =
Jr
x2_y;;O:O
ff(X, y) dx dy =
Jr
x2;;O:y
fdX dy

Io dx IX 1
2
I
= dy =-.
0 3
Note. It is recommended that the reader show the above result by finding first
the density of

that is,
fz(z) = {Jz+l
1- Jz
for - 1 < z ~ 0,
for 0 < z < 1.
204 Solutions

219. We have () = v 2 jg sin 29 and the distribution function of () is

F(x) = P [~ sin 29 ~ x J= P [sin 29 ~ ~: J


= P [0<- -21 arc sm. -gXJ
l?
v
+ P [0 > -n2 -
2
l?
- 2
. -gXJ
-1 arc sm
v2
2 . gx
=-arcsm-.
n v2
Hence the density is

f(x) = 2g 1 v2
0< x <-.
n Jv 4 _ g2x 2 g

This is also found by using (3.2) directly.


220. (a) The density function of Z = XjYis given by

fz(z) = f: lyl./(x.Y)(Yz, y) dy

_
-
foo
2na 2'oj 1 - p2
1
r;--;:z
0
exp
[Z2 - 2pz + 1
- 2 2(1
a - p
2) y
2J 2y dy
= _1 'oj ~ foo
1 - I' e-I dt
n Z2 - + °2pz 1

-oo<z<oo.
;- (z - p)2 + (1 _ p2)'

(b) Because of symmetry, we have


P[X < 0, Y> OJ = P[X > 0, Y < 0].
Since
P[XY < OJ = P[XjY < OJ = P[X < 0, Y> OJ + P[X > 0, Y < OJ,
it follows that
P[X < 0, Y> OJ = iP[XjY < OJ = iP[Z < OJ
1 fO 1 fO J1=pz
=2 -00 fz(z) dz = 2n -00 (z _ p)2 + (1 _ p2) dz

= ;n [arc tan (p)Ioo


Solutions 205

= ~[arc tan( -p ) - (-~)J


2n J1=P2 2
1 arc sin p
4 2n
221. The Cartesian coordinates (X, Y) of a point are connected with the
polar coordinates (R, e) by the relations
X = R sin e, Y = R cos e,
The joint density of (R, e) is given by
f(R,ek, 9) = f(x,n(r sin .9, r cos 9)IJ(r, 3)1,
But

, _ 1 -r2/2u 2
f(x, n(r sm 9, r cos 9) - 2na 2 e

and IJ(r, 9)1 = r, Thus we obtain

_ 1 -r2/21<u2 • 0< r < 0:::;; 9 < 2n.


f(R,e)(r, 9) - --2 e r, 00,
2na
Hence Rand e are independent.
The marginal density fR(r) of R is

{' ( r ) =
JR f
21<
{' (0) dO~ = _r2 e -r2/2u 2 _21
J(R,8)r,~
f21<
dO~ = ~ e -r 2/2u 2
2 , 0< r < 00,
o a n 0 a
that is, the Rayleigh distribution, and e is uniform in the interval (0, 2n).
222. (a) If fy(y) is the density of the random variable Y = aX + b then
1 (y - b)
fY(Y)=~fx -a- = -l/a (a <0), a+b<y<b,
{l/a (a > 0), b< y< a + b,

that is, Y has uniform density in the interval (b, a + b) if a > 0, or in the
interval (a + b, b) if a < O.
(b) Let A > 0, B > 0; then

x = x(y) = _l!.- + JB2 + 4(y - C)A > 0 for C< y < A + B + C,


2A 2A
and thus
1
fy(y) = fx(x) J 2 )' C<y<A+B+C.
B + 4A(y - C

Similarly, for the other cases of the constants.


206 Solutions

r:
223. (a) The density of Y = XI + Xl is given by

I
fy(y) = fx 1 (y - x l )fx 2 (x l ) dX l

f: dXl = y, O::s;; y ::s;; 1,

= II y-I
dx l =I-(y-I)=2-y, 1 ::s;; y::s;; 2,

that is,
fy(y) = I - II - yl, O::s;; y::s;; 2.

I1
(b) The density of Z = X I X 2 is given by

f
-

I dX l = I + z, -I ::s;; z::s;; 0,
fz(z) = f+oo fx (z + x2)fx (Xl) dX 2 = -Z

1 2 1- Z
-00 0 dX 2 = I - z, O::s;; z ::s;; I,

that is,
fAz) = I - Izl, -1::s;;z::s;;1.
(c)

We have
FN(W) = P[W::s;; W] = P[IZI::S;; W] = P[ -W::S;; Z::s;; W] = FAw) - FA-w)
and thus
fw(w) = fz(w) + fz( -W).
Because of (*), we have
fw(w) = (l -Iwl) + (I -I-wI) = 2(1 - w), O::s;;w::s;;1.
(d) The density of V = XI/Xl is given by (see (7.7»

fv(v) = f 00

IX21fx (xlv)fx (Xl) dX2 =


{II X2 dX l =~, o ::s;; v ::s;; 1,
-00 1 2 1~V 1
Xl dX l = -1' v ~ 1.
o 2v
224. Let XI be the time of arrival of A and let Xl be the time of arrival of
B, then
fXj(x) = 1, o ::s;; Xj ::s;; I, j = 1,2,

Fxj(xj ) = Xj o ::s;; Xj ::s;; 1, j = 1,2.


Solutions 207

Given that the lunch lasts 0.5 hours, the probability p of meeting (event A) is
given by
p = P(A) = P[IX I - X2 1 ::; 0.5] = P[ -0.5 < Xl - X 2 < 0.5].
The density of Z = Xl - X 2 , from Exercise 223, is given by
fAz) = 1 - Izl, Izl ::; 1.
Hence

p= f o
-0.5
(1 + z) dz +
fO.5
°
1
(1 - z) dz = -[(I + Z)2]g.5 - -[(1 - Z)2]g.5
2 2
1

3
-
4
If T is the time of meeting then T = max(X I , X 2) and let fT(t) denote the
(unconditional) density of T Then, taking the interval [0, 1] we have
(a)
fT(t) = 1/4 if t ¢: [0, 1],
fT(t) = 2t if 0::; t ::; 1/2,
fT(t) = 1 if 1/2::; t ::; 1.

Hence, the conditional density of T given A, since P(A) = 3/4, is given by

for 0::; t ::; 1. (1)

(b) Similarly as in (l).


225. The density of the product Z = X Y is given by the formula

fz(z) = fro
-00
-I
y Y
(z)
1I fy(y)fx - dy = -12 00 -I
7W Izl Y
1
1I ye-y2/2u2 J 1 - 1z2/y 2 dy.
For the limits of integration of y, we have Izl < y < 00, because IXI < 1 and
y > O. Thus

-oo<z<+oo,

that is, the density of N(O, (J2).


208 Solutions

226. (a)
n n
Y = L
j=l
2 log Xj = L
j=l
lj

where
lj = - 2 log Xj' j = 1, 2, ... , n,
independent variables with density

fy/y) = d~j e-
fx j (e- Yj / 2) I Yj / 21 = ~ e- Yj / 2, Yj > 0.

Thus lj has the Gamma distribution with parameters A. = 1/2 and s = 1,


Exercise 203 (i.e., exponential distribution). But the Gamma distribution has
the reproductive property and hence Y, as the sum of n independent and
isonomic Gamma variables, has Gamma distribution with parameters A. = 1/2
and s = n, that is,
1
fy(y) = 2nr(n) yn-I e- Y/2, y > 0,

that is, Y has the X2 distribution with 2n degrees of freedom.


(b) The joint density of the random variables (see (7.11))
~= J -2log Xl cos 2nX2' '1 = J -2log Xl sin 2nX2
is given by

But

and hence

f(~, '1) = ~e-~2/2 ~e-~2/2, -00 < ~ < 00, -00 < '1 < 00.
y2n y2n
From the above relation, we deduce that ~, '1 are independent N(O, 1).
227. Let Y = F(X). Then
Fy(y) = P[Y:os:; y] = P[F(X) :os:; y]
O' y:os:; 0,
={ P[X:OS:; F-I(y)]
1, y;o:: 1.
= F[F-I(y)] = y, °=s; y =s; 1, (1)

If Xl' X 2, ... , Xn are pseudorandom numbers, then YI = F-I(Xd, ... , Yn =


F- 1 (Xn ) are independent, and by (1) they are isonomic (identically distributed)
with distribution function F.
Solutions 209

228. By Exercise 239 we have


P[X E b] = R,
where R has density given by (2) of Exercise 239 with n = 10. Then the required
probability is

P[R ~ 0.95] = 10 x 9 f1 0.95


r 8 (1 - r) dr = 1 - 10(0.95)9 + 9(0.95)10.

fo
229. (a)

fx(X) = fy(lOgX)I~(lOg
dx
X)I = (J 2nx
exp [-~(lOg
2(J
x - 11)2J, x >0.

(b) We have
E(X) = E[e Y ] = My(l),
Var(X) = E(X2) - [E(X)]2 = My(2) - [My(1)]2,
where My(t) = e/ll+u212/2 is the moment generating functions of the normal
random variable Y. Hence
E(X) = e/l+ u2/2 and Var(X) = e2/l+ 2u2 _ e2/l+u2.
(c) log Xi is normal; hence the sum log Xl + ... + log Xn = log([17;1 X;)
is normal. Hence it follows that Y = [17;1 Xi is lognormal.
230. From the definition of the distribution function we have
Fz(z) = P[Z :s z] = P[X :s z, Y:s z] = F(z, z),
Fw(w) = P[W:s w] = 1 - P[W > w] = 1- P[X > w, Y> w].
Using the addition theorem
P[A u B] = P(A) + P(B) - P(AB) = 1 - P(A'B')
with A = {X:s w}, B = {Y:s w}, we have
1 - P[X > w, Y> w] = P(X :s w) + P(Y:s w) - P[X :s w, Y:s w] .
= Fx(w) + Fy(w) - F(w, w).
If F is continuous, we obtain the densities of Z, W by taking the derivative of
the corresponding distribution function; thus, we have

d d
fz(z) = dz F(z, z) = dz
JZ JZ
-00 -00 f(x, y) dx dy

= foo f(x, z) dx + foo f(z, y) dy, (1)

fw(w) = fx(w) + fy(w) - J:oo f(x, w) dx - J:oo f(w, y) dy.


210 Solutions

231. Because of the independence of X, Y (see Exercise 230(1)) we have

fz(z) = 2<1>(z)q>(z),

where <1> and q> is the distribution function and the density of N(O, 1), respec-
tively. Hence

E(Z) =f OO
-00 zfz(z) dz =;1 foo -00 ze- z2/ 2 dz
fZ -00 e- x2/ 2 dx

= -1 foo e- dx foo ze- z2/ 2 dz = -1 foo e- x2 dx = -1.


In
x2/ 2

n -00 -x n -00

232. The joint density of Y1 = Xl' Y2 = Xl + X 2, ... , Yk = Xl + X2+


... + X k is given by

Since Xl = Y1,X2 = Y2 - Y1, ... ,Xk = ¥,,- ¥,,_l,we haveJ(Yl,Y2'''',Yk) =


1 and so

. (1 - Yk)"k+l-t,

Then the marginal density of ¥" is

But

= y"l +"2-1 r(n1 )r(n 2).


r(n1 + n 2 )

Thus, by successive integrations, we finally obtain

233. The joint density of Y1 , Y2 , ... , ¥", Y = Xl + X 2 + ... + X U1 ' is


given by
f1(YYdf2(YY2) .. ·.f,.(YYd.f,.+1(Y(1 - Y1 - Y2 -'" - Yk))IJ(Y1' ... , Yk, Y)I,
(1)
Solutions 211

where

Xi> 0, i = 1,2, ... , k + 1,

and since Xl = YY1' X2 = YY2' ... , X k = YYk' Xk+l = y(1 - Yl - ... - Yk)' we
have

Y ° .. . °
° °
Y .. .
J(Yl'···'YbY)=

-Y° ° -Y -y
Y Yk
(1 - Yl - ... - Yk)
Then (1) is written as

which gives the required Dirichlet density.


234. (a) The density of the random variable Z = X + Y is given by

Zk = 0, 1, 2, ... ,

that is, Poisson with parameter . 1. + Jl.


(b)

P[X = klX + Y = n] = P[X = k, Y = n - k]


P[Z = n]

=
(n) (..1.+Jl )k(..1.+Jl)"-k'
k
..1. Jl k = 0,1, ... , n,

that is, binomial with parameters nand p = ..1./(..1. + Jl).


212 Solutions

235. (a)
n
P[X + Y = n] =
k=O
L P[X = k]P[Y = n - k]

= pnqA+M-n f. (A)(
k=O k
M)
n- k

= (A+n M) p qA+M-n, n
n = 0, 1, ... , M + A.
(b)

P[X = k, Y = n - k] P[X = k]P[Y = n - k]


P[X = klX + Y = n] = P[X + Y = n] = P[X + Y = n]

236. We have
n n n n-1
(n - 1)s2 = L Xi2 - nX 2 = L }? - (JnX)2 = L 112 - y"2 = L 112, (1)
i=1 i=1 i=1 i=1

since the transformation y = Hx gives


y'y = L" yJ = x'H'Hx = x'x = L" xf.
j=1 i=1
The covariance matrix of Y = (Y1, Y2, ... , y")' is D(Y) = HD(X)H' = (12HIH'
= (121, that is, Y1, ... , Y" are independent N(O, (12). Hence Y" = JnX, X is also
independent of Y1, ... , y"-I' and therefore of S2, because of (1). Moreover, the
distribution of (n - l)s2j(12 is X~-I' The independence of X and S2 is also
deduced from the fact that
COV(Xi - X, X) = 0, i = 1,2, ... , n.
This, by the joint normality of X, Xi - X, implies their independence, that is,
X is independent of Xl - X, X 2 - X, ... , Xn - X; and hence also ofs2. For
the Cov(X, S2), taking E(X;) = 0, without any loss of generality, we have by (1)
-
Cov(X, S2) = - -1 E [_
X L Xl
n ] n
- _ _ E(X3)
_
n- 1 i=1 n- 1

=
n(n - 1)
1 E [" LX.'' i L
i=1
" x ]
=I'
2 -
n(n - 1)
1 E [nL ]
i=1
x~
'

1 1 1
= --J1.3 - ----J1.3 = -J1.3·
n- 1 n(n - 1) n
Solutions 213

237. Show that


1 m
X- Y= - L Vj =
m ;=1
V,

where
1 m
Vj = uj + - - L y; - y
fo;=1

are independent N(lll - 1l2' a*2) with


m m
and L (Uj - iif = L (v; -
;=1 ;=1
V)2.

Hence for III = 112 we have the required distribution.


238. The density of the random sample Xl' X 2, ... , Xn from a continuous
distribution with density f is

Given that the n! permutations of Xl' ... , Xn give the same ordered sample
XC!), X(2), ... , X(n), it follows that the required density is

f*(x l , x 2, ... , xn) = n! f(x l , x 2, ... , xn) = nl n f(xJ


n

;=1

239. (a) By Exercise 227, Y = min(Y1' ... , Y,,), Z = max(Yl , ... , Y,,), where
Yl , Y2 , ••• , y" constitute a random sample from the uniform in (0, 1). Working
as in Exercise 238, we find that the joint density of Y, Z is
= n(n - 1Hz - yr 2 ,
f(y, z) 0< y < z. (1)

(b) The density of R = Z - Y is easily seen to be

fR(r) = n(n - 1) f0
l-r
r n - 2 dy = n(n - l)r n - 2 (1 - r), 0< r < 1. (2)

Hence we have

E(R) = n(n - 1) fo
1 r n - l (1 _ r) dr = n - 1.
n+1
240. The joint density of X w, X(k) (j < k), is given by

f(xj, Xk) = (j _ 1)1 (k _ ;!_ 1)1 (n _ k)l [F(xj )]i- 2[F(Xk) - F(xj)]k-j-1

. [1 - F(xk)]n-kf(xj)f(x k), Xj < Xk'


because j - 1 of the X; must be less than Xj' k - j - 1 between Xj and
Xk, n - k must be larger than Xk' one in the interval (Xj' Xj + dxj) and one in
the interval (Xk' Xk + dx k) (f(x) and F(x) represent the density and the distri-
214 Solutions

bution function of the original distribution, respectively). Hence we have


E[X(k+l) - X(ka

= f~<Xl f~oo (y - x)f(x, y) dx dy

= (k _ I)! (;!_ k _ I)! {f~<Xl Fk-1(X)f(x) dx {<Xl y[1 - F(y)]n-k-If(y) dy

- f~<Xl xFk-1(X)f(x) dx {<Xl [1 - F(y)]n-k-If(y) dY }.

Integrating by parts gives the result. This can also be found from the relation
E[X(k+l) - X(k)J = E[X(k+l)J - E[X(k)J,
using the density of X(k) in Exercise 241.
241. (a) The density of X(k) is (cf. Exercise 240)

n! k-l n-k
h(x) = (k _ I)! (n _ k)! [F(x)] [1 - F(x)] f(x), (1)

where for the uniform f(x) = 1 (0 ~ x ~ 1), F(x) = x (0 ~ x ~ 1). Hence we


have
n! I k n-k k
E[X(k)J = (k _ I)! (n _ k)! x (1 - x) dx = n + 1.
(b) The density of R is

fR(r) = n(n - 1) f: f(x)f(x + r) [F(x + r) - F(x)]n-2 dx. (2)

For the uniform density we have

fR(r) = n(n - 1) r-r r n- 2 dx = n(n - l)rn-2(1 - r). (3)

(c) From (3) we find

E(R) = n(n - 1) II r n- l (1 - r) dr = n - 11.


° n+
242. Because of (1) of Exercise 241, the density of Y2 is
[) - ! < x < [) + !.
Hence the required probability is

6 f .9+0.4
(x - [) + !)([) - x + !) dx = 6 1°·9 y(1 - y) dy = 0.944.
.9-0.4 0.1
Solutions 215

243. The joint density of Y1, Y2, Y3, by Exercise 238, is


g(Y1' Y2, Y3) = 3! (2yd(2Y2)(2Y3) = 48Y1Y2Y3'
According to (7.11) the density of Zl' Z2, Z3 is
f(Zl, Z2, Z3) = g(Y1, Y2, YJ)IJ(Zl' Z2, z3)1 = 48z1Z~Z~,

because

So we have the independence of Zl, Z2' Z3'


244. Show that the density of Y = J Xl X 2 is given by

fy(Y) = 2y f co
-co
1 (y2)
~ fX t ~ fx 2 (x) dx

= cy2nt J[1y2 X- 3/2 [( 1 - ~


y2) (1 - x) Jn -1 dx.
2

245. Direct application of(7.11)withz = gl(X, y) = Y - x, w = g2(X, y) = x.


Z is Gamma with parameter n and it is independent of X.
246. Apply Exercise 244.
247. Application of (7.6). Y is Gamma with parameter m + n.
248. Setting X = R cos B, Y = R sin B, we have
Z = R sin 2B, W= R cos 2B, (1)
where, by Exercise 221, Rand B are independent with densities
1
f(B) = - , 0 < B < 2n.
2n
Now the joint density of Z, W can easily be found to be

f(z, w) =
1 (Z2 +
2n0'2 exp - 20'2
W2) '
which shows the independence of Z and W
249. By applying (7.12) to (5.10) we find that Y is N(AJ1., ALA').
250. Let (Xi' 1;) be the impact point of Ai (i = 1,2). The required probabil-
ity is equal to

P[R 1 < R 2] = P[R 21 < 2


R2 ] = lCO F1 (r)f2(r) dr = 1- 2
O't 2'
o 0'1 + 0'2
216 Solutions

since Rf = X? + Y?, by Exercise 221, has density

and distribution function

Fi(r) = 1 - ex p ( -~ ;2). i = 1,2.

251. Apply (8.9)


252. Follows from (8.5) and

Var(Xn ) = :2 t~ Var(X;) + :t: COV(Xk' Xk+d} 2

1
n {nM + 2(n - I)M} ---+0.
:::; -2
n-X)

253. Follows from

L Var(X
n
Var(Xn ) < n- 2 i ) :::; n- 1 c -+ 0 as n -+ 00.
i=l

254. Show that


Var(Xn ) -+ 0 as n -+ 00.

255. We will show that for every continuity point x of Fx


lim P[Xn + Y" :::; xJ = Fx(x - c). (1)

We have
P[Xn + Y" :::; x] = P[Xn + Y" :::; x, I Y" - ci :::; 8]
+ P[Xn + Y" :::; x, I Y" - ci > 8J,
where
P[Xn + Y" :::; x, I Y" - cl > 8J :::; P[I Y" - cl > 8] -+ 0 as n -+ 00,

and the probability tends to F(x - c).


P[Xn + Y" :::; x, I Y" - cl :::; 8]
Similarly, one can show (b) and (c).
256. Let
k k
Ln = L CjXjn
j=l
and L = L
j=l
CjXj.

Then
CfJLJU) = E(e iuLn ) = CfJ~JUC l' UC 2 , •.. , UC k ), (1)

CfJL(U) = E(e iUL ) = CfJ~(UC1' UC 2 , ..• , UC k )· (2)


Solutions 217

Since

we have

that is,
qJ~JCi' C2 , · · · , Ck ) -> qJ~(Ci' c2 , · · · , cd
for every (C l' ... , ck ). Hence by the continuity theorem of characteristic func-
tions for multivariate distributions

en -> e·
L

257. From P[IXkl ::; MJ = 1 it follows that X k - E(Xd are uniformly


bounded and since s; -> 00 for every a > 0, there is an N such that for n > N
the relation
P[IXk - E(Xdl < aSn' k = 1,2, ... , nJ = 1
holds. Hence the condition of Lindeberg- Feller's theorem is satisfied and the
CLT holds.
258. If Xn denotes a Poisson random variable with parameter ), = n=
E(Xn), then we have

e- n f n~k.
k=i
= P[Xn ::; nJ = p[Xn
v
~ n ::; 0] -> <1>(0) = ~
n 2
as n -> 00

by the normal approximation to the Poisson.


259. We have
X!. Ai' Y!. A2 , X + Y ~ Ai + A2 ,
X + Y ~ N()'i + ..1 2 , Ai + A2 )·
The result follows by application of Exercise 255(c) with C = JA i + A2 •
260. Verify that the Lindeberg-Feller condition is satisfied.
261. According to the CLT
P[ -1 < S: < 1J -> <1>(1) - <1>( -1) = 0.68,
while Chebyshev's inequality gives no information since
P[ISnl ~ c5J ::; c5- 2 = 1 for c5 = 1.
For k = 2, Chebyshev's inequality gives
P[ -2 < S: < 2J ~ 0.75,
while the CLT gives as a limit <1>(2) - <1>( - 2) = 0.9544. Similarly, for k = 3,
we have 0.8888 and 0.9974, respectively.
218 Solutions

262. 11 = E(X) = Loo x24x- 4 dx = 24 Loo x- 3 dx = 3,

q(<5) = P[IX - Ill> <5] = P[IX - 31 > <5]


= 1 - P[IX - 31 < <5]

= 1 - P[3 - <5 < X < <5 + 3] = 1- f3H 24x- 4dx


3-~

= 1 - 8 [(3 ~ <5)3 - (3 : <5)3]

8(3 + <5)-3, <5 ;?: 1,


{
= 1 - 8[(3 - <5)-3 - (3 + <5)-3], <5 < 1.

According to Chebyshev's inequality

Var(X) 3
P[IX - III > <5]:5: <52 = <5 2 '

The following table and figure show the difference between q(<5) and 3<5- 2 •
Note that Chebyshev's inequality for <5 :5: a = J3
gives the obvious upper
bound 1.

y
y = 1

By
q(<5) Chebyshev
1/2 0.673 1
1 0.125 1
2 0.064 0.750
3 0.037 0.333
5 0.016 0.120

263. Applying the CLT we find that n will satisfy the relation

(a) P[IX - III > 0.05a] ~ P[IZI > 0.05Jn] < 0.05.

Hence 0.05Jn > 1.96 (Z '" N(O, 1)). Thus n ;?: 1537
Solutions 219

(b) P[IX -Ill> 0.0511]::; p[IX -); > 0.05~5J


a/yn a/ n

~ p [ IZI
0.25 y r:.J
> -3- n .

Hence In
> 196· (3/25) and n > 553.
According to Chebyshev's inequality n must satisfy the relation
- 1
(a) P[IX - Ill> 0.5a] ::; 2 < 0.05 => n > 20 3 ,
(0.05) n
_ a2 9
(b) P[IX -Ill > 0.0511] ::; n(0.OWIl2 < (0.25)2n

< 0.05 => n > 2880.


264. (i) According to the De Moivre- Laplace theorem and putting Z '"
N(O, 1), the required size n of the sample will satisfy the relation

from which
n> 196 2 pq.
Then we have (a) n > 196 2 x 0.3 x 0.7 or n > 8067. (b) The maximum value
of pq is attained for p = 0.5. Hence n > 196 2 x 0.25 or n > 9604.
(ii) Working as in (i), we find (a) n > 323, (b) n > 384.
265. According to the Moivre-Laplace theorem and putting Z '" N(O, 1),
the number of seats k must satisfy the relation
k - 300 x 1/3 J 1
P[S300 > k] ~ P[Z > < -,
)300 x 1/3 x 2/3 20
k - 100
fiN\ > l.645 or k ~ 112.
y 300/3
266. If the X;'s are independent uniform random variables in ( - 0.05, 0.05),
we have, according to the CLT with II = 0, Var(XJ = 0.01/12,

p[lli~O Xi I < 2J ~ P[IZI < 0.!1 J = P[IZI < 2.19] = 2<1>(2.19) - 1

= 0.97.

267. As in Exercise 266 we have, since E(XJ = 5, Var(XJ = 90 2 /12,

p[Ii=l Xi> 500J ~ p[Z > 220


200J ~ 1 - <1>(0.91) = 0.18.
220 Solutions

(b) c must satisfy the relation

P[ Z <
c - 300
201.6
J 1
~ 10 =>
c - 300
201.6 = -1.28 => c = 42.

268. The profit Xi of the casino during the game i, is a two-valued random
variable Xi with E(X;) = - 1/37. Thus

P[ L
n

k=l
Xi ~ 1,000 ~ P Z ~
J [ 1,000-nI 37J
In
n(1
~ -,21
where Z is N(O, 1), and hence
n
1,000 - 37 ~ ° => n ~ 37,000,

P[ofa loss] = p[37fOO Xi <i=l


oJ ~ p[Z < - ~J ~ 0.
37,000
269. Use Exercise 215(d) and the CLT, or show directly that the charac-

r r
teristic function of the standardized Gamma tends to exp( - t 2 /2).

270. E[g(X)] = J:oo g(x) dF(x) ~ g(x) dF(x) ~c dF(X)

= cP[X E (a, b)].

271. Take as g of Exercise 270, g(x) = (x + C)2 with c > 0. If E(X) = and
E(X2) = (12, then the minimum of (t + C)-2. E(X + C)2 is obtained when
°
c= (12/ t . Then we have the required inequality.
272. From the event {IX I - X21 ~ t} we have that at least one of
{IXll > ~ > t}, {IX21 > ~t} occurs. Thus we have
P[IX 1 - X 21 ~ t] ~ P[IX11 > 1-t or IX2 1 ~ 1-t] ~ 2P[IX 11> 1-t].
273. See Exercise 271.
274. If the function g(y) is decreasing for y > 0, then for;' 2 > ° we have

).
;'2 foo
9 0
g(y) dy ~ 4
_ foo y2g(y) dy. (1)

(i) In the special case when g is constant

we have
g(y) = {A
° °< for y < c,
for y ~ c,

;'2 J: g(y) dy = A;'2(c - ;').

This is maximized for;' = (2/3)c, as we can easily see by differentiation.


Solutions 221

(ii) We define the function

hey) = {go(A-) = const., 0 < y < A- + a,


y~ll+a,

where ageA-) = fT g(y) dy. Then we have

Let f(x) be the density of the continuous random variable X. Since Xo is the
only mode (maximum) of f, it is decreasing for x > xo. Hence taking

g(IX*I) = rf(x o + rIX*I), with X* = X - Xo


r
we have by (1) that

275. Application of Markov's inequality and of the relation


P[IXI ~ e] = P[g(X) ~ gee)].
276. E[g(X)] = E[g(x)IIXI ::; e]P[IXI ::; e]
+ E[g(X)IIXI > e]P[IXI > e]
::; g(e)P[IXI ::; e] + MP[IXI > e]
::; gee) + MP[IXI > e].
277. Since P[g(IXI) ::; geM)] = 1, then from Exercise 276 we have
E[g(X)IIXI > e] ::; geM).

278. p[max(IX1; 1L11, IX2~ 1L21) < e]


= p[IXl -1L11 <
al
e, IX 2 -1L21 <
a2
e]
= P[(x l , x 2 ) E R],

where R is the rectangle defined by the lines Xi = lLi ± ea i (i = 1,2). The


function

g(X ,Xl 2,t) = e 0 ~t2)[(Xl;lLly _2t(Xl;1L1)(X2~1L2)


2

for It I < 1,
222 Solutions

is nonnegative and larger than 1 for (Xl' ¢ R. We then have,

t
X2)

P[(Xl' X 2 ) E R] = f I(x l , x 2 ) dX l dX2

= 1 _ 2(1 - tp)
c 2 (1 - t 2 ) ,
It I < 1.

The expression [2(1 - tp)]/[c 2 (1 - t 2 )] is minimized when


1
t = -(1 - j1=?)
p
and thus

p[IXl - {Ill < c, IX 2 - {l21 < c] ~ 1 _ 1 - ~,


~ ~ c
from which the inequality follows.
279. For every vector t, the function
g/(X) = t'G(X)t

is a convex scalar function of the matrix X and by Jensen's inequality we have


g/(EX) :::;; E[g/(X)],

that is, for every t,


t'G(EX)t' :::;; Et'G(X)t.

Hence the relation


G(EX) :::;; E[G(X)]

follows.
280. By Exercise 279, it suffices to show that the matrix function G(X) =
X- l is convex. For this it must be shown that for symmetric matrices X > 0,
Y> 0, of order r (say) and 0:::;; A :::;; 1, the relation
[AX + (1 - A) y]-l :::;; AX- l + (1 - A) y-l (1 )
holds. There is a nonsingular matrix A such that X = AA', Y = ADA', where
D a diagonal matrix with elements d l ,... , dr. Hence (1) is equivalent to

[AI + (1 - A)Dr l :::;; AI + (1 - A)D-l,


which is satisfied by diagonal matrices; the assertion now follows from the
convexity of the scalar function y = x- l .
Note. For 0< A < 1, (1) holds with strict inequality.
Solutions 223

°
281. From (d 2jdy2) log y = _ljy2 < it follows that g(y) = log y is a
concave function; therefore, by Jensen's inequality,
E[log YJ ~ log E(Y).

282. If we consider the random variable Y,. = I7=11og Xi where Xl'


Xl' ... , Xn are independent and isonomic, it follows that E(Y,.) = nE[log Xa
and Var(Y,.) = n Var(log XJ. By Chebyshev's inequality, for every 6> 0,
Var( Y,.)
P[I Yn - E(y")1 < n6J ~ 1 - 22 '
n 6
or
al
L log Xi < n[E(log X;) + 6JJ ~ 1 -
n
P[n(E(log X;) - 6)J < -2'
i=l n6

or
P[exp{ n(E(log X;) - 6)}J < Xl X 2 .•• X. < exp{ n(E(log X;) + 6)}

Hence, and since by Exercise 281 E[log XJ ~ log E(X), we obtain the re-
quired one-sided inequality.
283. The area of the triangle is E = -!X Y, where X and Yare independent
and uniform in the intervals (0, a) and (0, b), respectively. (a and b denote the
length and width of the rectangle, respectively). The density of E is
2 2
f(x) = ab log(ab) + ab log 2x, 0< x < -!ab.

Hence calculate the required probability.


284. Let A be the origin of the axes and IX the abscissa of the point B. Then
the randomly selected points Xl and X 2 have the uniform distribution in (0, IX)
and the density of the ordered sample X < Y (say) is
2
f(x, y) = 2 ' 0< x < y < IX.
IX

In order that X, Y be such that the segments (AX), (Xy), (YB) form a triangle,
they must satisfy
IX IX
2X < IX, Y - X <- and Y> _.
2 2'
these define the triangle (CDE) (see figure) Hence
2 1
P[(X, Y) E (CDE)] = IXl X area of (CDE) = 4.
224 Solutions

~--------~------~L---.X
al2 a

285. The first point PI can be selected as any point of the circumference
and may be regarded as the origin of measuring the angles which define the
positions of the other points on the circumference (see figure). For every 8, the
probability (elementary) that 82 , which defines the position of P2 , is in the
interval (0,8), and that 83 (for the third point P3 ) is in the interval (8,8 + d8),
is equal to
8 d8 I
2n 2n = 4n2 8 d8.

Hence the required probability, say P3' will be

P3 = 3! I"-I8 d8
o 4n 2
= _3 8 2
4n 2
I"
0
3
=-.
4
The factor 3! is needed because there are 3! different permutations ofthe three
points PI' P2 , P3 •

286. Let P be the required probability and let (AB) be the given line
segment. Then
(I)
Solutions 225

where Ai is the event that the ith part is larger than 1/2. Since evidently
AiAj = 0 (i #- j), we obtain from (1)

5
P ~ 1 - i~ P(AJ = 1 - 5
(1)4
2" '

because the probability that any part is larger than 1/2 is equal to the
probability that each of the four points is contained in a subset of AB with
length 1/2, that is,

287. As in Exercise 284, we find that the points X, Y must satisfy the
relations
a+b a+b a+b
X<-- X+ Y>-2- and Y<-2-'
2 '

which define the triangle 0ZH (shaded area). Hence

area of (ZH0) b
P [(X ,Y) E (0
-HZ )] = - - - - -
area of (OrE,1) 2a'

a+b

a+b
-2-k-----~

o~------~~~----~~--x
a+b a a+b
2

288. Let X be the distance of the nearest line to the middle M of the needle
AB and qJ the formed angle. A necessary and sufficient condition for the needle
to cross the line is
X:::;; Jl sin qJ

and the required probability p, in view of the uniform distribution of X in


226 Solutions

(0, a) and cp in (0, n), is

P = -1
an
I" 0
Jl sin cp dcp = -.
2Jl
an

2a
A

289. At first we consider a convex polygon A 1 A2 ... An (with diameter less


than 2a). Then a given line (of the parallel ones) can be crossed either by none
of the sides of the polygon or by two sides only. The required probability P is
given by
P = L Pij' i, j = 1,2, ... , n, (1)
i<j

where Pij = P (of crossing sides i and j). On the other hand, the probability Pi
(say) of being crossed by the side i, by Exercise 288, is given by
Jli
Pi = L Pij=-'
i#j na
(2)

where Jli the length of side i (i = 1,2, ... , n). The probability p, by (1) and (2), is
n

lin i~ Jli P
P= L Pij = -2 L Pij = -2 L Pi = -2na- =
i<j i#j i=l
-,
2na
(3)

where P is the perimeter of the polygon. Observing that P is independent of


the number n of the sides of the convex polygon and depends only on its
perimeter, going to the limit when n -+ 00 and max Jli -+ 0, we see that the
required probability for a convex and closed curve is given by (3), where P is
the perimeter (length) of the curve.
290. Let an and bn be the expected numbers of lO-drachma coins of A and
B, respectively, after n exchanges. Then
an + bn = a + b and ao = a. (1)
Moreover, the difference equation
(2)
holds, where En is the event that A gets a lO-drachma coin the nth exchange.
Solutions 227

Since
bn - 1
PEEn] = -b- = 1 - P[E~],
relation (2) becomes

n = 1,2, ....

Taking into account (1), we obtain its general solution

a =~+~(l_~_~)n
n a+b a+b a b '
n= 1,2, ....
Hence the required probability Pn (say) that A gives a 10-drachma coin to B

r
after n exchanges is

Pn =~ =a: b +a ! b( 1- ~- ~
291. (a) The required probability Pn (say) that the game finishes with an
ace after no more than n tosses is

where
Pa = P(of ace) = 1/6, Pb = P(of six) = 1/6,
and
Po = P(neither ace nor six) = 2/3.
Relation (1) is found from the fact that the event Ak that the game finishes
after k tosses with an ace occurs if: (i) One ace appears on the first k - 1
tosses, neither ace nor six on k - 2 tosses, and ace on the kth toss. (ii) One ace
and one six appear on the first k - 1 tosses (hence neither ace nor six during
k - 3 tosses), and a six appears on the kth toss.
(b) The probability, P (say), that the game finishes with ace is the limit of
Pn as n -+ 00. Thus from (1) we find
p;(1 - Po + 2Pb) 1
P= (1 - Po)3 2
Similarly, the probability that the game terminates with six is 1/2 and therefore
the game terminates with probability 1. For fixed n there is a positive probabi-
lity 1 - 2Pn that the game continues after n tosses.
292. We have
Pn = (1 - Pn-l )P(of selection of A in the nth year) = t(1 - Pn-l)
228 Solutions

with PI = 1. The general solution of the difference equation


Pn + !Pn-I =!
is
n = 1,2, ....
293. Pn satisfies the difference equation of order 2
Pn - Pn-I + PQPn-2 = 0, n 2 2, Po = PI = 1.
(i) For P =I Q it has the solution

n 2 O.

(ii) For P = Q = 1/2 we have


Pn = (n + l)r n, n 2 O.
294. (a) We have
Yk+1 = Yk + 1 . P(the k + 1 ball is placed in an empty cell)

= Yk + n ~ Yk = (1 - ~) Yk + 1.
The resulting difference equation

Yk+1 - (1-~)Yk = 1

has the general solution

Yk=n+C(I-~y
where c = - n since YI = 1.
if cell i is occupied,
(b) where X., = {I0
if cell i is empty.
Then

E(XJ = 1- (1 - ~y, i = 1,2, ... , n,

and thus we find

295. Let A be the event of a double six in a toss of two dice. Then we have
35 1
Pn = Pn-I' P[A C
] + (l - Pn-d' peA] = Pn-I' 36 + (1 - Pn-I) 36'
Solutions 229

Hence the required difference equation, with particular solution Pn = a =


1/2, and general solution of the corresponding homogeneous Pn = c·
(17/18t. Thus the general solution is Pn = 1/2 + c(17/18)n, where from the
initial condition Po = 1 we find that c = 1/2. Hence

Pn = ~[1 + G~)"l n = 0,1,2, ....

296. Let Pk be the probability that a white ball is drawn bearing the number
k (k = 1,2, ... , n). Then Pn satisfies the relation
A+l A
Pn = A + M + 1 Pn-l + A + M + 1(1 - Pn-l)

1 A
= A + M + 1 Pn-l + A + M + 1
with initial condition
(1)

We find
A 1
Pn = A + M + c (A + M + It'
where, in view of (1), c = 0. Hence
A
Pn = A + M' n = 1,2, ....

Remark. If we suppose that the ball transferred first is white, then


A M A
Pl = 1 and P
n
=
A+ M
+ (A + M)(A + M + It ---+.
1 n+co A+ M
297. Let Pn be the required probability. Then it will satisfy the relation
Pn = PPn-l + p'(l - Pn-d, Pl = P (n=2,3, ... ).
Its general solution is

Pn=--+-~
pi q ( P-P ')n .
q + pi q + pi
For q = 1 - P = pi it gives
1 1 1
Pn = 2 + 2(p - q)n -+ 2 as n -+ 00.

298. We have
Pn = Pn-l P(of one ace) + Pn-2P(of double ace)
10 1
= Pn-l 36 + Pn-2 36'
230 Solutions

with initial conditions Po = 25/36, PI = 10/36 x 25/360. Solving this homo-


geneous difference equation we find Pn' For the probability, say qn' that one
obtains n points, when the game is not interrupted, we have the difference
equation

Compare Pn and qn'


299. Let An be the expected number of good tubes obtained after n trials
and Pn the required probability. Then
An+1 = An + 1· P(of drawing a bad tube during the (n + 1) draw)

= An+1 (1 - ~n b)
a

and Ao = a. Hence we find

and

300. Define success (S) on the rth draw of a ball to be that the (r - 1) ball
was drawn from the same cell r = 2, ... , k. The probability of such a success
Sis l/n. If in this sequence there is no sample SS (event A), then in the sequence
of trials there is no cell receiving more than two balls successively. If qk is the
probability of A in a sequence (S and SC) of n trials, then Pk = qk-I (k > 2) and
qk satisfies the relation
n-1 In-I
qk = --qk-I
n
+ ~--qk-2'
n n
from which we have
n-l n-I
Pk = Pk(n) = --Pk-I
n
+ --qk-2'
n
k > 2, (1)

with PI = qo = 1, P2 = ql = 1. If AI' A2 are the roots of the characteristic


equation of (1), then Pn is given by
Pk = [(1 - A2))"~ - (I - AI))"~-I]/(AI - A2)'
(a) lim Pk = 0 as k ~ 00, because A~ ~ 0, )"~ ~ O.
(b) lim Pk(n) = 1 as n ~ 00, because Al ~ 1, A2 ~ O.
301. Applying Holder's inequality (9.5) with P = q = 1/2 for the random
variables IXI(t+h)/2 and IXI(t-h)/2 we have
o ~ h ~ t. (1)

Setting tl = t + h, t2 = t - h, equation (1) becomes


E 2 [IXI(t,+t 2)/2] ~ E(IXl t')E(IXl t2),
Solutions 231

or, taking logarithms,


2 log E[IXI(t,+t 2)/2] :::;; log E(IXl t ,) + log E(lxlt2).
Setting g(t) = log E(lxlt), we have

that is, g(t) satisfies (9.1) with A = 1/2 and hence it is convex since it is also
continuous. Since g(O) = 0, the slope g(t)/t of the line that connects the origin
with the point (t, g(t)) increases as t increases and hence
eg(t)/t = [E(IXlt)]l/t

is an increasing function of t.
302. CPo(t), as the real characteristic function of a random variable X, is
given by

CPo(t) = f: cos txfx(x) dx

and the required inequality for CPo follows by taking the expected value in the
inequality
1 - cos 2tX = 2(1 - cos 2 tX) :::;; 4(1 - cos tX).
For the other inequality we observe that if cp(t) is the characteristic function
of the random variable X then Icp(tW = cp(t)cp(t) is the characteristic function
of Xl - X 2 , where Xl' X 2 are independent having the same distribution as
X, and it is a real function. Taking CfJo(t) = Icp(tW in the above inequality, we
have the required inequality.
303. Applying the Schwartz inequality

to

cp(t) = f: (cos tx)f(x) dx

with
f(x) = cos txJlW, g(x) = JJ(X),
we have

ICPo(tW:::;; f oo
-00 cos 2 txf(x) dx =
foo
-00
1
2(1 + cos 2tx)f(x) dx.
Hence we have the required inequality.
232 Solutions

304. E(XiXj) = Cov(X i , Xj) (i -# j), implies that at most one of the Xi'
say Xl' has mean -# O. Since the central moments of order 3 are

we have E(X 2 X 3 X 4 ) = O. It must be shown now that for jointly normal


random variables Xl' X 2 , X 3 , X 4 with mean 0, we have

From the joint characteristic function of Xl' X 2' X 3' X 4

CP(tl, t 2 , t 3 , t 4 ) = exp [-! ±t 2 i=l j=l


(Jijtitj ]

and from the relation (cf. 6.15)

04cp(t l , t 2 , t 3 , t 4)1
= ,
~1234
ot ot ot ot
l 2 3 4 (0,0.0.0)

(1) is readily obtained.


305. By Exercise 241, the density of nX(I) is
n-l

fn(x) = (
1- ~) , 0< x < n,

and its characteristic function is

that is, the characteristic function of an exponential with density f(x) = e- X •

306. We have
E(2) = E(Z) = P[(Xj - X)(lj - Y) > 0]
and let p(Xj - X, lj - Y) = p*. Then (see Exercise 220),
( 1 arc sin p*
E Zj) = 2" + --n--'

307. Let Xl' X 2 , •.• , Xr be the selected lucky numbers and

The Xi are isonomic random variables with distribution

1
P[Xi = k] =-, i= 1,2, ... ,r, k= 1,2, ... ,n.
n
Solutions 233

Hence we have
1· n+1
E(Xi )=- L
k=-,
n k=1 2
(n + 1)(2n + 1)
E(Xt) = L• _k2=_(12+22+"'+n2)=
1 1
,
k=1 n n 6
n2 - 1
u 2 = Var(XJ = E(Xl) - [E(XJJ2 = -12-' (1)

Therefore we have

E(S) = r(n + 1)
r 2'
(2)
r
Var(Sr) = L Var(Xi) + r(r - 1) COV(Xi' X) = ru 2 + r(r - 1)pu 2,
i=1
where p = COV(Xi' Xj) = P(Xi' Xj), the correlation of Xi' lj. Since the joint
distribution of Xi' lj is independent of r, taking r = n, relation (2) gives

Var(S.) = varC~ X) = nu 2 + n(n - l)pu 2 = 0, (3)

because L7=1 Xi = L7=1 i = constant. Hence p = -lj(n - 1) and from (1), (2),
and (3) we find that

( r) -_ r(n2 - 1)
VarS
12
(1 r -
--- .
n- 1
1)
308. The probability generating function of X, because of the independence
of the Xi' is

p.(t) = n•
i=1
Px,(t) = n (Pit + qi)'

i=1
(1)

Taking logarithms in (1), we obtain


• •
log p.(t) = L log(pit + qi) = L log(1 + Pi(t - 1»
i=1 i=1
• (t - 1)2 •
= (t - 1) L
i=1
Pi -
2 i=1
L pt + .... (2)

By hypothesis

as n -+ 00

and the right-hand member of (2) tends to (t - 1).A., that is, p.(t) tends to the
probability generating function of the required Poisson.
234 Solutions

309. Putting q = A/n, we have

P[X = k] = ( n+k-1) qkp•


k
=
k-l(
n
1=0
1+-i)Ak(
, 1 - -A).,
n k. n
and since

(1 +~) ~ 1, as n ~ 00,

it follows that

k = 0,1,2, ....

310. X will have a Gamma distribution and the unconditional distribution


of Y is a continuous mixture of the Poisson with parameter A, which follows
the Gamma distribution of X (see Exercise 327).
311. The distribution of IX - f.11/(J is exponential with density
f(x) = e- X
and characteristic function (Exercise 202)
cp(t) = (1 - itrl.
Hence

~Y=
(J j=l
± X-"..j_-_f.1_1
_I

(J
has characteristic function
CP.(t) = cp·(t) = (1 - it)-·
and (2n/(J) Y has characteristic function (1 - 2iW·, that is, ofax~ •. Finally,
from E(X~.) = 2n, it follows that E(Y) = (J. Thus the so-called mean deviation
of the sample, that is, Y in the case of a Laplace distribution, gives an unbiased
estimator of the standard deviation (J of the distribution.

L Xn
2
312. _i .!!.. E(Xl) = 1 (WLLN) and S.Jn ~ N(O, 1) (CLT).

According to Exercise 255(c),


L
y" -+ N(O, 1).

Similarly for Z •.
313. The distribution of X(l)"'" X(r), given that X(r+l) = X, is the distribu-
tion of the ordered sample of r independent uniform Xl' ... , Xr in (0, x).
Moreover, Xl + .,. + Xr = X(l) + ... + X(r) = Y,. and hence the conditional
distribution of Y,., given that X(r+l) = x, is the distribution of the sum of
observations of a random sample from the uniform in (0, x). Thus, the condi-
Solutions 235

tional distribution of Y,.I X('+l)' given X(,+l) = X,


is the distribution of the sum
of r independent uniform variables X I lx, ... , X,lx in (0, 1).

i
314. By induction. For n = 2 we have, by the convolution formula,

f2(X) =
f: dy=x, x:s;1

S:-X dy = 2 - x,

= (2 ~ I)! {(~)x - G)(X - l)}.


Suppose the relation holds for n = k, that is,

J;(x)
k
= 1
(k - I)!
{(k)Xk-I_(k)(X_l)k-I+
0 1
... }.
We shall show that it holds for n = k + 1. Indeed, we have
J;.+l (x) = LX_ I
J;.(y)fl (x - y) dy.

For 0 < x < 1 the lower limit of integration is 0, while for k :s; x :s; k + 1 this
is equal to x-I; the upper limit is x for J;.(y) > 0 and 0 < y :s; k. Hence the
assertion follows.
The second expression of fn(x) follows from the first one if x is replaced by
n - x, since if Xl' ... , Xn are uniform in (0,1), then 1 - Xl' ... , 1 - Xn are
also uniform in (0, 1) and therefore the distribution of Sn is the same as that
ofn - Sn.
315. The probability generating function of Sn is P(t) = P;,(t) where
1 tm
P(t) = [ (; )
In = L tkp[Sn = k].
00

m - t k=O

From this we find

P[Sn = k] = ~ f (_I)k+i+im(~)(
m i=O I
-n .),
k - ml
(1)

where, in reality, the sum extends only over all i for which the relation
k - mi ~ 0 holds.
316. Because of (1) of Exercise 315, the required probability is
P[Sn = k - n] = P[S3 = 10 - 3] = P[S3 = 7]

= ;3 [(_1)7G)( ~3) + (-1)14G)( ~3)J


1 1
= 63 (36 - 9) = s.
236 Solutions

317. Let C l , ... , Cm +l denote the r = m + 1 boxes. Then


r
(a) P[X I = Xl' ..• , Xm = Xm] = L P[Ai' Xl
i=l
= Xl' ... , Xm = Xm], (1)

where, e.g.,
r(rN - Xo)
P[A"X l =Xl> ... 'Xm =xm]= m rxo-rN-l, (2)
r(N + 1) TI (N - x;)!
i=l

where we put Xo = Xl + ... + X m, since Cr is found empty (event Ar) and


Xl' ... , Xmmatches in the remaining m boxes requires that N - Xi matches are
taken from Ci (i = 1,2, ... , m) before the (N + l)st selection of Ci (i = 1,2,
... , m) (negative multinomial, see (s.7» and

.. 1
P (0 f selectton of Ci III every draw) = -, i = 1,2, ... , r.
r

But, by symmetry, the r probabilities in (1) are equal and therefore


P[XI = Xl' ... , Xm = Xm]
= rP[A k , Xl = Xl' ... , Xm = Xm]
r(rN - XO)
_ _ _ _m
_ _ _ _ rXo-rN,
Xi = 0, 1, ... , N.
r(N + 1) TI (N - Xi)!
i=l

(b) Now N - Xi matches must be drawn from Ci before the Nth selection
of Cr. Thus

(Here Xi = 1,2, ... , N (i = 1,2, ... , m).)


318. The required probability, sayan is equal to
N N
ar = L ... L P[XI = Xl'· .. , Xm = Xm],
xI:;:1 xm=l

where, putting Xo = Xl + ... + X m , we have (cf. Exercise 317)


r(rN - xo) m pf- x ,
P[XI = Xl' ..• , Xm = Xm] = r(N) I] (N - X;)!

In the special case r = 2, az can be written as an incomplete B function (see


Exercise 329):

2
r(2N)
a =- -
r(N)
f0
P2
X N- l (1 - X)N-l dx == I (N N)
pz ' •
Solutions 237

319. By definition of independence the conditions are necessary. That they


are sufficient follows from the fact that the second one gives
P[XI = Xl' X 2 = X2] = P[XI = Xl]' P[X2 = x 2] for all (Xl' x 2 ). (1)

The first one gives


P[XI = Xl' X 2 = X2, X3 = X3]
P [X 3 = Xl' X2 = X 2, Xl = Xl] = ---=---==-::----=--=-----=-=-=----=--:=----=-=-
P[XI = Xl' X 2 = X2]
= P[X3 = X3].
This with (1) gives the condition of complete independence
P[XI = Xl' X 2 = X2, X3 = X3] = P[XI = X I ]P[X 2 = X2]P[X 3 = X3]
for all (Xl' X2, X3)' For n variables Xl' ... , X n, the conditions become
P[Xi = x;lXj = Xj' j = 1,2, ... , i - I ] = P[Xi = xJ, i = 1,2, ... , n.

320. E(S;;I) exists because P[Sn- 1 < Xli] = 1. Since E(XJSn) = a (i = 1,


2, ... , n), we have

1 = E( X.) = L E (X)
L~n
~ = nE (X)
n
~ .
Sn
i=l Sn Sn
i=1

Hence,

a= E(::) = n- l .

321. By Exercise 320, we have

Sm) (Xi) m for m:S;; n,


E ( Sn = mE Sn =~
while, for m z n, since S;;l and X n+1' •.• , Xm are independent, we have

E(Sm)
Sn
= E(I + ~.
Sn
fl=n+1
Xi) = 1 + (m - n)E(Xi)
Sn

= 1 + (m - n)E(X;)E GJ.
322. (a) The required probability P is given by

P= L 00

n=l
P(An) =
n=l
L -31 (2)n-1
00
-3 = 1,
where An is the event that it gets stuck exactly after n steps, i.e., at time t = n
(geometric distribution with P = 1/3).
(b) Let Pa, Pb, Pc be the probabilities that the caterpillar eventually gets
stuck coming from A, B, C, respectively. Then we have
Pa + Pb + Pc = 1,
238 Solutions

since the caterpillar goes in one step to B with probability 1/3 and having
gone there it has the same probability of getting stuck (coming from B) as if
it had started at A. Hence
Pb = Pc = 1/5, Pa = 3/5.

323. (a) (i) E(Xn) = naE(cos 0i) = na(2n)-1 I2l< cos °dO = 0.
Similarly, E(Y,,) = 0,
Var(Xn) = E(X;) = na 2 E(cos 2 0i) + n(n - l)a 2 E(cos 0i cos OJ)

= na 2 E(cos 2 0;), (1)


since the independence of 0i gives
E(cos 0i cos 0) = E(cos Oi)E(cos OJ) = 0, i ¥= j.
From (1), we find
Var(Xn) = tna 2 • Similarly, (2)
(ii) From (1) and (2) we find
E(R;) = na 2 •
(b) Cov(Xn' Yn) = E(Xn Y,,) = na 2 [E(sin 0i cos 0i)
+ (n - I)E(sin 0i cos OJ)] = 0.
Hence the coordinates X n , Yn of the position of the flea after n jumps are
uncorrelated. However, we shall show that they are not independent. Let
n = 1. Then we have, e.g.,

that is, Xl' Y1 are not independent. Similarly, by induction, the dependence
of X n , Y" is shown.
(c) By the CLT we have

2
L
Xn -+N (
0'2
na ) '

Hence X n , Y" are asymptotically normal and independent as well, because they
are uncorrelated. Then, the asymptotic (as n -+ (0) distribution of Rn , by
Exercise 221, has the density

I"
JR
() _
r - -2r2 e-"Ina' .
n na

From the above relation we conclude that the expected distance of the flea
Solutions 239

from the origin (for large n) is

E(Rn) = fo~.
324. Let ~j = (Xl' "., X j - l , X j +1, "., Xn) (j = 1,2, .'" n). The density of
~j is

where

Hence, any subvector of ~j and therefore any subset of Xl"'" Xn has the multi-
normal distribution with independent components. Yet, X = (X I ' ... , Xn)
with density f(x) does not have an n-dimensional normal distribution and X I,
X 2 , ••• , Xn are not completely independent.
Remark. If X I ' ... , Xn are uncorrelated, that is, P(Xi' X) = 0 (i # j), and all
the marginal distributions are normal, then X I, X 2, ... , Xn are jointly normal
if and only if XI' X 2 , " . , Xn are completely independent.
325. (a) Because of the uniform distribution of the angle (J, Exercise 288,
the mean number of crossings of the needle by horizontal or vertical lines is
the same, and equal to the probability of crossing one of the parallels, i.e.,
2f1/na. But the mean number of crossings of the lines of the table is equal to
the sum of the expected crossings ofthe horizontal and vertical lines, i.e., 4f1/na.
Thus for a needle having length equal to the side of the squares, the mean
number of crossings is 4/n ~ 1.27.
(b) If the needle has arbitrary length I, let us imagine that this is divided
into n equal pieces of length less than 2a. Each of the pieces separately gives
mean number of crossings equal to 21/nna. But the mean of the sum is equal
to the sum of the means and hence the required mean number of crossings by
the needle is 21/na. Throwing the n pieces of the needle separately instead of
throwing the needle as a solid body does not influence the mean.
Note. Use the result for the estimation of the value of n, e.g., throwing a
toothpick on a flat surface divided into little squares.
326. (Cacoullos, JASA, 1965)
(a) We have

G(u) = P [X - Y ]
2JXY
~ uJ = 2 r 21
n
(n/)
2 f (x-Yl=2uJ~Y
(xy)(n/2 l -l e -(x+Yl/2 dx dy,
240 Solutions

where putting first x = p COS 2 0, Y = P sin 2 0, and then w = cot 20, we have
by the Legendre duplication formula,
22n-1 r(n)r(n + t) = fir(2n),

r(~)fU dw
G(u) = fir(~) -00 (1 + w2)(n+1)/2'

Hence we obtain the required distribution of Z. Since Z can be written


as

we have the relation between tn and Fn,n'


(b) For 0 < IX < 0.5 we have (for Fn,n = 1, tn = Z = 0)
P[O < tn < tn(lX)] = P[l < Fn,n < Fa], (1)

where Fa is the solution F > 1 of the equation

tn(lX) = f( JF - ft). (2)


that is,
1 + t;(IX).
n

From (1) and from the relation P[Fn,n ::; 1] = 1/2, we have
P[Fn,n ::; Fa] = 1 - IX

and hence Fn,n(lX) = Fa. For 0.5 < IX < 1, as is well known, we have
tn(lX) =- tn(1 - IX) and Fn,n(lX) = Fn~~(1 - IX).
327. According to the generalized theorem of total probability, we
have

Pk = J e-
OO.A.
o
k
A _ f(.A.) d.A. = _a_
k!
P

r(p)k!
Joo .A.k+p-1 e
0
-(a+1)A d.A.

aP r(k +
= r(p)k! (a + l)k+ P =
p) (-p)
k (- p)
k
qP,

where we made use of r(x + 1) = xr(x) (x > 0). Hence the negative binomial
distribution was obtained as a mixture of the Poisson and Gamma distribu-
tions.
Solutions 241

328. By definition we have


E[Var(XI Y)] = E[E(X21 Y) - {E(XI y)}2]

= E[E(X2IY)] - E[E(Xly)]2, (1)


Var[E(XI Y)] = E[E(XI y)]2 - {E[E(XI y)}2, (2)
and since
E[E(XI Y)] = E(X),
Adding (1) and (2), we have
E(X2) - {E(X)}2 = Var(X).
329. The distribution function of X(k), by Exercise 241, is

Fk(p) = P[X(k) :::; p] = f: !xk(x) dx


(1)
= 1 fP Xk- 1(1 - x)n-k.
B(k, n - k + 1) 0

On the other hand, we have that X(k) :::; P if and only if at least k of the Xi are
less than p. Since the Xi are independent and P[Xi :::; p] = p, we have

(2)

From (1) and (2) the assertion follows.


Bibliography

1. Cacoullos, T. (1969). A Course in Probability Theory (in Greek). Leousis-


Mastroyiannis, Athens.
2. Cramer, H. (1946). Mathematical Methods of Statistics. Princeton University
Press, Princeton, NJ.
3. Dacunha-Castelle, D., Revuz, D., and Schreiber, M. (1970). Recueil de Problemes
de Calcul des Probabilites. Masson Cie, Paris.
4. Feller, W. (1957). Introduction to Probability Theory and its Applications, Vol. I.
Wiley, New York.
5. Fisz, M. (1963). Probability Theory and Mathematical Statistics. Wiley, New York.
6. Gnedenko, B.V. (1962). The Theory of Probability. Chelsea, New York.
7. Mosteller, F. (1965). Fifty Challenging Problems in Probability. Addison-Wesley,
Boston, MA.
8. Parzen, E. (1960). Modern Probability Theory and its Applications. Wiley, New
York.
9. Rahman, N.A. (1967). Exercises in Probability and Statistics. Griffin, London.
10. Sveshnikov, A.A. (1968). Problems in Probability Theory. Saunders, Philadelphia.
Index

Addition theorem for probabilities 4 Cartesian product 6


Arc sine distribution 58, 59 Cauchy distribution 19, 58
Axioms of probability theory 4 characteristic function of 197
infinite divisibility of 54
Banach match box problem 80 mixture of 197
Bayes formula 5,41 Central limit theorems 66-67
Behrens-Fisher problem 61 Central moments 25
Bernoulli, Daniel 93 Characteristic function 48, 50, 117
Bernoulli trials 18, 65, 67, 79, 105, 115, continuity theorem for 50, 51, 217
118, 120 inversion formula for 49,51
Bernstein polynomial III Chebyshev's inequality. See Inequality
Bertrand's paradox 36 Chi-squared (X 2 -) distribution 19
Beta distribution 19 approximation by normal 111
characteristic function of 105 approximation of noncentral X2 52
in evaluating the binomial generalized X2 -distribution 96
distribution function 82 Circular normal distribution 59, 62
Beta function 19 Combinatorics 5, 8
Billiards 104 Compound distribution 47,52, 111
Binomial coefficients 5, 8, 9 Poisson 48
Binomial distribution 18 Conditional
factorial (probability) generating characteristic function 54
function 51, 121 density 40
normal approximation of 20,21,45, distribution 40
53 probability 4
reproductive property of 60 Condorcet, Marquis De 93
Birthdays 11, 12, 35, 88 Convergence 63-69, 109-114
Bochner-Khintchine theorem 117 in distribution (law) 64, 111-114
Borel, E. 3, 65 in probability 63, 113
Borel-Cantelli lemma 115,117 in quadratic mean 63, 113
Bose- Einstein statistics 12 with probability one (strongly, almost
Bridge 11,30 surely) 63, 64, 113, 117
Buffon's needle problem 76 Convex function 70, 75
246 Index

Convolution 56 Factorial moment 25, 26


binomial 60 generating function 25
exponential 58 Feller, W. 33,67,80,90, 115, 120,200,
Poisson 60 243
uniform 59, 79, 120 Finite population correction 163
Correlation coefficient 41
Coupon collector's problem 35, 52 Gamma distribution 19,53
Covariance 41 approximation by normal 69
matrix 42 characteristic function of 53
Cramer, H. 49, 50, 243 in evaluating the Poisson distribution
Craps, game of 33 function 91
Cumulant generating function 49 infinite divisibility of 54
Cumulants 49, 53 mixing for a Poisson 79, 82
Cumulative distribution function. See Gamma function 19
Distribution function Gauss-Laplace distribution. See
Normal distribution
DeMoivre-Laplace theorem 67, 115 Geiger- Muller counter 109
Difference equations 72-73,76-77, 121 Generating function 47, 50, 10 1-105
Difference of sets 3 factorial moment 25,47
probability of 4 moment 26, 48
symmetric 10 probability 25,47
Dirac b-function 103 Geometric distribution 18
Dirichlet distribution 43, 44, 60, 97 Geometric probabilities 72, 75, 76,
Dispersion matrix. See Covariance 90
matrix Gnedenko, B.Y. 65, 117, 120, 243
Distribution
continuous 17, 39, 43 Hamel's functional equation 105
discrete 17, 39, 42 Huyghens, C. 14,92,200
singular 42 Hypergeometric distribution 6, 18,28,
Distribution function 17, 39, 40 60
Distribution of functions of random double 43
variables 22, 23, 55-62, 106-109 multiple 44
Dodge, H.F. 179
Independence, of normal sample mean
Elliptic symmetry 109 and variance 61
Entropy 100 Independent events 5
Erehwon 35 completely 5
Erlang distribution 90,91 Independent experiments 5
Euler's constant 185 Independent random variables 42,51
function 33 pairwise 44
problem 93 Inequality 70-71, 74-75
Event Berge 75
impossible Bonferoni 10
sure 1 Cantelli 74
Expected value 24,28,41 Cauchy-Schwarz 26, 71
approximate 98 Chebyshev (Binayme) 64, 111
geometric interpretation 27 Gauss 74
limiting 110 Hajek-Renyi 66
Exponential distribution 19 Holder 71
as a limit of extremes 122 Jensen 70
cumulants of 53 for matrix functions 75
Kolmogrov 66, 119
F -distribution 20, 109 Markov 64
relation to t 81 Minkowski (triangle) 71
Index 247

Infinitely divisible distribution 50 Multivariate normal distribution 43,


Iterated logarithm, law of 115 44,94,95,97
characteristic function of 103
Khinchine's, law of the iterated conditional distributions in 96
logarithm 115 distribution of linear functions 62
weak law of large numbers 65 distribution of quadratic forms 100
Kolmogorov's, probability axioms 4 marginal distributions of 81,96
three-series theorem 115, 117 moments of 78
zero-one law 116 probability in ellipsoid 95

Negative binomial distribution 18


Laplace, P.S. 4, 93 as a compound Poisson 34
Laplace distribution 19, 79 as a mixture of Poisson and gamma
characteristic function of 102 79,82
moments of 102 infinite divisibility of 54
Laws oflarge numbers Negative multinomial 43
strong 65, 67 as a mixture of a multiple Poisson
weak 64,67 and a gamma distribution 54
Legendre duplication formula for the moments of 54
r -function 240 Newton 35, 88
Levy, P. 50, 66 Normal distribution 19
Lindeberg-Feller theorem 67,217 approximations by 66
Lognormal distribution 23, 60 asymptotic evaluation of the
Lyapunov's theorem 66 distribution function of 116
characterization of 105
Marginal distribution 40 folded 32
Mathematical induction 7 moments of 196
Maxwell-Boltzmann statistics 90
Mean. See Expected value Occupancy number 6, 11
Median 25, 26 Order statistics 61
Mixed distribution 29 from exponential 108
Mixture of distributions 53 from uniform 79, 82
Mode 45,74 Orthogonal transformation 53, 108
Moment-generating function 26
Moments 25 Paccioli 92
about the origin 25 Pap test 15
central 25 Pareto distribution 68
condition for existence of 28 Pascal, distribution. See Negative
mixed 41,51 binomial distribution,
Monte Carlo method 114 points 33, 89
Montmort, P.R. 89, 93 triangle 5, 8
Mosteller, F. 243 Pepys, Samuel 35
Multinomial coefficients 6 Percentiles 25
Multinomial distribution 43 Permutations 5
conditional of 44 cyclical 127
moments of 54 Poincan':'s formula 4
multiple Poisson limit of 45 Poisson distribution 18
Multiplication formula for probabilities approximation by normal 110
4 approximation of binomial 20, 21,
Multiplication principle 6 45,54
Multivariate distribution 39 approximation of Pascal 102
characteristic function of 50 compound 48
probability density of 40 in evaluating the gamma distribution
probability function of 39 function 90, 91
248 Index

Poisson distribution (cont.) Runs, maximum length 118


multiple 45, 54
normal approximation of 217 Sampling
probability generating function 51 with replacement 13, 28, 31
truncated 29 without replacement 6, 13,28,31,32
Poisson, S., a theorem of 87 Sigma (Borel) field 3
Poker II Singular distribution 42
Polya, G. 93 Smith, John 88
Probability Spherical coordinates 97
conditional 4 Spherically symmetric distribution 53,
density function 17, 40 96-100
differential 18 Spinoza, B. 92
function 17, 39 Standard deviation 25
generating function 25,47, 50 Standard normal distribution 29
integral transformation 59 asymptotic evaluation of 116
space 63 Strong law oflarge numbers 65, 67
Probable error 95 Student's t-distribution 19
Pseudorandom numbers 59. See also elliptic 109
Random numbers relation to F 81

Quadratic functions of normal random Tolerance limits 60


variables, distribution of 100 Total probability formula 5, 32, 40
Quality control 34, 35 Triangular distribution 21
Quantiles 25 Trinomial distribution 188
Truncated
Rahman, N.A. 179, 243 Poisson distribution 29
Random digits 22, 116 random variable 1I5, 119
Random numbers 22,59, 114
Random variables 17 Uniform distribution 19
complex 48 in an ellipse 96
continuous 17,39,43 in a simplex 44,100
discrete 17, 39,42 multivariate 43
isonomic 208 n-fold convolution of 79
orthogonal 46 on the unit sphere 98
vector 39 Uniqueness theorem for characteristic
Range of a random sample 61 functions 49
Rayleigh distribution 205
Rectangular distribution. See Uniform Variance 25
distribution approximation of 98
Regression, function 42 of a product of random variables 89
linear 108 Venn diagram 7 .
Relations between modes of convergence
64 Weierstrass approximation theorem
Round-off error 69 III

Das könnte Ihnen auch gefallen